You are on page 1of 115

PRESTORMING – 2023-24

TEST 19 – POLITY FULL TEST - EXPLANATION


INDEX
1. ............................................................................................................................................ 1
2. ............................................................................................................................................ 1
3. ............................................................................................................................................ 2
4. ............................................................................................................................................ 3
5. ............................................................................................................................................ 4
6. ............................................................................................................................................ 6
7. ............................................................................................................................................ 7
8. ............................................................................................................................................ 8
9. ............................................................................................................................................ 9
10. .......................................................................................................................................... 10
11. .......................................................................................................................................... 12
12. .......................................................................................................................................... 12
13. .......................................................................................................................................... 14
14. .......................................................................................................................................... 15
15. .......................................................................................................................................... 17
16. .......................................................................................................................................... 18
17. .......................................................................................................................................... 18
18. .......................................................................................................................................... 20
19. .......................................................................................................................................... 21
20. .......................................................................................................................................... 22
21. .......................................................................................................................................... 23
22. .......................................................................................................................................... 24
23. .......................................................................................................................................... 25
24. .......................................................................................................................................... 26
25. .......................................................................................................................................... 27
26. .......................................................................................................................................... 28
27. .......................................................................................................................................... 28
28. .......................................................................................................................................... 29
29. .......................................................................................................................................... 30
30. .......................................................................................................................................... 31
31. .......................................................................................................................................... 32
32. ……… .................................................................................................................................. 33
(1-A)
SIA-A-GS I
33. .......................................................................................................................................... 34
34. .......................................................................................................................................... 35
35. .......................................................................................................................................... 36
36. .......................................................................................................................................... 37
37. .......................................................................................................................................... 38
38. ………….. ............................................................................................................................. 39
39. .......................................................................................................................................... 40
40. .......................................................................................................................................... 41
41. .......................................................................................................................................... 43
42. .......................................................................................................................................... 45
43. .......................................................................................................................................... 46
44. .......................................................................................................................................... 47
45. ……………….......................................................................................................................... 48
46. .......................................................................................................................................... 49
47. .......................................................................................................................................... 50
48. .......................................................................................................................................... 51
49. .......................................................................................................................................... 52
50. .......................................................................................................................................... 53
51. .......................................................................................................................................... 55
52. .......................................................................................................................................... 56
53. .......................................................................................................................................... 57
54. .......................................................................................................................................... 58
55. .......................................................................................................................................... 59
56. .......................................................................................................................................... 60
57. …………… ............................................................................................................................ 63
58. .......................................................................................................................................... 64
59. .......................................................................................................................................... 65
60. .......................................................................................................................................... 67
61. .......................................................................................................................................... 67
62. .......................................................................................................................................... 68
63. .......................................................................................................................................... 69
64. .......................................................................................................................................... 70
65. .......................................................................................................................................... 72
66. .......................................................................................................................................... 73
(2-A)
SIA-A-GS I
67. .......................................................................................................................................... 74
68. .......................................................................................................................................... 75
69. .......................................................................................................................................... 77
70. .......................................................................................................................................... 78
71. .......................................................................................................................................... 79
72. .......................................................................................................................................... 80
73. ………. ................................................................................................................................. 81
74. .......................................................................................................................................... 82
75. .......................................................................................................................................... 83
76. .......................................................................................................................................... 84
77. .......................................................................................................................................... 84
78. .......................................................................................................................................... 85
79. .......................................................................................................................................... 86
80. .......................................................................................................................................... 87
81. .......................................................................................................................................... 88
82. .......................................................................................................................................... 89
83. .......................................................................................................................................... 91
84. .......................................................................................................................................... 92
85. .......................................................................................................................................... 93
86. .......................................................................................................................................... 94
87. .......................................................................................................................................... 95
88. .......................................................................................................................................... 96
89. .......................................................................................................................................... 97
90. .......................................................................................................................................... 98
91. .......................................................................................................................................... 99
92. ........................................................................................................................................ 100
93. ........................................................................................................................................ 101
94. ........................................................................................................................................ 103
95. ........................................................................................................................................ 104
96. ........................................................................................................................................ 105
97. ........................................................................................................................................ 107
98. ........................................................................................................................................ 108
99. ........................................................................................................................................ 109
100. ........................................................................................................................................ 110
(3-A)
SIA-A-GS I
(4-A)
SIA-A-GS I
1. Consider the following statements with respect to the no-confidence motion:
1. Neither the Constitution nor the Rules of Rajya Sabha provide for the No-confidence motion.
2. The House of People is not empowered to entertain the no-confidence motion.
Which of the statements given above is/are correct?
(a) 1 only
(b) 2 only
(c) Both 1 and 2
(d) Neither 1 nor 2
EXPLANATION:
Article 75 of the Constitution says that the council of ministers shall be collectively responsible to the
Lok Sabha. It means that the ministry stays in office so long as it enjoys the confidence of the majority
of the members of the Lok Sabha. In other words, the Lok Sabha can remove the ministry from office by
passing a no-confidence motion. The motion needs the support of 50 members to be admitted. For testing
the collective responsibility, the rules of Lok Sabha (Not the Rajya Sabha) provide a particular mechanism
for a motion of no-confidence. Thus, Neither the Constitution nor the Rules of Rajya Sabha provide for
the No-confidence motion. So, Statement 1 is correct.
Any member of the Lok Sabha can move a no-confidence motion. It can be moved only in the Lok Sabha
(House of People) and not Rajya Sabha. Rule 198 of the Rules of Procedure and Conduct of Lok Sabha
specifies the procedure for moving a no-confidence motion. Thus, the House of People is empowered to
entertain the no-confidence motion. So, Statement 2 is not correct.
ADDITIONAL INFORMATION:
NO-CONFIDENCE MOTION
About ➢ A government can function only when it has majority support in the Lok Sabha.
➢ The party can remain in power when it shows its strength through a floor test
which is primarily taken to know whether the executive enjoys the confidence of
the legislature.
➢ If any member of the House feels that the government in power does not have a
majority, then he/she can move a no-confidence motion.
➢ If the motion is accepted, then the party in power has to prove its majority in the
House. The member need not give a reason for moving the no-confidence motion.
How it works ➢ Any member of the House can move a no-confidence motion. It can be moved only
in the Lok Sabha and not Rajya Sabha.
➢ Rule 198 of the Rules of Procedure and Conduct of Lok Sabha specifies the
procedure for moving a no-confidence motion.
➢ The member has to give written notice of the motion before 10 am, which will be
read out by the Speaker of the House.
➢ A minimum of 50 members have to accept the motion, and accordingly, the
Speaker will announce the date for discussion of the motion.
➢ The allotted date has to be within 10 days from the day the motion is accepted.
Otherwise, the motion fails, and the member who moved the motion will be
informed about it.
➢ If the government is not able to prove its majority in the House, then the
government of the day has to resign.

2. The Preamble to the Constitution of India is


(a) a source of power to the legislature but doesn't place prohibitions on its exercise
(b) not a source of power to the legislature and doesn't place prohibitions on its exercise
(c) a source of power to the legislature and places certain prohibitions on its exercise
(d) not a source of power to the legislature but places certain prohibitions on its exercise

1
EXPLANATION:
The Preamble to the Indian Constitution is based on the 'Objectives Resolution,' drafted and moved by
Pandit Nehru and adopted by the Constituent Assembly. The Constitution of India begins with a
Preamble. The Preamble contains the ideals, objectives, and basic principles of the Constitution. The
Preamble to the Constitution has played a predominant role in shaping the destiny of the country. The
Preamble is of great utility as a guide to the interpretation of the constitutional provisions of the
Constitution.
➢ The Preamble is neither a source of power to the legislature nor a prohibition upon the powers of
the legislature.
➢ It is non-justiciable; that is, its provisions are not enforceable in courts of law.
So, Option (b) is correct.

ADDITIONAL INFORMATION:
PREAMBLE TO THE INDIAN CONSTITUTION
History ➢ In the Berubari Union case (1960), the Supreme Court said that the Preamble shows
the general purposes behind the several provisions in the Constitution and is thus a
key to the minds of the makers of the Constitution. Further, where the terms used in
any article are ambiguous or capable of more than one meaning, some assistance at
interpretation may be taken from the objectives enshrined in the Preamble. Despite
this recognition of the significance of the Preamble, the Supreme Court specifically
opined that the Preamble is not a part of the Constitution.
➢ In the Kesavananda Bharati case (1973), the Supreme Court rejected the earlier
opinion and held that the Preamble is a part of the Constitution. It observed that the
Preamble is of extreme importance, and the Constitution should be read and
interpreted in the light of the grand and noble vision expressed in the Preamble.
➢ In the LIC of India case (1995) also, the Supreme Court again held that the Preamble
is an integral part of the Constitution. Like any other part of the Constitution, the
Preamble was also enacted by the Constituent Assembly, but after the rest of the
Constitution was already enacted. The reason for inserting the Preamble at the end
was to ensure that it conformed to the Constitution as adopted by the Constituent
Assembly. While forwarding the Preamble for votes, the President of the Constituent
Assembly said, 'The question is that Preamble stands part of the Constitution.' The
motion was then adopted. Hence, the current opinion held by the Supreme Court that
the Preamble is a part of the Constitution is in accordance with the opinion of the
founding fathers of the Constitution.

3. With reference to the Special Officer for Linguistic Minorities, consider the following statements:
1. It was created in pursuance of the provision of Article 350 of the Original Constitution.
2. At the Central level, the Commissioner for Linguistic Minorities falls under the Ministry of Home
Affairs.
3. The Constitution of India mandates that the Union and the State Governments shall consult the
Special Officer on all major policy matters affecting linguistic minorities.
How many statements given above are correct?
(a) Only one
(b) Only two
(c) All three
(d) None

2
EXPLANATION:
Originally, the Constitution of India did not make any provision with respect to the Special Officer for
Linguistic Minorities. Later, the States Reorganisation Commission (1953–55) made a recommendation
in this regard. Accordingly, the Seventh Constitutional Amendment Act of 1956 inserted a new Article
350B in Part XVII of the Constitution. Hence, the Special Officer for Linguistic Minorities was not an
original provision of the Constitution, but it was added through the Constitutional Amendment Act.
So, Statement 1 is not correct.
In pursuance of the provision of Article 350-B of the Constitution, the office of the Special Officer for
Linguistic Minorities was created in 1957. He is designated as the Commissioner for Linguistic
Minorities. At the Central level, the Commissioner falls under the Ministry of Minority Affairs (not under
the Ministry of Home Affairs). Hence, he submits the annual reports or other reports to the President
through the Union Minority Affairs Minister. So, Statement 2 is not correct.
According to the Constitution of India, It does not mandate the Union and the State Governments to
consult the Special Officer on all major policy matters affecting linguistic minorities. However, the Union
and the State Governments need to consult the Special Officer of SC, ST and BC on all major policy
matters affecting them. So, Statement 3 is not correct.
ADDITIONAL INFORMATION:
LINGUISTIC MINORITIES
About The linguistic minorities are considered as a group or collectivities of individuals residing
in the territory of India or any part thereof having a distinct language or script of their
own. The word linguistic minorities has not been defined in the Constitution of India.
Commissioner ➢ The Commissioner of Linguistic Minorities has his headquarters in Allahabad (Uttar
of Linguistic Pradesh).
Minorities ➢ He has three regional offices at Belgaum (Karnataka), Chennai (Tamil Nadu) and
Kolkata (West Bengal). An Assistant Commissioner heads each.
➢ The Commissioner is assisted at headquarters by a Deputy Commissioner and an
Assistant Commissioner. He maintains liaison with the State Governments and
Union Territories through nodal officers appointed by them.
➢ To promote and preserve linguistic minority groups, the Ministry of Minority Affairs
has requested the State Governments / Union Territories to give wide publicity to
the constitutional safeguards provided to linguistic minorities and to take necessary
administrative measures.
➢ The state governments and UT Administrations were urged to accord priority to the
implementation of the scheme of safeguards for linguistic minorities.
It shall be the duty of the Special Officer to investigate all matters relating to the
safeguards provided for linguistic minorities under this Constitution and report to the
President upon those matters at such intervals as the President may direct. The
President shall cause all such reports to be laid before each House of Parliament and
sent to the Governments of the States concerned.

4. Consider the following statement with respect to India’s First Night Sky Sanctuary:
1. The reserve aims to promote livelihoods through eco-friendly activities like Astro tourism.
2. It is located within the Nubra Shyok Wildlife Sanctuary.
3. It is the world’s highest-located site for optical, infrared, and gamma-ray telescopes.
How many of the statements given above are correct?
(a) Only one
(b) Only two
(c) All three
(d) None
3
EXPLANATION:
A Dark Sky Reserve is a designation given to a place that has policies in place to ensure that a tract of
land or region has minimal artificial light interference. The International Dark Sky Association is a U.S.-
based non-profit that designates places as International Dark Sky Places, Parks, Sanctuaries and
Reserves, depending on the criteria they meet. Several such reserves exist around the world but none so
far in India.
The Night Sky Reserve aims to promote Livelihood through eco-friendly activities of Astro tourism, spread
awareness about astronomy and boost scientific research with reduced artificial light and wildlife
conservation. So, Statement 1 is correct.
The Dark Sky Reserve will be located at Hanle village in Eastern Ladakh as a part of Changthang Wildlife
Sanctuary (not in the Nubra Shyok Wildlife Sanctuary). It will boost Astro-tourism in India and will be
one of the world’s highest-located sites for optical, infra-red, and gamma-ray telescopes.
The Night Sky Reserve is spread over 1,073 square kilometres and located within the Changthang Wildlife
Sanctuary. It is adjacent to the Indian Astronomical Observatory, the second-highest optical telescope
in the world, of Indian Institute of Astrophysics at Hanle at an elevation of 4500 metres.
So, Statement 2 is not correct and Statement 3 is correct.
ADDITIONAL INFORMATION:
HANLE DARK SKY RESERVE (HDSR)
About ➢ Hanle Dark Sky Reserve (HDSR) aims to control man-made light pollution in the area
in order to preserve the pristine dark skies for astronomical research.
➢ A dark sky is the night sky as nature meant for it to be, without any light pollution.
➢ The Reserve is promoted as a tourist destination for enjoying the night sky and also
helps in socio-economic development of the local villages.
➢ HDSR is India’s first dark sky region which is centred at Hanle in Eastern Ladakh,
around the Indian Astronomical Observatory.
➢ It is India's first Night Sky Sanctuary is located within the Changthang Wildlife
Sanctuary.
➢ The Wildlife Sanctuary is adjacent to the Indian Astronomical Observatory, the second-
highest optical telescope in the world.
➢ This Dark Sky Reserve is among only the 15 or 16 of its kind in the world that will offer
a spectacular view of the night sky.
➢ The Star Party is also likely to evolve into an annual event.
➢ Bortle scale - The Bortle scale is a 9 level numeric scale that measures the night sky's
brightness of a particular location.
➢ The scale ranges from Class 1, the darkest skies available over the earth, through to
Class 9, which denotes the pale, light-marred skies over the insides of cities.
➢ HDSR region falls under the Bortle Class 1.

5. Consider the following statements with reference to the State Information Commission (SIC):
1. The Chairman and the members of SIC are appointed by the President but removed by the
Governor.
2. The chairman is not eligible for reappointment after the expiry of the tenure.
3. The salary, allowances and other service conditions of the members of SIC are determined by the
Parliament.
How many of the above statements are correct?
(a) Only one
(b) Only two
(c) All three
(d) None

4
EXPLANATION:
The State Information Commission is a high-powered an independent body that looks into the complaints
made to it and decides the appeals. It entertains complaints and appeals pertaining to offices, financial
institutions, public sector undertakings, etc., under the concerned state government.
The Commission consists of a State Chief Information Commissioner and not more than ten State
Information Commissioners. They are appointed by the Governor (not by the President) on the
recommendation of a committee consisting of the Chief Minister as Chairperson, the Leader of Opposition
in the Legislative Assembly and a State Cabinet Minister nominated by the Chief Minister.
The Governor can remove the State Chief Information Commissioner or any State Information
Commissioner from the office. So, Statement 1 is not correct.
The State Chief Information Commissioner and a State Information Commissioner shall hold office for
such term as prescribed by the Central Government or until they attain the age of 65 years, whichever
is earlier. They are not eligible for reappointment after the expiry of the tenure.
So, Statement 2 is correct.
The salary, allowances and other service conditions of the State Chief Information Commissioner and a
State Information Commissioner shall be such as prescribed by the Central Government (It is not
determined by the Parliament). But, they cannot be varied to his disadvantage during service. So,
Statement 3 is not correct.
ADDITIONAL INFORMATION:
STATE INFORMATION COMMISSION (SIC)
Powers and It is the duty of the Commission to receive and inquire into a complaint from any person:
Functions • who has not been able to submit an information request because of the non-
appointment of a Public Information Officer;
• who has been refused information that was requested;
• who has not received a response to his information request within the specified
time limits;
• who thinks the fees charged are unreasonable;
• who thinks the information given is incomplete, misleading or false;
• Any other matter relating to obtaining information.
➢ The Commission can order an inquiry into any matter if there are reasonable
grounds (suo-moto power).
➢ While inquiring, the Commission has the powers of a civil the court in respect of the
following matters:
• summoning and enforcing the attendance of persons and compelling them to give
oral or written evidence on oath and to produce documents or things;
• requiring the discovery and inspection of documents;
• receiving evidence on affidavit;
• requisitioning any public record from any court or office;
• issuing summons for examination of witnesses or
• documents; and
• Any other matter which may be prescribed.
➢ During the inquiry of a complaint, the Commission may examine any record which
is under the control of the public authority, and no such record may be withheld
from it on any grounds. In other words, all public records must be given to the
Commission during inquiry for examination.

5
➢ The Commission has the power to secure compliance of its decisions from the public
authority. This includes:
• providing access to information in a particular form;
• directing the public authority to appoint a Public Information Officer where none
exists;
• publishing information or categories of information;
• making necessary changes to the practices relating to management, maintenance
and destruction of records;
• enhancing training provision for officials on the right to information;
• seeking an annual report from the public authority on compliance with this Act;
• requiring the public authority to compensate for any loss or other detriment
suffered by the applicant;
• imposing penalties under this Act;
• Rejecting the application.
➢ The Commission submits an annual report to the State Government on the
implementation of the provisions of this Act. The State Government places this report
before the State Legislature.
➢ When a public authority does not conform to the provisions of this Act, the
Commission may recommend (to the authority) steps that ought to be taken to
promote such conformity.

6. Which one of the following Constitutional Amendments empowered the State to impose restrictions
on the rights of freedom of speech and expression in the interests of the sovereignty and integrity of
India?
(a) 16th Amendment
(b) 42nd Amendment
(c) 44th Amendment
(d) 91st Amendment
EXPLANATION:
The Sixteenth Amendment Act of 1963 has two provisions as follows:
➢ Empowered the state to impose further restrictions on the rights to freedom of speech and expression,
to assemble peaceably and to form associations in the interests of the sovereignty and integrity of
India.
➢ Included sovereignty and integrity in the forms of oaths or affirmations to be subscribed by
contestants to the legislatures, members of the legislatures, ministers, judges and CAG of India.
So, Option (a) is correct.

ADDITIONAL INFORMATION:
OTHER IMPORTANT AMENDMENTS
42nd It is known as "Mini Constitution'.
Amendment ➢ Added three new words (i.e., socialist, secular and integrity) in the Preamble.
➢ Added Fundamental Duties by the citizens (new Part IV A).
➢ Made the president bound by the advice of the cabinet.
➢ Provided for administrative tribunals and tribunals for other matters (Added Part
XIV A).
➢ Froze the seats in the Lok Sabha and state legislative assemblies on the basis of
1971 census till 2001.

6
➢ Made the constitutional amendments beyond judicial scrutiny.
➢ Curtailed the power of judicial review and writ jurisdiction of the Supreme Court and
high courts.
➢ Added three new Directive Principles, viz., equal justice and free legal aid,
participation of workers in the management of industries and protection of the
environment, forests and wildlife.
➢ Facilitated the proclamation of national emergency in a part of the territory of India.
44th ➢ Restored the original term of the Lok Sabha and the state legislative assemblies (i.e.,
Amendment 5 years)."
➢ Restored the provisions with regard to quorum in the Parliament and state
legislatures.
➢ Gave constitutional protection to publication in newspapers of true reports of the
proceedings of the Parliament and the state legislatures.
➢ Empowered the president to send back the advice of the cabinet for reconsideration.
But, the reconsidered advice is to be binding on the president.
➢ Replaced the term "internal disturbance" with "armed rebellion" in respect of
national emergency.
➢ Deleted the right to property from the list of Fundamental Rights and made it only a
legal right.
➢ Provided that the fundamental rights guaranteed by Articles 20 and 21 cannot be
suspended during a national emergency.
91st To limit the size of the Council of Ministers, to debar defectors from holding public
Amendment offices, and to strengthen the anti-defection law:
➢ The total number of ministers, including the Prime Minister, in the Central Council
of Ministers shall not exceed 15% of the total strength of the Lok Sabha.
➢ A member of either house of Parliament belonging to any political party who is
disqualified on the grounds of defection shall also be disqualified to be appointed as
a minister.
➢ The total number of ministers, including the Chief Minister, in the Council of
Ministers in a state shall not exceed 15% of the total strength of the Legislative
Assembly of that state. But, the number of ministers, including the Chief Minister,
in a state shall not be less than 12.
➢ A member of either House of a state legislature belonging to any political party who
is disqualified on the ground of defection shall also be disqualified to be appointed
as a minister.
➢ A member of either the House of Parliament or the House of a State Legislature
belonging to any political party who is disqualified on the grounds of defection shall
also be disqualified from holding any remunerative political post.
➢ The provision of the Tenth Schedule (anti-defection law) pertaining to an exemption
from disqualification in case of a split by one-third of members of the legislative party
has been deleted. It means that the defectors have no more protection on the grounds
of splits.

7. Consider the following statements with respect to the Pro Tem Speaker?
1. The pro tem speaker is appointed by the President of India.
2. He appoints the new speaker of the Lok Sabha.
3. The office of the Pro-term speaker is permanent.
Which of the above statements are correct?
(a) 1 only
(b) 1 and 3 only
(c) 2 only
(d) 2 and 3 only

7
EXPLANATION:
As provided by the Constitution, the Speaker of the last Lok Sabha vacates his office immediately before
the first meeting of the newly-elected Lok Sabha. Therefore, the President appoints a member of the Lok
Sabha as the Speaker Pro Tem. Usually, the seniormost member is selected for this. The President
himself administers oath to the Speaker Pro Tem.
Pro-tem Speaker is a temporary speaker appointed for a limited time period to conduct proceedings in
Parliament or State legislatures after the General elections. The office of the Pro-term speaker is not
permanent.
So, Statement 1 is correct and Stataement 3 is not correct.
A pro-tem Speaker is ordinarily elected for the first sitting of a new legislative assembly where the
Speaker is yet to be elected.
The main duty of the pro-tem speaker is to administer the oath of office to new members of the house.
He also enables the House to elect a new speaker. Once the new speaker is elected, the office of the pro-
tem speaker ceases to exist. Thus, he will not appoint the new speaker of the Lok Sabha. So, Statement
2 is not correct.

8. With reference to the Comprehensive and Progressive Agreement for Trans-Pacific Partnership
(CPTPP), consider the following statements:
1. It is a free trade agreement between Pacific Rim countries.
2. Recently, India signed the CPTPP Accession Protocol and became the 12th member of the CPTPP.
3. All the CPTPP members are also members of APEC.
Which of the statements given above are correct?
(a) 1 and 2 only
(b) 2 and 3 only
(c) 1 and 3 only
(d) 1, 2 and 3
EXPLANATION:
The Comprehensive and Progressive Agreement for Trans-Pacific Partnership (CPTPP) is a free trade
agreement between Canada and 10 other countries in the Indo-Pacific. CPTPP covers virtually all sectors
and aspects of trade between Canada and member countries to reduce trade barriers and facilitate trade.
Therefore, Comprehensive and Progressive Agreement for Trans-Pacific Partnership is a free trade
agreement between Pacific Rim countries. So, Statement 1 is correct.

On 16 July 2023, the CPTPP member countries and the UK signed the UK's Accession Protocol. This will
officially add the UK as the 12th member of the CPTPP and expand the Agreement beyond the Asia-

8
Pacific region. It makes the United Kingdom the first new member and first European nation to join the
bloc since it was created in 2018.
Thus, India did not sign the Comprehensive and Progressive Agreement for Trans-Pacific Partnership
(CPTPP). So, Statement 2 is not correct.
The Comprehensive and Progressive Agreement for Trans-Pacific Partnership ("CPTPP") is a Free Trade
Agreement ("FTA") between 11 countries: Australia, Brunei Darussalam, Canada, Chile, Japan, Malaysia,
Mexico, New Zealand, Peru, Singapore and Vietnam. All 11 countries are members of the Asia-Pacific
Economic Cooperation (APEC). So, Statement 3 is correct.
ADDITIONAL INFORMATION:
ASIA-PACIFIC ECONOMIC COOPERATION
About ➢ The Asia-Pacific Economic Cooperation (APEC) is a regional economic forum
established in 1989 to leverage the growing interdependence of the Asia-Pacific.
➢ APEC's 21 members aim to create greater prosperity for the people of the region by
promoting balanced, inclusive, sustainable, innovative and secure growth and by
accelerating regional economic integration.
➢ APEC's 21 member economies are Australia; Brunei Darussalam; Canada; Chile;
People's Republic of China; Hong Kong, China; Indonesia; Japan; Republic of Korea;
Malaysia; Mexico; New Zealand; Papua New Guinea; Peru; The Philippines; The
Russian Federation; Singapore; Chinese Taipei; Thailand; United States of America;
and Viet Nam.

9. Consider the following:


1. Chairperson of the National Commission for Minorities
2. Chairperson of the National Commission for Women
3. Chairperson of the National Commission for Backward Classes
4. Commissioner for Linguistic Minorities
5. Chief Commissioner for Persons with Disabilities
6. Chairperson of the National Commission for the Protection of Child Rights
How many of the above are ex-officio members of the National Human Rights Commission?
(a) Only three
(b) Only four
(c) Only five
(d) All six
EXPLANATION:
The National Human Rights Commission is a statutory (and not a constitutional) body. It was established
in 1993 under legislation enacted by the Parliament, namely, the Protection of Human Rights Act 1993.
The commission is a multi-member body consisting of a chairperson and five members who act as full-
time members.
In addition to these full-time members, the commission also has seven ex-officio members.
➢ The chairperson of the National Commission for Minorities. So, Statement 1 is correct.
➢ The chairperson of the National Commission for Women. So, Statement 2 is correct.
➢ The chairperson of the National Commission for Backward Classes. So, Statement 3 is correct.
➢ The chairperson of the National Commission for Scheduled Caste.
➢ The chairperson of the National Commission for Scheduled Tribes.
➢ The Chief Commissioner for Persons with Disabilities. So, Statement 5 is correct.
➢ The chairperson of the National Commission for the Protection of Child Rights. So, Statement 6 is
correct.

9
However, the Commissioner for Linguistic Minorities is not an ex-officio member of the National Human
Rights Commission. So, Statement 4 is not correct.

ADDITIONAL INFORMATION:
NATIONAL HUMAN RIGHTS COMMISSION
About The National Human Rights Commission (NHRC) is the watchdog of human rights in
the country, that is, the rights relating to life, liberty, equality and dignity of the
individual guaranteed by the Constitution or embodied in the international covenants
and enforceable by courts in India.
Composition ➢ The commission is a multi-member body consisting of a chairperson, five
of NHRC members, and some ex-officio members.
➢ The chairperson of the NHRC should be a retired chief justice of India or a judge
of the Supreme Court.
➢ Members should be a serving or retired judge of the Supreme Court, a serving or
retired chief justice of a high court and three persons (out of which at least one
should be a woman) having knowledge or practical experience with respect to
human rights.
➢ The chairperson and members are appointed by the president on the
recommendations of a six-member committee consisting of
• The prime minister as its head,
• The Speaker of the Lok Sabha,
• the Deputy Chairman of the Rajya Sabha,
• Leaders of the Opposition in both the Houses of Parliament
• The Central home minister.
➢ Further, a sitting judge of the Supreme Court or a sitting chief justice of a high
court can be appointed only after consultation with the chief justice of India.
Term of Office ➢ The chairperson and members hold office for a term of three years or until they
attain the age of 70 years, whichever is earlier. They are elligible for re-
appointment.
➢ After their tenure, the chairperson and members are not eligible for further
employment under the Central or a state government.

10. With reference to the Attorney General (AG) of India, consider the following statements:
1. According to the Constitution of India, the Solicitor General and Additional Solicitor General shall
assist the AG in performing his duties.
2. AG has the right to speak and to take part in the proceedings of a joint sitting of the Parliament.
3. During his term in office, the AG is barred from private legal practice.
How many of the statements given above are correct?
(a) Only one
(b) Only two
(c) All three
(d) None

10
EXPLANATION:

In addition to the Attorney General (AG), there are other law officers of the Government of India. They
are the solicitor general of India and additional solicitor general of India. They assist the AG in the
fulfilment of his official responsibilities. It should be noted here that only the office of the AG is created
by the Constitution. In other words, Article 76 does not mention about the solicitor general and additional
solicitor general. Thus, Constitution of India does not mention that the Solicitor General and Additional
Solicitor General shall assist the AG in performing his duties. So, Statement 1 is not correct.

In the performance of his official duties, the Attorney General has the right of audience in all courts in
the territory of India. Further, he has the right to speak and to take part in the proceedings of both the
Houses of Parliament or their joint sitting and any committee of the Parliament of which he may be
named a member, but without a right to vote. He enjoys all the privileges and immunities that are
available to a member of Parliament. So, Statement 2 is correct.

The Attorney General is not a fulltime counsel for the Government. He does not fall in the category of
government servants. Further, he is not debarred from private legal practice subject to the limitations
imposed upon his office by Section 8 of the Law Officer (Conditions of Service) Rules, 1987. So,
Statement 3 is not correct.

ADDITIONAL INFORMATION:

ATTORNEY GENERAL OF INDIA

About ➢ The Constitution (Article 76) has provided for the office of the Attorney General for India.
He is the highest law officer in the country.
➢ The Attorney General (AG) is appointed by the president. He must be a person who is
qualified to be appointed a judge of the Supreme Court.
➢ In other words, he must be a citizen of India and he must have been a judge of some
high court for five years or an advocate of some high court for ten years or an eminent
jurist, in the opinion of the president.
➢ The term of office of the AG is not fixed by the Constitution. Further, the Constitution
does not contain the procedure and grounds for his removal.
➢ He holds office during the pleasure of the president. This means that he may be removed
by the president at any time.
➢ He may also quit his office by submitting his resignation to the president.
Conventionally, he resigns when the government (council of ministers) resigns or is
replaced, as he is appointed on its advice.
➢ The remuneration of the AG is not fixed by the Constitution. He receives such
remuneration as the president may determine

11
11. Consider the following statements:
An amendment to the Constitution of India can be initiated by the:
1. Lok Sabha
2. Rajya Sabha
3. State Legislature
4. President
Which of the above statements is/are correct?
(a) Only 1
(b) 1, 2 and 3
(c) 2 and 4
(d) 1 and 2
EXPLANATION:

Article 368 in Part XX of the Indian Constitution, an amendment of the Constitution can be initiated only
by the introduction of a bill for the purpose in either House of Parliament (Lok Sabha or Rajya sabha)and
not in the state legislatures. The procedure for the amendment of the Constitution, as laid down in Article
368, is as follows:
➢ The bill can be introduced either by a minister or by a private member and does not require prior
permission of the president.
➢ The bill must be passed in each House by a special majority, that is, a majority of the total membership
of the House and a majority of two-thirds of the members of the House present and voting.
➢ Each House must pass the bill separately. In case of a disagreement between the two Houses, there is
no provision for holding a joint sitting of the two Houses for the purpose of deliberation and passage
of the bill.
➢ If the bill seeks to amend the federal provisions of the Constitution, it must also be ratified by the
legislatures of half of the states by a simple majority, that is, a majority of the members of the House
present and voting.
➢ After duly passed by both Houses of Parliament and ratified by the state legislatures, where necessary,
the bill is presented to the president for assent.
➢ The president must give his assent to the bill. He can neither withhold his assent to the bill nor return
the bill for reconsideration by the Parliament.
➢ After the president’s assent, the bill becomes an Act (i.e., a constitutional amendment act), and the
Constitution stands amended in accordance with the terms of the Act. So, option (d) is correct.

12. Which of the following statements is correct with respect to the Participatory Guarantee System of
India (PGS-India) ?
(a) It is a quality assurance initiative for the certification of organic produce.
(b) It is an intiative for the manufacturing of high-efficiency solar PV modules.
(c) It is to ensure financial inclusion and provide banking services.
(d) None of the above.

12
EXPLANATION:
Under PGS-India programme, Government is implementing Participatory Guarantee System of India as
a quality assurance initiative for certification of organic produce that is locally relevant, emphasizing the
participation of stakeholders, including producers / farmers and consumers and operates outside the
frame of third-party certification. In the operation of PGS-India, stakeholders (including farmers/
producers) are involved in decision making and essential decisions about the operation of the PGS-India
certification itself by assessing, inspecting and verifying the production practices of each other and
collectively declare produce as organic.
So, Option (a) is correct.
The Production Linked Incentive (PLI) Scheme for high efficiency solar PV modules, aims to provide
impetus to the domestic industry to scale up manufacturing in the coming years so that majority of this
demand can be met domestically itself and India does not have to depend on imports to fulfill its
demands. This will also help build India’s capacity and technological capability to become part of the
global supply chain for high efficiency solar PV modules and cater to the international market as well.
So, Option (b) is not correct.
Pradhan Mantri Jan Dhan Yojana (PMJDY) is the biggest financial inclusion initiatives in the world by
the Ministry of Finance. Pradhan Mantri Jan Dhan Yojana (PMJDY) is National Mission for Financial
Inclusion to ensure access to financial services, namely, Banking/ Savings & Deposit Accounts,
Remittance, Credit, Insurance, Pension in an affordable manner.
It continuously endeavours to provide financial inclusiveness and support to the marginalised and
economically backward sections through its financial inclusion led interventions. Financial Inclusion (FI)
promotes equitable and inclusive growth as well as delivery of financial services at an affordable cost to
vulnerable groups such as low-income groups and weaker sections that lack access to basic banking
services.
So, Option (c) is not correct.

ADDITIONAL INFORMATION:
THE PARAMPARAGAT KRISHI VIKAS YOJANA (PKVY)
About ➢ The Paramparagat Krishi Vikas Yojana (PKVY), launched in 2015, is an extended
component of Soil Health Management (SHM) under the Centrally Sponsored Scheme
(CSS), National Mission on Sustainable Agriculture (NMSA).
➢ PKVY aims at supporting and promoting organic farming, in turn resulting in
improvement of soil health.
➢ The scheme promotes Participatory Guarantee System (PGS) For India (PGS- India)
form of organic certification that is built on mutual trust, locally relevant and
mandates the involvement of producers and consumers in the process of certification.
➢ PGS – India operates outside the framework of “Third Party Certification”.
➢ Funding pattern under the scheme is in the ratio of 60:40 by the Central and State
Governments respectively. In case of North Eastern and Himalayan States, Central
Assistance is provided in the ratio of 90:10 (Centre: State) and for Union Territories,
the assistance is 100%.
Objective The objective is to produce agricultural products free from chemicals and pesticides
residues by adopting eco- friendly, low- cost technologies. Key Thrust areas of PKVY in
promoting organic farming include the following:

13
➢ Promote organic farming among rural youth/ farmers/ consumers/ traders
➢ Disseminate latest technologies in organic farming
➢ Utilize the services of experts from public agricultural research system in India
➢ Organize a minimum of one cluster demonstration in a village

13. Consider the following statements:


Statement-I:
After the Government of India Act of 1935 came into effect, Provincial Legislatures in India
possessed the features of a sovereign Legislature.
Statement-II:
As per the Government of India Act of 1935, Provinces were no longer delegates of the Central
Government but were autonomous units of administration.
Which one of the following is correct in respect of the above statements?
(a) Both Statement-I and Statement-II are correct and Statement-II is the correct explanation for
Statement-I
(b) Both Statement-I and Statement-II are correct and Statement-II is not the correct explanation for
Statement-I
(c) Statement-I is correct but Statement-II is incorrect
(d) Statement-I is incorrect but Statement-II is correct
EXPLANATION:
The Government of India Act of 1935 abolished dyarchy in the provinces and introduced ‘provincial
autonomy’ in its place. The executive authority of the Centre was vested in the Governor-General. The
legislative powers of both the Central and Provincial Legislatures were subject to various limitations and
neither could be said to have possessed the features of a sovereign legislature.
So, Statement-I is incorrect
The Government of India Act of 1935 introduced provincial autonomy were it allowed to act as
autonomous units of administration in their defined spheres. The Act divided legislative powers between
the Provincial and Central Legislatures, and within its defined sphere, the Provinces were no longer
delegates of the Central Government, but were autonomous units of administration.
The Governor was required to act with the advice of ministers responsible to the provincial legislature.
The executive authority of a Province was also exercised by the governor on behalf of the crown and not
as a sub-ordinate of the Governor-General. So, Statement-II is correct.
ADDITIONAL INFORMATION:
GOVERNMENT OF INDIA ACT OF 1935
Features ➢ It provided for the establishment of an All-India Federation consisting of provinces and
princely states as units. The Act divided the powers between the Centre and units in
terms of three lists such as the Federal List (for the Centre, with 59 items), the
Provincial List (for provinces, with 54 items), and the Concurrent List (for both, with 36
items). Residuary powers were given to the Viceroy. However, the federation never came
into being as the princely states did not join it.
➢ It abolished dyarchy in the provinces and introduced ‘provincial autonomy’ in its place.
The provinces were allowed to act as autonomous units of administration in their
defined spheres. Moreover, the Act introduced responsible governments in provinces;
that is, the governor was required to act with the advice of ministers responsible to the
provincial legislature. This came into effect in 1937 and was discontinued in 1939.

14
➢ It provided for the adoption of dyarchy at the Centre. Consequently, the federal subjects
were divided into reserved subjects and transferred subjects. However, this provision
of the Act did not come into operation at all.
➢ It introduced bicameralism in six out of eleven provinces. Thus, the legislatures of
Bengal, Bombay, Madras, Bihar, Assam, and the United Provinces were made
bicameral, consisting of a legislative council (upper house) and a legislative assembly
(lower house). However, many restrictions were placed on them.
➢ It further extended the principle of communal representation by providing separate
electorates for depressed classes (scheduled castes), women and labour (workers).
➢ It abolished the Council of India, established by the Government of India Act of 1858.
The secretary of state for India was provided with a team of advisors.
➢ It extended the franchise. About 10 per cent of the total population got the voting right.
➢ It provided for the establishment of a Reserve Bank of India to control the currency and
credit of the country.
➢ It provided for the establishment of not only a Federal Public Service Commission but
also a Provincial Public Service Commission and Joint Public Service Commission for
two or more provinces.
➢ It provided for the establishment of a Federal Court, which was set up in 1937.

14. With regard to the head of the Indian Parliamentary system being an elected President instead of a
hereditary monarch, the makers of the Indian Constitution followed the precedent of which of the
following?
(a) Constitution of the United States of America
(b) Constitution of Canada
(c) Constitution of Ireland
(d) Constitution of South Africa
EXPLANATION:
The makers of the Constitution rejected the Presidential system of government, as it obtains in America,
on the ground that under that system the Executive and the Legislatures are separate from and
independent of each other.
But though the British model of a Parliamentary or Cabinet form of government was adopted, a hereditary
monarch or ruler at the head could not be installed because India had declared a "Republic". Instead of
a monarch, an elected President was to be at the head of the Parliamentary system. In introducing this
system, The makers of the Indian Constitution followed the Irish Precedent.
Further, the Indian Constitution superimposes an elected President upon the Parliamentary system of
responsible government, which is like the Constitution of Ireland.
Also, the makers of the Indian Constitution borrowed some provisions from the Constitution of Ireland:
➢ Directive Principles of State Policy
➢ Method of Election of the President
➢ Members' nomination to the Rajya Sabha by the President.
So, Option (c) is correct.
ADDITIONAL INFORMATION:
INDIAN CONSTITUTION BORROWED FROM OTHER COUNTRIES
About In a parliamentary system, the prime minister is the head of government. Most
Parliamentary parliamentary systems have a president or a monarch who is the nominal Head of
system

15
state. In such a system, the role of the president or monarch is primarily ceremonial,
and the prime minister, along with the cabinet, wields effective power.
It is either a president (elected either popularly or by the parliament) or a hereditary
monarch (often in a constitutional monarchy).
India's parliamentary system is bicameral, consisting of the President and two houses:
the Rajya Sabha (Council of States) and the Lok Sabha (House of the People).

16
15. Consider the following provisions under the Directive Principles of State Policy as enshrined in the
Indian Constitution:
1. Prohibition of the slaughter of cows
2. Prevention of concentration of wealth
3. Opportunities for the healthy development of children
4. Promotion of equal justice
How many of the above are the Gandhian principles that are reflected in the DPSP?
(a) Only one
(b) Only two
(c) Only three
(d) All four
EXPLANATION:
The Constitution does not contain any classification of Directive Principles of State policy. However, on
the basis of their content and direction, they can be classified into three broad categories, viz, socialistic,
Gandhian and liberal-intellectual.
In order to fulfil the dreams of Gandhi, some of his ideas were included as Directive Principles. They
require the State:
➢ To organize village panchayats and endow them with the necessary powers and authority to enable
them to function as units of self-government (Article 40).
➢ To promote cottage industries on an individual or cooperation basis in rural areas (Article 43).
➢ To promote voluntary formation, autonomous functioning, democratic control and professional
management of cooperative societies (Article 43B).
➢ To promote the educational and economic interests of SCs, STs, and other weaker sections of society
and to protect them from social injustice and exploitation (Article 46).
➢ To prohibit the consumption of intoxicating drinks and drugs which are injurious to health (Article
47).
➢ To prohibit the slaughter of cows, calves and other milch and draught cattle and to improve their
breeds (Article 48). So, Statement 1 is correct.
These principles are based on Gandhian ideology. They represent the programme of reconstruction
enunciated by Gandhi during the national movement.
Some Directive principles of state policy reflect the ideology of socialism. They lay down the framework
of a democratic socialist state, aim at providing social and economic justice, and set the path towards
the welfare state. They direct the state:
➢ To promote the welfare of the people by securing a social order permeated by justice–social,
economic, and political and to minimize inequalities in income, status, facilities, and opportunities
(Article 38).
➢ To secure (a) the right to adequate means of livelihood for all citizens; (b) the equitable distribution
of material resources of the community for the common good; (c) prevention of concentration of
wealth and means of production; (d) equal pay for equal work for men and women; (e) preservation
of the health and strength of workers and children against forcible abuse; and (f) opportunities for
healthy development of children (Article 39).
➢ To promote equal justice and to provide free legal aid to the poor (Article 39 A).
➢ To secure the right to work, education and public assistance in cases of unemployment, old age,
sickness and disablement (Article 41).
➢ To make provision for just and humane conditions of work and maternity relief (Article 42).
➢ To secure a living wage, a decent standard of life and social and cultural opportunities for all workers
(Article 43).
17
➢ To take steps to secure the participation of workers in the management of industries (Article 43 A).
➢ To raise the level of nutrition and the standard of living of people and to improve public health
(Article 47).
So, Statements 2, 3 and 4 are not correct.

16. Which of the following present-day state witnessed a Kharsawan massacre in 1948 that is similar to
the Jallianwala Bagh massacre in 1919 ?
(a) Orissa
(b) Jharkhand
(c) Bihar
(d) Chhattisgarh
EXPLANATION:
Every year, Adivasis from Jharkhand’s Kolhan region observe January 1 as a day of mourning and
remembrance of Kharsawan massacre happened in 1948.
Kharsawan was a small princely state, less than 400 sq km in area, lying to the west of Jamshedpur. At
the time of Independence, Kharsawan, alongside 24 other princely states in eastern India, decided to
accede to the Union of India and join the state of Orissa. After all, Kharsawan and neighbouring Saraikela
(which today form the Seraikela Kharsawan district in Jharkhand), had more Odia speakers than Hindi
speakers.
But most Adivasis did not support this merger. They also did not want to join Bihar, the alternative
option. They wanted a separate Adivasi state. In protest, a huge meeting was called at Kharsawan on
January 1, 1948, the day when the merger was to take place. This was also the day of the weekly haat
in the town. Jaipal Munda himself was supposed to be present and address the crowd.
On January 1, 1948, the town of Kharsawan witnessed a massacre reminiscent of what happened
in Jallianwala Bagh in 1919. Police opened fire at a crowd gathered for a protest and the
weekly haat (market), killing hundreds, or by some accounts, thousands of Adivasis.
So, Option (b) is correct.

ADDITIONAL INFORMATION:
KHARSAWAN MASSACRE
Struggle for ➢ In 1912, the Bengal Presidency was partitioned to create the Bihar and Orissa
an Adivasi Province. However, within this new province, there existed a large Adivasi population
state with its distinct culture and many grievances with both the British and non-tribal
populations.
➢ Thus, in 1912 itself, the demand for a separate tribal state was articulated for the
first time at St Columba’s College, Hazaribagh.
➢ Orissa was carved out in 1936, Adivasis’ demands remained unheard. In 1938 the
Adivasi Mahasabha was formed to continue the struggle, with Jaipal Singh Munda
(1903-70), former captain of the Indian hockey team, emerging as its tallest leader.

17. Consider the following statements about family courts in India:


1. The state governments establish the family court after consulting with the respective High Courts.
2. A person who served as an advocate of a High Court for seven years is qualified to be appointed
as a judge of the family court.
3. An order passed by the family court shall have the same force and effect of a civil court.

18
Which of the statements given above are correct?
(a) 1 and 2 only
(b) 2 and 3 only
(c) 1 and 3 only
(d) 1, 2 and 3
EXPLANATION:
Setting up of Family Courts and its functioning lies within the domain of State Government in
consultation with their respective High Courts. The Family Courts Act, 1984 provides for the
establishment of Family Courts by the State Governments in consultation with the High Courts to
promote conciliation and secure speedy settlement of disputes relating to marriage and family affairs. As
per the Act, it is mandatory for the State Government to set up a Family Court for every city or a town
whose population exceeds one million. In other areas of the States, the Family Courts may be set up if
the State Governments deems it necessary. So, Statement 1 is correct.
A person shall not be qualified for appointment as a Judge of a family court unless,
➢ He has for at least seven years held a judicial office in India or the office of a Member of a Tribunal
or any post under the Union or a State requiring special knowledge of law; or
➢ He has for at least seven years been an advocate of a High Court or of two or more such Courts in
succession; or
➢ He possesses such other qualifications as the Central Government may, with the concurrence of the
Chief Justice of India as prescribed. So, Statement 2 is correct.
A decree or an order other than an order under Chapter IX of the Code of Criminal Procedure, 1973
passed by a Family Court shall have the same force and effect as a decree or order of a civil court and
shall be executed in the same manner as is prescribed by the Code of Civil Procedure, 1908 for the
execution of decrees and orders. So, Statement 3 is correct.
ADDITIONAL INFORMATION:
FAMILY COURTS
About The salient features of the Family Courts Act, 1984 are as follows:
➢ It provides for the establishment of Family Courts by the State Governments in
consultation with the High Courts.
➢ It makes it obligatory on the State Governments to set up a Family Court in every
city or town with a population exceeding one million.
➢ It enables the State Governments to set up Family Courts in other areas also, if they
deem it necessary.
➢ It exclusively provides within the jurisdiction of the Family Courts the matters
relating to:
• matrimonial relief, including nullity of marriage, judicial separate divorce,
restitution of conjugal rights, or declaration as to validity of marriage or as to
the matrimonial status of any person
• the property of the spouses or of either of them;
• declaration as to the legitimacy of any person;
• guardianship of a person or the custody of any minor; and
• Maintenance of wife, children and parents.
➢ It makes it obligatory on the part of the Family Court to endeavour, in the first
instance to effect reconciliation or a settlement between the parties to a family
dispute. During this stage, the proceedings will be informal and rigid rules of
procedure shall not apply.

19
➢ It provides for the association of social welfare agencies, counsellors, etc., during
conciliation stage and also to secure the service of medical and welfare experts.
➢ It provides that the parties to a dispute before a Family Court shall not be entitled,
as of right, to be represented by legal practitioner. However, the Court may, in the
interest of justice, seek assistance of a legal expert as amicus curiae.
➢ It simplifies the rules of evidence and procedure so as to enable a Family Court to
deal effectively with a dispute.
➢ It provides for only one right of appeal which shall lie to the High Court

18. Consider the following statements with respect to the Indian Citizenship:
1. A person having adequate knowledge of a language specified in the eighth schedule shall obtain
citizenship through naturalization.
2. Minor Children of persons who are citizens of India shall avail citizenship through registration.
Which of the statements given above is/are correct?
(a) 1 only
(b) 2 only
(c) Both 1 and 2
(d) Neither 1 nor 2
EXPLANATION:
The Citizenship Act of 1955 prescribes five ways of acquiring citizenship, viz, birth, descent, registration,
naturalization and incorporation of territory.
The Central Government may, on an application, grant a certificate of naturalization to any person (not
being an illegal migrant) if he possesses adequate knowledge of a language specified in the Eighth
Schedule to the Constitution. So, Statement 1 is correct.
Under the Registration way of acquiring citizenship,
The Central Government may, on an application, register as a citizen of India any person (not being an
illegal migrant) if he belongs to any of the following categories, namely:
➢ A person of Indian origin who is ordinarily resident in India for seven years before making an
application for registration;
➢ A person of Indian origin who is ordinarily resident in any country or place outside undivided India;
➢ A person who is married to a citizen of India and is ordinarily resident in India for seven years before
making an application for registration;
➢ Minor children of persons who are citizens of India, So Statement 2 is correct.
➢ A person of full age and capacity whose parents are registered as citizens of India;
➢ A person of full age and capacity who, or either of his parents, was earlier a citizen of independent
India and is ordinarily resident in India for twelve months immediately before making an application
for registration;
➢ A person of full age and capacity who has been registered as an overseas citizen of India cardholder
for five years, and who is ordinarily resident in India for twelve months before making an application
for registration.
ADDITIONAL INFORMATION:
CITIZENSHIP
About ➢ The Constitution deals with the citizenship from Articles 5 to 11 under Part II.
However, it contains neither any permanent nor any elaborate provisions in this
regard. It only identifies the persons who became citizens of India at its
commencement (i.e., on January 26, 1950).

20
➢ It does not deal with the problem of acquisition or loss of citizenship subsequent to
its commencement.
➢ It empowers the Parliament to enact a law to provide for such matters and any
other matter relating to citizenship.
➢ Accordingly, the Parliament has enacted the Citizenship Act (1955), which has been
amended from time to time.
Citizenship The Central Government may, on an application, grant a certificate of naturalisation
through to any person (not being an illegal migrant) if he possesses the following qualifications:
Naturalization ➢ That he is not a subject or citizen of any country where citizens of India are
prevented from becoming subjects or citizens of that country by naturalisation;
➢ That, if he is a citizen of any country, he undertakes to renounce the citizenship of
that country in the event of his application for Indian citizenship being accepted;
➢ That he has either resided in India or been in the service of a Government in India
or partly the one and partly the other throughout twelve months immediately
preceding the date of the application;
➢ That during the fourteen years immediately preceding the said period of twelve
months, he has either resided in India or been in the service of a Government in
India, or partly the one and partly the other, for periods amounting in the aggregate
to not less than eleven years;
➢ That he is of good character;
➢ That in the event of a certificate of naturalization being granted to him, he intends
to reside in India or to enter into or continue in service under a Government in
India or under an international organization of which India is a member or under
a society, company or body of persons established in India.
➢ However, the Government of India may waive all or any of the above conditions for
naturalization in the case of a person who has rendered distinguished service to
the science, philosophy, art, literature, world peace or human progress. Every
naturalized citizen must take an oath of allegiance to the Constitution of India.

19. Which one of the following has recently prepared the Model Prisons Act of 2023, which has to be
adopted by States and union territories in their respective jurisdictions?
(a) Ministry of Law and Justice
(b) Ministry of Home Affairs
(c) Law Commission of India
(d) National Legal Services Authority
EXPLANATION:
The Ministry of Home Affairs (MHA) has prepared the ‘Model Prisons Act 2023’ which has to be adopted
by States and union territories in their respective jurisdictions. The act will replace a British-era law to
overhaul the prison administration that will focus on the reformation and rehabilitation of inmates.
Some of the focus areas of the Model Act provide security assessment and segregation of prisoners,
individual sentence planning, grievance redressal, prison development board, attitudinal change towards
prisoners and provision of separate accommodation for women prisoners, transgender, etc.
The Model Act also talks about the provision for the use of technology in prison administration with a
view to bringing transparency in prison administration, provision for video-conferencing with courts,
scientific and technological interventions in prisons, etc. So, Option (b) is correct.

21
ADDITIONAL INFORMATION:
MODEL PRISONS ACT OF 2023
Recently in Home Ministry prepares Model Prisons Act 2023 to replace British-era law. There is no
News provision for reform and rehabilitation of prisoners in the existing Act, it says.
About ➢ Along with the Prisons Act 1894, the Prisoners Act 1900 and the Transfer of
Prisoners Act 1950 have also been reviewed by the Ministry of Home Affairs. The
relevant provisions of these Acts have been “assimilated” in the ‘Model Prisons Act,
2023.
➢ State governments and union territory administrations can benefit from the Model
Prisons Act of 2023 by adopting it in their jurisdictions, with such modifications
which they may consider necessary, and repeal the existing three Acts in their
jurisdictions.
➢ Salient features of the Prison Act,
• Provisions of punishment for prisoners and jail staff for use of prohibited items
such as mobile phones in jails,
• Establishment and management of high-security jails, open jails (open and
semi-open), and
• Provisions for protecting society from the criminal activities of hardened
criminals and habitual offenders.
• It also contains provisions for providing legal aid to prisoners, parole, furlough
and premature release to incentivize good conduct.

20. Consider the following statements with respect to the National Transit Pass System (NTPS):
5. The NTPS is to facilitate the seamless transit of timber, bamboo, and other forest produce across
the country.
6. It is envisioned as a "One Nation-One Pass" regime launched by the Ministry of Road Transport
and Highways.
Which of the above statements are correct?
(a) 1 only
(b) 2 only
(c) Both 1 and 2
(d) Neither 1 nor 2
EXPLANATION:
National Transit Pass System (NTPS) to facilitate the seamless transit of timber, bamboo, and other forest
produce across the country. Currently, transit permits are issued for the transport of timber and forest
produce based on state-specific transit rules. So, Statement 1 is correct.
The NTPS launched by the Ministry of Environment, Forest and Climate Change is envisioned as a "One
Nation-One Pass" regime, which will enable seamless transit across the country. This initiative will
streamline the issuance of timber transit permits by providing a unified, online mode for tree growers
and farmers involved in agroforestry across the country, contributing to the ease of doing business. So,
Statement 2 is not correct.

ADDITIONAL INFORMATION:
NATIONAL TRANSIT PASS SYSTEM (NTPS)
About ➢ NTPS offers seamless transit permits, managing records for both inter-state and
intra-state transportation of timber, bamboo and other forest produce obtained
from various sources like private lands, government-owned forests and private
depots.
➢ NTPS is designed for user convenience, featuring desktop and mobile
applications for easy registration and permit applications. Transit permits will
be issued for tree species that are regulated, while the users can self-generate
No Objection Certificates for exempted species.
➢ The QR-coded transit permits generated under NTPS will allow check gates
across various states to verify the validity of the permits and allow seamless
transit.

22
➢ Presently, 25 States and Union Territories have embraced the unified permit
system, streamlining interstate business operations for producers, farmers, and
transporters. This move is expected to provide a significant impetus to the
agroforestry sector.
➢ Prior to the introduction of NTPS, obtaining transit permits from different states
along the route was a time-consuming process, causing hurdles in transporting
timber and forest products across the states.

21. Which of the following are regarded as the main features of the ‘Proportional Representation’?
1. Voters vote for the party
2. Voters vote for a candidate
3. The candidate who wins the election gets the majority of votes
Select the correct answer using the code given below:
(a) 1 only
(b) 1 and 3 only
(c) 2 only
(d) 2 and 3 only
EXPLANATION:
The system of proportional representation aims at removing the defects of territorial representation.
Under this system, all sections of the people get representation in proportion to their number. Even the
smallest section of the population gets its due share of representation in the legislature. Every party gets
seats in the legislature in proportion to the percentage of votes that it gets.
In the Proportional Representation (PR) system, Voters vote for the party, while in the First Past the Post
(FPTP) system, Voters vote for a candidate. So, Statement 1 is correct, and Statement 2 is not correct.
In the Proportional Representation system, the Candidate who wins the elections gets the majority of
votes. It is one of the main features of Proportional Representation. While in the First Past the Post
System Candidate who wins the election may not get majority (50+1) votes. So, Statement 3 is correct.

ADDITIONAL INFORMATION:
FIRST PAST THE POST SYSTEM VS PROPORTIONAL REPRESENTATION

23
22. With reference to the Government of National Capital Territory of Delhi (Amendment) Act of 2023,
consider the following statements:
1. It established the National Capital Civil Services Authority consisting of the Chief Minister, the
Chief Secretary, and the Principle Home Secretary of Delhi.
2. The Authority will make recommendations to the Lieutenant Governor (LG) of Delhi regarding
transfers and postings of officials and disciplinary matters.
3. The Act empowers the LG to exercise his sole discretion on summoning, proroguing and dissolving
the Delhi Legislative Assembly.
How many of the statements given above are correct?
(a) Only one
(b) Only two
(c) All three
(d) None
EXPLANATION:
The Government of National Capital Territory of Delhi (Amendment) Act of 2023 establishes the National
Capital Civil Services Authority to make recommendations to the Lieutenant Governor of Delhi (LG) on
certain matters related to services. These include,
➢ transfers and postings,
➢ matters related to vigilance,
➢ disciplinary proceedings, and
➢ prosecution sanctions of Group A of All India Services (except Indian Police Service), and
➢ DANICS.
The Authority will consist of the Chief Minister of Delhi as Chairperson, the Principal Home Secretary of
the Delhi government as Member Secretary, and the Chief Secretary of the Delhi government as member.
The central government will appoint both the Principal Home Secretary and Chief Secretary. All decisions
of the Authority will be based on a majority vote of the members present and voting. The quorum for a
meeting is two people.
The Authority will make recommendations to the Lieutenant Governor (LG) regarding transfers and
postings of officials and disciplinary matters. So, Statements 1 and 2 are correct.
Under the Act, matters where the LG may act at his discretion are:
➢ Matters outside the legislative competence (summoning, prorogation and dissolution) of the Delhi
Legislative Assembly but which have been delegated to the LG or
➢ Matters where he is required by law to act in his discretion or exercise any judicial or quasi-judicial
functions.
The Bill specifies that in these matters, the LG will act in his sole discretion. It expands the discretionary
role of the LG by giving him powers to approve the recommendations of the Authority or return them for
reconsideration. In the case of a difference of opinion between the LG and the Authority, the former’s
decision will be final. So, Statement 3 is correct.

ADDITIONAL INFORMATION:
GOVERNMENT OF NATIONAL CAPITAL TERRITORY OF DELHI (AMENDMENT) ACT OF 2023
About other Disposal of matters by Ministers:
provisions ➢ A Minister of the Delhi government may issue standing orders for the disposal of
matters brought to his attention. The order should be issued in consultation with
the concerned Department Secretary. Certain matters must be submitted to the LG,

24
through the Chief Minister and the Chief Secretary, for his opinion prior to the issue
of any order.
➢ These include proposals affecting:
• the peace and tranquillity of Delhi,
• relations between the Delhi government and the central government, Supreme
Court, or other state governments,
• summoning, prorogation, and dissolution of the Legislative Assembly, and
• Matters on which LG is to give an order in his sole discretion.
Duties of Secretaries: Additionally, the concerned Department Secretary must bring
certain matters to the notice of the LG, the Chief Minister, and the Chief Secretary.
These include matters which may bring the Delhi Government into controversy with the
central or any state government, the Supreme Court, or the High Court of Delhi.

23. Which among the following provisions with respect to the Prime Minister was added to the
Constitution of India through the 91st Constitutional Amendment Act of 2003?
(a) The advice tendered by the Council of Ministers headed by the Prime Minister to the President
shall not be inquired into any court.
(b) When no party has a clear majority in the Lok Sabha, then the President may exercise her
discretion in the selection and appointment of the Prime Minister.
(c) The total number of Ministers, including the Prime Minister, shall not exceed fifteen
percent of the total number of members of the House of the People.
(d) As far as the Prime Minister enjoys the majority support in the Lok Sabha, he cannot be removed
by the President.
EXPLANATION:
Article 74 of the Indian Constitution reads that there shall be a Council of Ministers with the Prime
Minister at the head to aid and advise the President in the exercise of his functions. The question of
whether any, and if so what, advice was tendered by Ministers to the President shall not be inquired into
in any court. This provision was not added by the 91st Constitutional Amendment Act of 2003. So,
Option (a) is not correct.
The constitution does not contain any specific procedure for the selection and appointment of the Prime
Minister. Article 75 says only that the PM shall be appointed by the president. However, this does not
imply that the president is free to appoint anyone as the Prime Minister.
In accordance with the conventions of the Parliamentary system of government, the President has to
appoint the leader of the majority party in the Lok Sabha as the Prime Minister. But, when no party has
a clear majority in the Lok Sabha, then the President may exercise his personal discretion in the selection
and appointment of a PM.
In such a situation, the president usually appoints the leader of the largest party or coalition in the Lok
Sabha as the PM and asks him to seek a vote of confidence in the House within a month. This discretion
was exercised by the President for the first time in 1979, when Neelam Sanjiva Reddy (the then President)
appointed Charan Singh (the coalition leader) as the PM after the fall of the Janata Party government
headed by Morarji Desai. This provision was not added by the 91st Constitutional Amendment Act of
2003. So, Option (b) is not correct.
Article 72 of the Constitution prescribes that the total number of Ministers, including the Prime Minister,
in the Council of Ministers shall not exceed 15 percent of the number of members of the House of the
People. Prior to January 1, 2004 (the effective date of the 91st Amendment of the Constitution), the Prime
Minister had the discretion to appoint any number in his council of ministers. But the Constitution
(Ninety-first Amendment) Act in 2003 made a drastic change in curbing such power of the Prime Minister.

25
Thus, this provision was added by the 91st Constitutional Amendment Act of 2003. So, Option (c) is
correct.
The Prime Minister holds office at the pleasure of the President. However, this does not mean that the
President can dismiss the Prime Minister at any time. So long as the Prime Minister enjoys the majority
support in the Lok Sabha, he cannot be dismissed by the President. This provision was not added by the
91st Constitutional Amendment Act of 2003. So, Option (d) is not correct.
ADDITIONAL INFORMATION:
91ST AMENDMENT ACT OF 2003
Provisions ➢ The total number of ministers, including the Prime Minister, in the Central
Council of Ministers shall not exceed 15 per cent of the total strength of the Lok
Sabha.
➢ A member of either House of Parliament belonging to any political party who is
disqualified on the ground of defection shall also be disqualified to be appointed
as a minister.
➢ The total number of ministers, including the Chief Minister, in the Council of
Ministers in a state shall not exceed 15 per cent of the total strength of the
Legislative Assembly of that state. But, the number of ministers, including the
Chief Minister, in a state shall not be less than 12.
➢ A member of either House of a state legislature belonging to any political party
who is disqualified on the ground of defection shall also be disqualified to be
appointed as a minister.
➢ A member of either House of Parliament or either House of a State Legislature
belonging to any political party who is disqualified on the ground of defection
shall also be disqualified to hold any remunerative political post. The expression
“remunerative political post” means (i) any office under the Central Government
or a state government where the salary or remuneration for such office is paid
out of the public revenue of the concerned government; or (ii) any office under a
body, whether incorporated or not, which is wholly or partially owned by the
Central Government or a state government and the salary or remuneration for
such office is paid by such body, except where such salary or remuneration paid
is compensatory in nature.
➢ The provision of the Tenth Schedule (anti-defection law) pertaining to exemption
from disqualification in case of split by one-third members of legislature party
has been deleted. It means that the defectors have no more protection on
grounds of splits.

24. With reference to the Mulya Pravah 2.0, consider the following statements:
1. It is a guideline to promote ethical practices and human values in higher education institutions.
2. The Ministry of Skill Development and Entrepreneurship issued the Mulya Pravah 2.0 guidelines.
3. The primary focus is to build value-based institutions by developing respect for fundamental
duties, constitutional values, and national bonding.
How many of the statements given above are correct?
(a) Only one
(b) Only two
(c) All three
(d) None

26
EXPLANATION:
Mulya Pravah 2.0 guideline seeks to inculcate human values and professional ethics in Higher Education
Institutions (HEIs). The first outcome of this endeavour is to create institutions with the highest level of
values and ethics. The entire infrastructure like, physical, psychological, knowledge-based, and
financial, needs to be built entirely on high values and ethical practices.
So, Statement 1 is correct.
The University Grants Commission (UGC) under the Ministry of Education developed guidelines on
“Mulya Pravah — Inculcation of Human Values and Professionals Ethics in Higher Educational
Institutions.” These guidelines have now been revised to include the key recommendations of the
National Education Policy, 2020 (NEP-2020), announced on July 29, 2020. The Ministry of Skill
development and Entrepreneurship does not issue these guidelines. So, Statement 2 is not correct.
The primary focus of Mulya Pravah 2.0 is to propose the curriculum and pedagogy of HEIs to develop
deep respect towards Fundamental Duties and Constitutional Values among the students, bonding with
one’s country, and conscious awareness of the roles and responsibilities in a changing world, including
universal human values of truth (Satya), righteous conduct (dharma), peace (shanti), love (prema), non-
violence (ahimsa), scientific temper, citizenship values, and also life-skills. So, Statement 3 is correct.

ADDITIONAL INFORMATION:
MULYA PRAVAH 2.0
Recently in An ambitious push for values ethics in higher education. Unless there are sincere efforts
News to ensure that the provisions of Mulya Pravah 2.0 take effect, the UGC’s move may be a
mere formality.
About ➢ Mulya Pravah 2.0 underscores the need for utmost transparency in administration
and highlights that decision-making in higher education institutions must be solely
guided by institutional and public interest and not be vitiated by biases.
➢ It seeks to abolish the discriminatory privileges of officials and urges the
administration to punish the corrupt.
➢ It lays stress on the criticality of ‘encouraging persons at all levels to think and give
their advice freely’.
➢ The guideline expects higher education institutions to ‘ensure integrity, trusteeship,
harmony, accountability, inclusiveness, commitment, respectfulness,
belongingness, sustainability, constitutional values and global citizenship’.
➢ It further asserts that officers and staff must ‘refrain from misappropriating financial
and other resources, and refuse to accept gift, favour, service, or other items from
any person, group, private business, or public agency which may affect the impartial
performance of duties’.

25. The power to remit a life imprisonment sentence by the State government is provided by the:
(a) Constitution of India
(b) Code of Criminal Procedure
(c) Indian Penal Code
(d) Model Prisons Act of 2023
EXPLANATION:
Section 432 of the Code of Criminal Procedure (CrPC), 1973 provides when any person has been
sentenced to punishment for an offense, the appropriate State Government may, at any time, without

27
conditions or upon any conditions which the person sentenced accepts, suspend the execution of his
sentence or remit the whole or any part of the punishment to which he has been sentenced.
In the case of life imprisonment convicts, this remission can be done only after 14 years in jail as per
Section 433A of the CrPC. So, Option (b) is correct.

ADDITIONAL INFORMATION:
PARDONING POWERS OF GOVERNOR
Recently in The Supreme Court on January 8 set aside the remission of 11 convicts sentenced
News to life imprisonment during the 2002 communal riots in Gujarat. The Gujarat
government passed the remission order in August 2022.
About ➢ The Governor can pardon, reprieve, respite, remit, suspend or commute the
punishment or sentence of any person convicted of any offense against a state
law.
➢ He cannot pardon a death sentence. Even if a state law prescribes for death
sentence, the power to grant a pardon lies with the President and not the
governor. However, the governor can suspend, remit or commute a death
sentence.
➢ He cannot grant pardon, reprieve, respite, suspension, remission or
commutation with respect to punishment or sentence by a court-martial
(military court).

26. According to the Constitution of India, under which of the following conditions will a law made by a
State containing a provision repugnant to a law made by Parliament prevail in that State?
(a) When the House of the People has declared by resolution supported by an effective majority
(b) When the State law has received the assent of the Governor
(c) When the State law has been reserved for the consideration of the President and has
received his assent
(d) When the Council of States has declared by resolution supported by an effective majority
EXPLANATION:
Article 254(2) of the Constitution of India addresses the issue of repugnancy between laws made by the
Parliament and the State Legislatures on matters enumerated in the Concurrent List. The Concurrent
List contains subjects on which both the Central (Parliament) and State governments can legislate.
According to Article 254(2), If a State Legislature makes a law on a matter in the Concurrent List and
that law is inconsistent or repugnant to a law made by Parliament on the same matter or to an existing
law (law made by Parliament), then the State law will prevail in that State if it has received the President's
assent. . So, option (C) is correct.

27. Consider the following provisions with respect to the passing of bills in the State legislature with a
legislative council:
1. A Bill pending in the Legislative Council of a State which the Legislative Assembly has not passed
shall not lapse on a dissolution of the Assembly.
2. A Bill pending in the Legislature of a State shall not lapse by reason of the prorogation of either
of the Houses.
3. A bill that has been passed by the Legislative Assembly and is pending in the Legislative Council
shall lapse on a dissolution of the Assembly.
How many of the statements given above are not correct?
(a) Only one
28
(b) Only two
(c) All three
(d) None
EXPLANATION:
The legislative council, being a permanent house, is not subject to dissolution. Only the legislative
assembly is subject to dissolution. Unlike a prorogation, a dissolution ends the very life of the existing
House, and a new House is constituted after the general elections are held. The position with respect to
the lapsing of bills on the dissolution of the assembly is mentioned below:
➢ A Bill pending in the assembly lapses (whether originating in the assembly or transmitted to it by
the council).
➢ A Bill pending in the Legislative Council of a state but not passed by the legislative assembly does
not lapse on the dissolution of the assembly. So, Statement 1 is correct.
➢ A Bill passed by the assembly but pending in the council lapses. So, Statement 3 is correct.
➢ A Bill passed by the assembly (in a unicameral state) or passed by both the houses (in a bicameral
state) but pending assent of the governor or the President does not lapse.
➢ A Bill passed by the assembly (in a unicameral state) or passed by both the Houses (in a bicameral
state) but returned by the president for reconsideration of House does not lapse.
➢ A Bill pending in the Legislature of a State shall not lapse by reason of the prorogation of of either
of the Houses. So, Statement 2 is correct.

28. Consider the following statements:


Statement-I:
At present, Aadhaar is neither a proof of citizenship nor a proof of Date of Birth in India.
Statement-II:
The Union Ministry of Home Affairs has deleted the term 'Aadhaar' from the list of documents that
can be acceptable as proof of citizenship in India.
Which of the following is correct in respect of the above statements?
(a) Both Statement-I and Statement-II are correct and Statement-II is the correct Explanation for
Statement-I.
(b) Both Statement-I and Statement-II are correct and Statement-II is not the correct Explanation for
Statement-I.
(c) Statement-I is correct but Statement-II is incorrect.
(d) Statement-I is incorrect but Statement-II is correct.
EXPLANATION:
The new Aadhaar cards and PDF versions of the identity document have started to include the more
explicit and prominent disclaimer that they are "a proof of identity, not of citizenship or date of birth".
This made the government departments and other organizations not to use it for those purposes.
Aadhaar has never been proof of citizenship. In fact, Foreign Nationals are eligible to obtain one if they
have been living in India for half a year, but different government departments accept it for purposes
reserved for citizens or adults.
For instance, the Election Commission of India explicitly accepts Aadhaar as proof of date of birth for
enrolling people to vote. These new clarifications, prominently printed in the identity document, may
challenge such allowances. So, Statement I is correct.
The Employees' Provident Fund Organization (EPFO), which administers the mandatory retirement fund
for salaried employees in India, has stopped accepting Aadhaar as proof of date of birth. It is part of
India's Ministry of Labour and Employment (not the Union Ministry of Home Affairs).

29
Accordingly, Aadhaar has been removed from the list of acceptable documents for proof of date of birth
(DoB) for correction updation purposes, as mentioned in Table-B of Annexure -1 of the JD SOP under
reference. So, Statement II is incorrect.

ADDITIONAL INFORMATION:
AADHAAR
About ➢ Aadhaar number is a 12-digit random number issued by the UIDAI ("Authority") to
the residents of India after satisfying the verification process laid down by the
Authority.
➢ Any individual, irrespective of age and gender, who is a resident of India may
voluntarily enroll to obtain an Aadhaar number.
➢ A person willing to enroll has to provide minimal demographic and biometric
information during the enrolment process which is totally free of cost.
➢ An individual needs to enroll for Aadhaar only once and after de-duplication only
one Aadhaar shall be generated, as the uniqueness is achieved through the process
of demographic and biometric de-duplication.
➢ Aadhaar is a strategic policy tool for social and financial inclusion, public sector
delivery reforms, managing fiscal budgets, increase convenience and promote
hassle-free people-centric governance.

29. Consider the following committees :


1. Public accounts committee
2. Estimated Committee
3. Committee of public understanding
4. Department-related standing committee
In how many of the above committees does a Member of Parliament cease to be a Member if he is
appointed as a Minister?
(a) Only one
(b) Only two
(c) Only three
(d) All four
EXPLANATION:
The public accounts committee was set up first in 1921 under the provisions of the Government of India
Act of 1919 and has since been in existence. The members are elected by the Parliament every year from
amongst its members according to the principle of proportional representation by means of the single
transferable vote. The term of office of the members is one year. A minister cannot be elected as a member
of the committee.
Thus, a Member of Parliament ceases to be a Member of the Public Accounts Committee if he is appointed
as a Minister. So, Statement 1 is correct.
The Committee on Estimates constituted for the first time in 1950, is a Parliamentary Committee
consisting of 30 members, elected every year by the Lok Sabha from amongst its Members. The Speaker
appoints the Chairperson of the Committee from amongst its members. A Minister cannot be elected as
a member of the Committee. If a member, after selection to the Committee, is appointed a Minister, the
member ceases to be a Member of the Estimates Committee from the date of such appointment. So,
Statement 2 is correct.

30
The Committee on Public Undertakings was created in 1964 on the recommendation of the Krishna
Menon Committee to examine the reports and accounts of public undertakings. The members of this
committee are elected by the Parliament every year from amongst its members according to the principle
of proportional representation by means of a single transferable vote. Thus, all parties get due
representation in it.
A Minister is not eligible to be elected as a member of the Public Undertakings Committee. If a member
after election to the Committee is appointed a Minister, such member ceases to be a member of the
Committee from the date of such appointment. So, Statement 3 is correct.
On the recommendation of the Rules Committee of the Lok Sabha, 17 Departmentally-Related Standing
Committees (DRSCs) were set up in the Parliament in 1993. The main objective of the DRSCs is to secure
more accountability of the Executive (i.e., the Council of Ministers) to the Parliament, particularly
financial accountability. They also assist the Parliament in debating the budget more effectively.
A minister is not eligible to be nominated as a member of any of the standing committees. In case a
member, after his nomination to any of the standing committees, is appointed a minister, he then ceases
to be a member of the committee. So, Statement 4 is correct.

30. Consider the following statements :


1. The term Veto is not mentioned in the Indian Constitution.
2. Under the Veto power of the President, Money bills can be returned to the House of People for
reconsideration.
3. The President shall not exercise his/her Veto power for a returned ordinary bill which the
Parliament passes without any amendments.
How many of the statements given above are correct?
(a) Only one
(b) Only two
(c) All three
(d) None
EXPLANATION:
The term veto is defined as an official power or right to refuse to accept or allow something. The term
Veto is not mentioned in the Constitution of India. So, Statement 1 is correct.
Article 110 of the Constitution deals with the money bills. When a money bill is presented to the
president, he may either give his consent to the bill or withhold his assent to the bill but cannot return
the bill for reconsideration of the Houses. Normally, the president gives his consent to a money bill as it
is introduced in the Parliament with his prior permission. Thus, under the Veto power of the President,
Money bills cannot be returned to the House of People for reconsideration. So, Statement 2 is not
correct.
The President exercises a Suspensive veto when he returns an ordinary bill for reconsideration by the
Parliament. However, suppose the bill is passed again by the Parliament with or without amendments
and again presented to the President; in that case, the President must give his consent to the bill and
shall not exercise his/her Veto power for a returned ordinary bill. This means that the presidential veto
is overridden by a re-passage of the bill by the same ordinary majority (and not a higher majority as
required in the USA). So, Statement 3 is correct.

31
ADDITIONAL INFORMATION:
VETO POWER OF THE PRESIDENT
About ➢ A bill passed by the Parliament can become an act only if it receives the assent
of the President. When such a bill is presented to the President for his assent,
he has three alternatives (under Article 111 of the Constitution):
• He may give his consent to the bill
• He may withhold his assent to the bill
• He may return the bill (if it is not a Money bill) for reconsideration by the
Parliament.
➢ Thus, the President has the veto power over the bills passed by the Parliament.
That is, he can withhold his consent to the bills.
➢ The object of conferring this power on the President is two-fold:
• To prevent hasty and ill-considered legislation by the Parliament,
• To prevent legislation that may be unconstitutional.
➢ The veto power enjoyed by the executive in modern states can be classified into
the following four types:
• Absolute veto, that is, withholding of consent to the bill passed by the
legislature.
• Qualified veto, which the legislature can override with a higher majority.
• Suspensive veto, which the legislature can override with an ordinary
majority.
• Pocket veto, that is, taking no action on the bill passed by the legislature.
➢ Of the above four, the President of India is vested with three veto powers:
absolute veto, suspensive veto and pocket veto. There is no qualified veto in the
case of the Indian President; the American President possesses it.

31. With reference to the working of Cabinet Committees in India, consider the following statements:
1. Only Cabinet Ministers are eligible to be appointed as their members.
2. Cabinet Committees are empowered to make decisions on matters placed before it for
consideration.
3. Among others, the Cabinet Committee on Parliamentary Affairs is often referred to as a Super-
Cabinet.
Which of the above statements is/are not correct?
(a) 1 only
(b) 1 and 3 only
(c) 2 only
(d) 2 and 3 only
EXPLANATION:
The membership of the Cabinet Committees varies from three to eight. They usually include only Cabinet
Ministers. However, the non-cabinet Ministers are not debarred from their membership. They not only
include the Ministers in charge of subjects covered by them but also include other senior Ministers.
Thus, both cabinet and non-cabinet Ministers are eligible to be appointed as members of the Cabinet
Committees. So, Statement 1 is not correct.
The Cabinet works through various committees called cabinet committees. They are an organizational
device to reduce the enormous workload of the Cabinet.

32
Cabinet Committees are empowered to make decisions on matters placed before it for consideration.
They not only sort out issues and formulate proposals for the consideration of the Cabinet but also make
decisions. However, the Cabinet can review their decisions. So, Statement 2 is correct.
The four important cabinet committees are the Political Affairs Committee, the Economic Affairs
Committee, the Appointments Committee and Parliamentary Affairs Committee.
The Political Affairs Committee deals with all policy matters pertaining to domestic and foreign affairs.
The Prime Minister chairs it. Of all the Cabinet Committees, the most powerful is the Political Affairs
Committee (not the Committee on Parliamentary Affairs), often described as a “Super-Cabinet.”
So, Statement 3 is not correct.

ADDITIONAL INFORMATION:
CABINET COMMITTEES
About ➢ The Cabinet Committees are extra-constitutional in emergence. In other words, they
are not mentioned in the Constitution. However, the Rules of Business provide for
their establishment.
➢ They are of two types–standing and ad hoc. The former is permanent, while the latter
is temporary. The ad hoc committees are constituted from time to time to deal with
special problems. They are disbanded after their task is completed.
➢ They also facilitate in-depth examination of policy issues and effective coordination.
Head of ➢ The Prime Minister mostly heads the Cabinet Committees. Sometimes, other
Cabinet Cabinet Ministers, particularly the Home Minister or the Finance Minister, also act
Committees as their Chairman. But, in case the Prime Minister is a member of a committee, he
invariably presides over it.
➢ The Economic Affairs Committee directs and coordinates the governmental activities
in the economic sphere. It is chaired by the Prime Minister.
➢ The Appointments Committee decides all higher-level appointments in the Central
Secretariat, Public Enterprises, Banks and Financial Institutions. It is chaired by
the Prime Minister.
➢ Parliamentary Affairs Committee looks after the progress of government business in
the Parliament. It is chaired by the Home Minister.

32. The world's tallest statue of Dr. BR Ambedkar, known as the 'Statue of Social Justice,' was
inaugurated in which of the following states?
(a) New Delhi
(b) Maharashtra
(c) Bihar
(d) Andhra Pradesh
EXPLANATION:
The Government of Andhra Pradesh unveiled the world's tallest statue of Dr. BR Ambedkar on the
grounds of Swaraj Maida in Vijayawada. The Swaraj Maidan in the heart of Vijayawada will henceforth
be known as Dr. B.R. Ambedkar Swaraj Maidan.
The phenomenal Statue of Social Justice stands tall 125-ft. statue (excluding the 85-ft. concrete pedestal)
and is dedicated to Dr. B.R. Ambedkar who is famous as the Architect of the Indian Constitution. The
statue is India's tallest non-religious structure. It is made in India and is also the world's tallest statue
of Dr. Ambedkar. It comprises the Dr. B.R. Ambedkar Experience Centre, a 2,000-capacity convention
centre, a food court, a children's play area, waterbodies, a musical fountain, and long walkways with
greenery.
33
The structure of the building is made of a pile foundation with 539 piles of 30-meter-deep shear walls
and inclined RCC slabs and beams with an inclination of 50 degrees. The pedestal design replicates the
Kalachakra Maha Mandal of Buddhist architecture. The pedestal of the statue is a G+2 Isosceles
Trapezium shaped RCC framed structure. So, Option (d) is correct.

ADDITIONAL INFORMATION:
DR BHIM RAO AMBEDKAR
About ➢ Dr Bhim Rao Ambedkar (1891-1956) is considered the father of the Indian
Constitution and is also the best-known leader of the Dalits.
➢ Dr Ambedkar fought for the rights of the Dalit community.
➢ He was born into the Mahar caste, which was considered untouchable. The Mahars
were poor, owned no land and children born to them also had to do the work their
parents did. They lived in spaces outside the main village and were not allowed into
the village.
➢ Dr Ambedkar was the first person from his caste who completed his college
education and went to England to become a lawyer.
➢ He encouraged Dalits to send their children to school and college.
➢ He also urged Dalits to take on different kinds of government jobs in order to move
out of the caste system.
➢ He led many efforts of Dalits to gain entry into temples.
➢ Later in life, he converted to Buddhism in his search for a religion that treated all
members equally.
➢ Dr Ambedkar believed that Dalits must fight the caste system and work towards a
society based on respect not just for a few but for all persons.

33. Which one among the following commissions was set up in pursuance of a definite provision under
an article of the Constitution of India?
(a) University Grants commission
(b) National Commission of Women
(c) Election commission
(d) Central Vilgilance commission
EXPLANATION:
The University Grants Commission is a statutory organization under the Ministry of Education,
Government of India, established by an Act of Parliament in 1956 for the promotion and co-ordination
of University education and for the determination and maintenance of standards of teaching,
examination and research in Universities, and for the purpose of performing its functions under this Act.
In addition to providing grants to eligible universities and colleges, the Commission also advises the
Central and State Governments on the measures which are necessary for the development of Higher

34
Education. Thus, University Grants Commission was not set up in pursuance of a definite provision
under an article of the Constitution of India. So, Option (a) is not correct.
The National Commission for Women was set up as statutory body in January 1992 under the National
Commission for Women Act, 1990 to :
➢ review the Constitutional and Legal safeguards for women ;
➢ recommend remedial legislative measures ;
➢ facilitate redressal of grievances and
➢ advise the Government on all policy matters affecting women.
Thus, National Commission of Women was not set up in pursuance of a definite provision under an
article of the Constitution of India. So, Option (b) is not correct.
The Election Commission is a permanent and an independent body established by the Constitution of
India directly to ensure free and fair elections in the country. Article 324 of the Constitution provides
that the power of superintendence, direction and control of elections to parliament, state legislatures,
the office of president of India and the office of vice-president of India shall be vested in the election
commission. Thus, the Election Commission is an all-India body in the sense that it is common to both
the Central government and the state governments. Thus, Election commission was set up in pursuance
of a definite provision under an article of the Constitution of India. So, Option (c) is correct.
The Central Vilgilance commission is a statutory body and derives its powers from the Central Vigilance
Commission Act, 2003. The CBI derives its power from the Delhi Special Police Establishment Act,
1946. The Central Vigilance Commission (CVC) is an apex governmental body in India that is responsible
for promoting integrity, transparency, and accountability in the country's public administration. It was
established in 1964 as a result of the recommendations of the Santhanam Committee on Prevention of
Corruption. Thus, Central Vilgilance commission was not set up in pursuance of a definite provision
under an article of the Constitution of India. So, Option (d) is not correct.

34. With reference to the Constitution of India, the amendments to which of the following provisions are
not deemed to be amendments under Article 368?
1. Acquisition of Citizenship
2. Goods and Services Tax Council
3. Delimitation of constituencies
4. Conferment of more jurisdiction on the Supreme Court
5. Representation of states in Parliament
6. Elections to Parliament
Select the correct answer using the code given below:
(a) 1, 3, 4 and 6 only
(b) 1, 2, 3 and 4 only
(c) 2, 3 and 6 only
(d) 1, 4, 5 and 6 only
EXPLANATION:
The Constitution deals with the citizenship from Articles 5 to 11 under Part II. However, it contains
neither any permanent nor any elaborate provisions in this regard. It only identifies the persons who
became citizens of India at its commencement (i.e., on January 26, 1950).
It does not deal with the problem of acquisition or loss of citizenship subsequent to its commencement.
It empowers the Parliament to enact a law to provide for such matters and any other matter relating to
citizenship. Accordingly, the Parliament has enacted the Citizenship Act (1955), which has been
amended from time to time. Amendment related to the Acquisition of Citizenship is done by the simple
majority of the two Houses of Parliament outside the scope of Article 368. Thus, it is not deemed to be
amendments for the purposes of Article 368. So, Statement 1 is correct.

35
In order to implement GST, the Constitutional (122nd Amendment) Bill was introduced in the Parliament
and passed by the Rajya Sabha and Lok Sabha in August 2016. The Constitutional Amendment Bill was
passed by more than 15 states, and after that Hon’ble President gave assent to “The Constitution (One
Hundred And First Amendment) Act, 2016”.
Parliament and state legislatures will have concurrent powers to make laws on GST. Therefore,
Amendment to implement GST is done with special majority of Parliament and also with the consent of
half of the state legislatures by a simple majority. Thus, the amendments to the Tax regime are deemed
to be amendments for the purposes of Article 368. So, Statement 2 is not correct.
Under Article 82 of the Constitution, the Parliament, by law, enacts a Delimitation Act after every census.
After coming into force commencement of the Act, the Central Government constitutes a Delimitation
Commission. This Delimitation Commission demarcates the boundaries of the Parliamentary
Constituencies as per provisions of the Delimitation Act. The present delimitation of constituencies has
been done on the basis of 2001 census figures under the provisions of the Delimitation Act, 2002.
Notwithstanding the above, the Constitution of India was specifically amended in 2002 not to have
delimitation of constituencies till the first census after 2026. Therefore, Amendment related to the
delimitation of constituencies is done by the simple majority of the two Houses of Parliament outside the
scope of Article 368. So, Statement 3 is correct.
According to Article 138 of the Constitution, the Supreme Court shall have further jurisdiction and
powers with respect to any of the matters in the Union List as Parliament may by law confer.
The Supreme Court shall have such further jurisdiction and powers with respect to any matter as the
Government of India and the Government of any State may, by special agreement, confer if Parliament
by law provides for the exercise of such jurisdiction and powers by the Supreme Court. Thus, the
Conferment of more jurisdictions on Supreme Court provisions is not deemed to be an amendment to
Article 368 as it is made through the simple majority of the two Houses of Parliament. So, Statement 4
is correct.
The elected members of state legislative assemblies elect the representatives of states in the Rajya Sabha.
The election is held in accordance with the system of proportional representation by means of the single
transferable vote. The seats are allotted to the states in the Rajya Sabha on the basis of population.
The Fourth Schedule of the Constitution deals with the allocation of seats in the Rajya Sabha to the
states and union territories. The amendments to the representation of states, which provisions are
deemed to be amendments for Article 368 as it requires a special majority of Parliament and consent of
half the states of India. So, Statement 5 is not correct.
In Elections to Parliament, after every census, a readjustment is to be made in the following,
➢ The allocation of seats in the Lok Sabha to the states.
➢ Division of each state into territorial constituencies.
Parliament is empowered to determine the authority and the manner in which it is to be made.
Accordingly, the Parliament enacted the Delimitation Commission Acts in 1952, 1962, 1972 and 2002
for this purpose.
Provisions concerning the Elections to Parliament, such as Delimitation of Seats etc., are done by the
act of Parliament and are not deemed to be amendments to Article 368. So, Statement 6 is correct.

35. With reference to the Constitution of India, consider the following statements:
It shall be the fundamental duty of every citizen of India to :
1. Safeguard public property
2. Cast a vote during elections
3. Respect National Flag
4. Pay taxes to the Government

36
5. Engage in family planning after having two children
How many of the above statements are correct?
(a) Only two
(b) Only three
(c) Only four
(d) All five
EXPLANATION:
According to Article 51A, it shall be the duty of every citizen of India:
➢ To abide by the Constitution and respect its ideals and institutions, the National Flag and the
National Anthem. So, Statement 3 is correct.
➢ to cherish and follow the noble ideals that inspired the national struggle for freedom;
➢ to uphold and protect the sovereignty, unity and integrity of India;
➢ to defend the country and render national service when called upon to do so;
➢ to promote harmony and the spirit of common brotherhood amongst all the people of India,
transcending religious, linguistic and regional or sectional diversities and to renounce practices
derogatory to the dignity of women;
➢ to value and preserve the rich heritage of the country’s composite culture;
➢ to protect and improve the natural environment, including forests, lakes, rivers and wildlife and to
have compassion for living creatures;
➢ to develop scientific temper, humanism and the spirit of inquiry and reform;
➢ To safeguard public property and to abjure violence. So, Statement 1 is correct.
➢ to strive towards excellence in all spheres of individual and collective activity so that the nation
constantly rises to higher levels of endeavour and achievement,
➢ To provide opportunities for education to his child or ward between the ages of six and fourteen
years. The 86th Constitutional Amendment Act of 2002 added this duty.
The Swaran Singh Committee suggested the incorporation of eight Fundamental Duties in the
Constitution, but the 42nd Constitutional Amendment Act (1976) included ten Fundamental Duties.
Interestingly, certain recommendations of the Committee were not accepted by the Congress Party and,
hence, not incorporated into the Constitution. These include:
➢ The Parliament may provide for the imposition of such penalty or punishment as may be considered
appropriate for any non-compliance with or refusal to observe any of the duties.
➢ No law imposing such penalty or punishment shall be called into question in any court on the ground
of infringement of any of Fundamental Rights or on the ground of repugnancy to any other provision
of the Constitution.
➢ The duty to pay taxes should also be a Fundamental Duty of the citizens.
➢ In 1999, the Verma Committee recommended that the duty to vote in elections and participate in
the democratic process be included as a fundamental duty. The committee also recommended that
awareness about fundamental duties be created in educational institutions (these Verma Committee
on Fundamental Duties provisions are not included in the fundamental duties).
➢ Engaging in family planning after having two children is also not a fundamental duty.
So, Statements 2, 4 and 5 are not correct.

36. With reference to the Hague Apostille Convention, sometimes mentioned in the news, consider the
following statements:
1. It is a disarmament treaty that prohibits the development, production, acquisition, transfer,
stockpiling and use of biological and toxin weapons.

37
2. India is a signatory to the convention.
Which of the above statements is/are correct?
(a) 1 only
(b) 2 only
(c) Both 1 and 2
(d) Neither 1 nor 2
EXPLANATION:
The Hague Apostille Convention, 1961, abolishes the requirement for the legalization of foreign public
documents. Apostille is acceptable in all 105 member countries of the Hague Convention. Apostille is
done for personal documents like birth/death/marriage certificates, Affidavits, Power of Attorney, etc.
and educational documents like degrees, diplomas, matriculation and secondary-level certificates etc.
Since 2005, India has been a member of the Convention. As India is a member of the Hague Apostille
Convention, 1961, no further attestation or legalization of a document apostilled by a member country
should be required for using such apostilled document in India.
An apostilled document should, therefore, be treated as a legalized document for all purposes in India
by all concerned, in accordance with the international obligation under the Convention.
Therefore, the Hague Apostille Convention is not related to a disarmament treaty, and India is a signatory
to the convention. So, Statement 1 is not correct, and Statement 2 is correct.

ADDITIONAL INFORMATION:
THE BIOLOGICAL WEAPONS CONVENTION
About ➢ The Biological Weapons Convention (BWC) effectively prohibits the development,
production, acquisition, transfer, stockpiling and use of biological and toxin
weapons.
➢ It was the first multilateral disarmament treaty banning an entire category of
weapons of mass destruction (WMD).
➢ The BWC is a key element in the international community’s efforts to address WMD
proliferation, and it has established a strong norm against biological weapons.
➢ The Convention has reached almost universal membership with 185 States Parties
and four Signatory States.
➢ India is a party to this Convention.

37. Consider the following statements:


1. The 'Right to strike' is a fundamental right protected under Article 19(1)(c) of the Constitution of
India.
2. The 'Right to strike' has been recognized by the International Labour Organization and in India,
it is recognised under the Industrial Disputes Act.
Which of the above statements is/are correct?
(a) 1 only
(b) 2 only
(c) Both 1 and 2
(d) Neither 1 nor 2
EXPLANATION:
Article 19(1)(c) of the Constitution of India guarantees to all its citizens the right "to form associations
and unions or cooperative societies. It includes the right to form political parties, companies, partnership
firms, societies, clubs, organizations, trade unions, or any body of persons. It not only includes the right

38
to start an association or union but also to continue with the association or union as such. Further, it
covers the negative right of not to form or join an association or union.
The Supreme Court held that the trade unions have no guaranteed right to effective bargaining, right to
strike, or right to declare a lockout. The right to strike can be controlled by an appropriate industrial
law. Therefore, Right to Strike is a legal right but not the Fundamental right and the Supreme Court has
reiterated this view in various decisions. So, Statement 1 is not correct.
The right to strike is recognized by the International Labour Organization's supervisory bodies as an
intrinsic result of the right to organize, deriving from the right of workers' organizations to formulate
their Programmes of activities to further and defend the economic and social interests of their members.
In India, the right to protest is a fundamental right under Article 19 of the Constitution of India. However,
the right to strike is not a fundamental right but a legal right, and with this right, statutory restriction
is attached to the Industrial Dispute Act of 1947. So, Statement 2 is correct.

ADDITIONAL INFORMATION:
INTERNATIONAL LABOUR ORGANIZATION CONVENTIONS
About The eight Core Conventions of the ILO (also called fundamental/human rights conventions)
are:
The conventions ratified by India are:
➢ Forced Labour Convention (No. 29)
➢ Abolition of Forced Labour Convention (No.105)
➢ Equal Remuneration Convention (No.100)
➢ Discrimination (Employment Occupation) Convention (No.111)
➢ Minimum Age Convention (No.138)
➢ Worst forms of Child Labour Convention (No.182)
The conventions that have not been ratified by India are:
➢ Freedom of Association and Protection of Right to Organised Convention (No.87)
➢ Right to Organise and Collective Bargaining Convention (No.98)
➢ C155 - Occupational Safety and Health Convention, 1981 (No. 155)
➢ C187 - Promotional Framework for Occupational Safety and Health Convention, 2006
(No. 187)

38. With reference to Members of Parliament and State Legislatures, consider the following statements:
1. A person who is convicted and sentenced to not less than two years shall be disqualified from
being chosen as a Member from the date of conviction.
2. The disqualification of sitting members for any offense takes effect after three months from the
date of conviction.
Which of the statements given above is/are correct?
(a) 1 only
(b) 2 only
(c) Both 1 and 2
(d) Neither 1 nor 2

EXPLANATION:

39
Section 8(3) of the Representation of People Act (1951) provides that a person convicted of any offense and
sentenced to imprisonment for not less than two years shall be disqualified from the date of such
conviction and shall continue to be disqualified for a further period of six years since his release. So,
Statement 1 is correct.
Section 8(4) of the Representation of People Act (1951) states that the disqualification takes effect only
“after three months have elapsed” from the date of conviction. Within that period, the convicted lawmaker
could have filed an appeal against the sentence before a higher court.
However, this provision was struck down as “unconstitutional” in the Supreme Court’s landmark 2013
ruling in ‘Lily Thomas v Union of India.’ Before this judgment, convicted MPs could easily file an appeal
against their conviction and continue holding their official seats.
The effect of this judgment is that there is an instant disqualification of a sitting legislator as soon as he
is convicted. However, the Court made it clear that in the event of the appellate Court staying the
conviction and sentence, the disqualification would be lifted, and the membership would be restored to
him. So, Statement 2 is not correct.

ADDITIONAL INFORMATION:
DISQUALIFICATIONS OF PEOPLE REPRESENTATIVES
About Under the Constitution, a person shall be disqualified from being elected as a
member of Parliament/Legislature:
➢ If he holds any office of profit under the Union or state government (except
that of a minister or any other office exempted by Parliament).
➢ If he is of unsound mind and stands so declared by a court.
➢ If he is an undischarged insolvent.
➢ If he is not a citizen of India, has voluntarily acquired the citizenship of a
foreign state, or is under any acknowledgment of allegiance to a foreign state.
➢ If he is so disqualified under any law made by Parliament.

39. Consider the following:


1. Legislature of States
2. Central Bureau of Investigation
3. Cantonment Board
4. Town Area Committee
5. Gram Panchayat
6. Reserve Bank of India
The term ‘the State’ in Part III of the Indian Constitution includes how many of the given above
organs/agencies/bodies?
(a) Only three
(b) Only four
(c) Only five
(d) All six
EXPLANATION:
Article 12 of the Indian Constitution deals with the definition of ‘state.’ The term ‘state’ has been used in
different provisions concerning fundamental rights under part III. Accordingly, the state includes,
➢ The government and Parliament of India, that is, executive and legislative organs of the Union
government.
➢ Government and legislature of states, i.e., executive and legislative organs of state government. So,
Statement 1 is correct.

40
➢ All local authorities, that is, municipalities, panchayats, district boards, improvement trusts etc.,
Cantonment Board, and Town area Committee are the types of Municipalities for the administration
of urban areas. So, Statements 3, 4 and 5 are correct.
➢ All other authorities, that is, statutory or non-statutory authorities like Life Insurance Corporation
(LIC), ONGC, SAIL etc., the Central Bureau of Investigation is a non-statutory body, and it derives
powers from the Delhi Special Police Establishment Act, 1946. So, Statement 2 is correct.
➢ The Reserve Bank of India (RBI) is a statutory body established through the Reserve Bank of India Act
1935. So, Statement 6 is correct.
Thus, the State has been defined in a wider sense so as to include all its agencies. It is the actions of these
agencies that can be challenged in the courts as violating the Fundamental Rights. According to the
Supreme Court, even a private body or an agency working as an instrument of the State falls within the
meaning of the ‘State’ under Article 12 of the Indian Constitution.

40. Consider the following pairs:


Sea Bordering country
1. Baltic Sea - Russia
2. Black Sea - Ukraine
3. Mediterranean Sea - Israel
4. Red Sea - Iran
How many of the above pairs are correctly matched?
(a) Only one pair
(b) Only two pairs
(c) Only three pairs
(d) All four pairs
EXPLANATION:
The Baltic Sea is part of the North Atlantic Ocean, situated in Northern Europe. It has a coastline of
approximately 8000 km, shared by several countries, including Sweden, Poland, Lithuania, Latvia,
Finland, Estonia, Germany, Denmark and Russia. So, Pair (1) is correct.

The Black Sea is bordered to the north by Russia and Ukraine, Turkey to the south, Bulgaria to the west,
and Georgia to the east. Romania also has an opening to the Black Sea. The Black Sea basin covers an
area of approximately 2 million square kilometers and drains 25 countries.
So, Pair (2) is correct.

41
The European Countries bordering the Mediterranean Sea are Spain, France, Italy, Malta, Monaco,
Slovenia, Croatia, Bosnia and Herzegovina, Montenegro, Albania, and Greece.
The West Asian (Middle Eastern) countries bordering the Mediterranean Sea are Turkey, Syria, Lebanon,
Israel, the Palestine Gaza Strip and the divided island of Cyprus. Five North African nations have coasts
on the Mediterranean Sea: Morocco, Algeria, Tunisia, Libya and Egypt. Thus, Israel borders the
Mediterranean Sea. So, Pair (3) is correct.

A total of 6 countries of Asia and Africa border the Red Sea. The countries of Yemen and Saudi Arabia
border the Red Sea to the east. The Red Sea is bordered by Egypt to the north and west and by Sudan,
Eritrea, and Djibouti to the west. The Gulf of Aqaba is bordered by Egypt along with Israel, Jordan, and
Saudi Arabia.
Besides these 6 countries, some other areas, including Somalia, are also often recognized as Red Sea
Territories due to their proximity to the sea and the geographical similarities with the countries that border
the Red Sea. Thus, Iran is not bordered by the Mediterranean Sea.
So, Pair (4) is not correct.

42
41. Consider the following statements with respect to the Governor of a State:
1. The Constitution mandates a five-year term of office for the Governor.
2. The Governor appoints the State Election Commissioner on the recommendation of the Election
Commission of India.
3. The Governor, on the advice of the Council of Ministers, shall withhold the assent of the Private
Member Bill passed by the State Legislature.
How many of the statements given above are correct?
(a) Only one
(b) Only two
(c) All three
(d) None
EXPLANATION:
A governor holds office for a term of five years from the date on which he enters his office. However, this
term of five years is subject to the pleasure of the President. Further, he can resign at any time by
addressing a resignation letter to the President.
The Supreme Court held that the pleasure of the President is not justifiable. The governor has no security
of tenure and no fixed term of office. The President may remove him at any time. Thus, the Constitution
does not mandate a five-year term of office for the Governor. So, Statement 1 is not correct.
The governor appoints the state election commissioner and determines his conditions of service and
tenure of office. However, the state election commissioner can be removed only in a like manner and on
the like grounds as a judge of a high court. Thus, the Governor appoints the State Election Commissioner
but not on the recommendation of the Election Commission of India. So, Statement 2 is not correct.
Article 200 of the Constitution lays down that when a Bill passed by a State Legislature is presented to
the Governor for their consent, they have four alternatives. The governor,

43
➢ May give his assent to the Bill;
➢ May withhold his assent to the Bill;
➢ May return the Bill (if it is not a Money Bill) for reconsideration by the State Legislature; or
➢ May reserve the Bill for the consideration of the President.
Supreme Court, in various cases, including the Shamsher Singh case (1974), held that the Governor
does not exercise his discretionary powers while withholding assent or returning a Bill to the State
Legislature. They are required to act as per the advice of the Council of Ministers.
The situation of ‘withholding assent’ may arise in case of a Private Members’ Bill (bill by any Member of
State Legislature other than a Minister) passed by the State Legislature, which the council of ministers
does not want to be enacted into law. In such an instance, they would advise the Governor to ‘withhold
assent.’ So, Statement 3 is correct.

ADDITIONAL INFORMATION:
LEGISLATIVE POWERS OF GOVERNOR
About A governor is an integral part of the state legislature. In that capacity, he has the
following legislative powers and functions:
➢ He can summon or prorogue the state legislature and dissolve the state legislative
assembly.
➢ He can address the state legislature at the commencement of the first session after
each general election and the first session of each year.
➢ He can send messages to the house or houses of the state legislature with respect
to a bill pending in the legislature or otherwise.
➢ He can appoint any member of the State legislative assembly to preside over its
proceedings when the offices of both the Speaker and the Deputy Speaker fall
vacant. Similarly, he can appoint any member of the state legislature council to
preside over its proceedings when the offices of both Chairman and Deputy
Chairman fall vacant.
➢ He nominates one-sixth of the members of the state legislative council from
amongst persons having special knowledge or practical experience in literature,
science, art, cooperative movement and social service.
➢ He can nominate one member to the state legislature assembly from the Anglo-
Indian Community.
➢ He decides on the question of disqualification of members of the state legislature in
consultation with the Election Commission.
➢ When a bill is sent to the governor after the state legislature passes it, he can:
• Give his assent to the bill, or
• Withhold his assent to the bill, or
• Return the bill (if it is not a money bill) for reconsideration by the state
legislature. However, if the bill is passed again by the state legislature with or
without amendments, the governor has to give his consent to the bill or
• Reserve the bill for the consideration of the president. In one case, such
reservation is obligatory, that is, where the bill passed by the state legislature
endangers the position of the state high court.
➢ In addition, the governor can also reserve the bill if it is the following:
• Ultra-vires that is, against the provisions of the Constitution.
• Opposed to the Directive Principles of State Policy.
• Against the larger interest of the country.

44
• Of grave national importance.
• Dealing with compulsory acquisition of property under Article the Constitution.
➢ He can promulgate ordinances when the state legislature is not in session. The
state legislature must approve these ordinances within six weeks of their
reassembly. He can also withdraw an ordinance at any time. This is the most
important legislative power of the governor.
➢ He lays the reports of the State Finance Commission, the State Public Service
Commission and the Comptroller and Auditor General relating to the accounts of
the state before the state legislature.

42. Consider the following special provisions with respect to the state of Manipur in the Constitution of
India:
1. It provides for a minimum of thirty members in its legislative assembly.
2. The President may establish a committee to make rules for the Legislative Assembly of the State .
Which of the statements given above are correct?
(a) 1 only
(b) 2 only
(c) Both 1 and 2
(d) Neither 1 nor 2
EXPLANATION:
Article 371-C provides for special provisions for Manipur. It doesn’t provide any minimum members
criteria in its legislative assembly.
Instead, article 371-F of Sikkim (not for Manipur) provides special provisions for a minimum of thirty
members in its legislative assembly. So, Statement 1 is not correct.
Notwithstanding anything in the Constitution, the President may, by order made with respect to the
State of Manipur, provide for the constitution and functions of a committee of the Legislative Assembly
of the State consisting of members of that Assembly elected from the Hill Areas of the State, for the
modifications to be made in the rules of business of the Government and the rules of procedure of the
Legislative Assembly of the State and for any special responsibility of the Governor in order to secure the
proper functioning of such committee. So, Statement 2 is correct.

ADDITIONAL INFORMATION:
IMPORTANT SPECIAL PROVISIONS IN CONSTITUTION
Manipur Article 371-C makes the following special provisions for Manipur:
➢ The President is authorized to provide for the creation of a committee of the
Manipur Legislative Assembly consisting of the members elected from the Hill
Areas of the state.
➢ The President can also direct that the Governor shall have a special responsibility
to secure the proper functioning of the committee.
➢ The Governor should submit an annual report to the President regarding the
administration of the Hill Areas.
➢ The Central Government can give directions to the State Government as to the
administration of the Hill Areas.
Mizoram Article 371-G specifies the following special provisions for Mizoram:
➢ The Acts of Parliament relating to the following matters would not apply to Mizoram
unless the State Legislative Assembly so decides:

45
• Religious or social practices of the Mizos.
• Mizo customary law and procedure.
• Administration of civil and criminal justice involving decisions according to
Mizo customary law.
• Ownership and transfer of land.
➢ The Mizoram Legislative Assembly is to consist of not less than 40 members.
Arunachal Under Article 371-H, the following special provisions are made for Arunachal Pradesh:
➢ The Governor of Arunachal Pradesh shall have special responsibility for law and
order in the state. In the discharge of this responsibility, the Governor, after
consulting the Council of Ministers, exercises his judgment, and his decision is
final. This special responsibility of the Governor shall cease when the President so
directs.
➢ The Arunachal Pradesh Legislative Assembly is to consist of not less than 30
members.
Goa Article 371-I provides that the Goa Legislative Assembly is to consist of not less than
30 members.
Assam Under Article 371-B, the President is empowered to provide for the creation of a
committee of the Assam Legislative Assembly consisting of the members elected from
the Tribal Areas of the state and such other members as he may specify.

43. Consider the following statements with respect to the Doctrine of Basic Structure in India:
1. The Doctrine of Basic Structure is a form of judicial review.
2. It is applied to constitutional amendments Acts to ensure the amendment does not dilute the
fundamentals of the Constitution.
3. The right to property is not a part of the basic structure of the Indian Constitution.
How many of the statements given above are correct?
(a) Only one
(b) Only two
(c) All three
(d) None
EXPLANATION:
The Doctrine of Basic Structure is a form of judicial review that is used to test the legality of any
legislation by the courts. The Supreme Court evolved the doctrine in the 1973 landmark ruling
in Kesavananda Bharati v State of Kerala. In a 7-6 verdict, a 13-judge Constitution Bench ruled that the
'basic structure' of the Constitution is inviolable and could not be amended by Parliament. So,
Statement 1 is correct.
Based on the court judgement of Kuldip Nayar Case 2006, the Doctrine of Basic Structure is applied to
constitutional amendments to ensure the amendment does not dilute the fundamentals of the
Constitutional itself. The test is widely regarded as a check on majoritarian impulses of the Parliament
since it places substantive limits on the power to amend the Constitution. So, Statement 2 is correct.
Since the Constitution of India came into force in the 1950s, the right to property was given fundamental
status. Basically, two articles, Art. 31 and Art. 19(1)(f), ensure that any person's right against his
property remains protected.
In India, property is no longer a fundamental right but a Constitutional right after an amendment was
made in this regard in 1978. To this effect, Article 300-A was introduced in the Constitution in 1978 by

46
the 44th Constitutional Amendment Act, which states that 'no person shall be deprived of his property
save by authority of law.'
The Court ruled that while Parliament has vast powers to amend the Constitution, it cannot amend
certain "basic features." On land reforms, the Court upheld the amendment that removed the
fundamental right to property. The court ruled that, in spirit, the amendment would not violate the "basic
structure" of the Constitution. Thus, the right to property is not a part of the basic structure of the Indian
Constitution. So, Statement 3 is correct.

ADDITIONAL INFORMATION:
THE BASIC STRUCTURE
ELEMENTS The present position is that the Parliament, under Article 368, can amend any part of
the Constitution, including the Fundamental Rights, but without affecting the 'basic
structure' of the Constitution. However, the Supreme Court has yet to define or clarify
what constitutes the 'basic structure' of the Constitution. From the various judgments,
the following have emerged as 'basic features' of the Constitution or elements of the
'basic structure' of the Constitution:
➢ Supremacy of the Constitution
➢ Sovereign, democratic and republican nature of the Indian polity
➢ Secular Character of the Constitution
➢ Separation of powers between the legislature, the executive and the judiciary
➢ Federal character of the Constitution
➢ Unity and integrity of the nation
➢ Welfare state (socio-economic justice)
➢ Judicial review
➢ Freedom and dignity of the individual
➢ Parliamentary system
➢ Rule of law
➢ Harmony and balance between Fundamental Rights and Directive Principles
➢ Principle of equality
➢ Free and fair elections
➢ Independence of Judiciary
➢ Limited power of Parliament to amend the Constitution
➢ Effective access to justice
➢ Principles (or essence) underlying fundamental rights
➢ Powers of the Supreme Court under Articles 32, 136, 141 and 142
➢ Powers of the High Courts under Articles 226 and 227

44. Consider the following:


1. Switzerland
2. Germany
3. Iceland
4. Norway
5. Sweden
6. France
Which of the above countries form a part of the European Free Trade Association?
(a) 1, 3 and 4 only
(b) 2 and 6 only

47
(c) 1, 2, 4, 5 and 6 only
(d) 2, 4 and 5 only
EXPLANATION:
The European Free Trade Association (EFTA) is the intergovernmental organization of Iceland,
Liechtenstein, Norway and Switzerland. It was set up in 1960 by its then seven Member States for the
promotion of free trade and economic integration between its members.
The four EFTA States are competitive in several sectors vital to the global economy and score among the
highest in the world in competitiveness, wealth creation per inhabitant, life expectancy and quality of
life. EFTA was founded by the Stockholm Convention in 1960.
So, Option (a) is correct.

ADDITIONAL INFORMATION:
TRADE AND PARTNERSHIP AGREEMENT (TEPA)
About ➢ It is a type of economic partnership agreement.
➢ The recent Ministerial meeting marked a significant milestone in taking forward
the negotiations on a TEPA between India and EFTA.
➢ Both sides emphasized the importance of building their discussions on
principles of trust and respect for each other's sensitivities to achieve a fair,
equitable and balanced agreement.
➢ Indeed, a TEPA between EFTA and India could bring significant economic
benefits, such as integrated and resilient supply chains and new opportunities
for businesses and individuals on both sides, leading to increased trade and
investment flows, job creation, and economic growth.
➢ The delegations agreed to ramp up their efforts and continue their discussions
at a steady pace, with several more meetings planned over the coming months
to arrive at a common understanding of critical issues pertaining to TEPA.

45. The Parliamentary form of Government in India stresses which of the following principles/doctrines
concerning the relationship between legislative and executive organs?
(a) Principle of Procedure established by law
(b) Principle of cooperation and coordination
(c) Doctrine of Separation of powers
(d) Doctrine of Parliamentary sovereignty

EXPLANATION:

48
Article 21 of the Indian Constitution uses the phrase “procedure established by law” inspired by the
Japanese Constitution. It signifies that if the Parliament has passed a law by following the proper
procedure, then it will be a valid law.
In other words, ‘Procedure established by law’ is a principle that concerns the scope of judicial review
power of the Supreme Court of India and does not concern the relationship between the legislative and
executive organs of Government. So, Option (a) is not correct.
The Constitution of India has opted for the British Parliamentary System of Government rather than the
American Presidential System of Government.
The greatest advantage of the parliamentary system is that it ensures harmonious relationships,
cooperation and coordination between the legislative and executive organs of the government. The
executive is a part of the legislature, and both are interdependent at work. As a result, there is less scope
for disputes and conflicts between the two organs.
Therefore, the Parliamentary form of Government in India stresses principles of cooperation and
coordination concerning the relationship between legislative and executive organs. So, Option (b) is
correct.
The doctrine of separation of powers means the division of powers among the three independent organs
of the government, namely, the legislative, executive and judiciary. It is strictly followed in the American
model of the presidential system.
In India, a separation of functions rather than powers is followed. Unlike in the US, in India, the concept
of separation of powers is not adhered to strictly. However, a system of checks and balances has been
put in place in such a manner that the judiciary has the power to strike down any unconstitutional laws
passed by the legislature.
Thus, the Parliamentary form of Government in India does not stress the principle of the Doctrine of
Separation of Powers concerning the relationship between legislative and executive organs. So, Option
(c) is not correct.
The doctrine of Parliamentary Sovereignty makes Parliament the supreme legal authority which can
create or end any law. Generally, the courts cannot overrule its legislation, and no Parliament can pass
laws that future Parliaments cannot change. Parliamentary sovereignty is the most important part of the
UK constitution.
The Indian Parliament, on the other hand, cannot be regarded as a sovereign body in a similar sense as
there are ‘legal’ restrictions on its authority and jurisdiction. The parliamentary form of Government in
India does not stress the doctrine of Parliamentary Sovereignty. So, Option (d) is not correct.

46. Consider the following provisions:


1. Linking of Voter-Id with Aadhaar
2. Qualifying dates for enrolment as a voter increased from once in a year to four times in a year
3. Use of premises for election purposes
How many of the provisions were provided by the Election Laws (Amendment) Act of 2021?
(a) Only one
(b) Only two
(c) All three
(d) None
EXPLANATION:
The Election Laws (Amendment) Act 2021 was introduced in Lok Sabha on December 20, 2021. The Act
amends the Representation of the People Act of 1950 and the Representation of the People Act of 1951
to implement a few pertinent electoral reforms. The 1950 Act deals with the allocation of seats and

49
delimitation of constituencies for elections, qualifications of voters, and preparation of electoral rolls,
whereas the 1951 Act deals with the conduct of elections and offenses and disputes related to elections.
The Act seeks to achieve the following objectives:
➢ Linking of Voter-Id with Aadhaar: Amendment of section 23 of the Representation of the People Act,
1950, enabling the linking of electoral roll data with the Aadhaar to curb the menace of multiple
enrolments of the same person in different places. So, Statement 1 is correct.
➢ Qualifying dates for enrolment as a voter increased from once a year to four times in a year:
Amendment of clause (b) of section 14 of the RP Act, 1950 specifying the 1st day of January, 1st day
of April, 1st day of July and 1st day of October in a calendar year as qualifying dates in relation to
the preparation or revision of electoral rolls. So, Statement 2 is correct.
➢ Allowing spouses of Service Voters to cast their vote in person or through postal ballot: Amendment
in Section 20 of the RP Act, 1950 and Section 60 of the Representation of the People Act, 1951 for
substitution of the word ‘wife’ with the word ‘spouse’ making the statutes gender neutral.
➢ Use of premises for election purposes: Amendment of section 160 of the Representation of the People
Act, 1951 to enable the requisition of premises that are needed or likely to be needed for the purpose
of being used as polling stations, for counting, for storage of ballot boxes, voting machines (including
voter-verifiable paper audit trail) and poll-related material after a poll has been taken. So, Statement
3 is correct.

47. Consider the following statements with respect to the Metropolitan area:
1. A Metropolitan area is formed by including regions from one or more districts.
2. It shall consist of two or more Municipalities or Panchayats.
3. A Metropolitan area is specified by the State Legislature by law.
How many of the statements given above are correct?
(a) Only one
(b) Only two
(c) All three
(d) None
EXPLANATION:
According to the Article 243P of the Indian Constitution, 'Metropolitan area' means
➢ an area having a population of ten lakhs or more, comprised in one or more districts and
➢ consisting of two or more Municipalities or Panchayats or other contiguous areas, So,
Statements 1 and 2 are correct.
➢ specified by the Governor (not by the State Legislature by law) by public notification to be
Metropolitan area. So,Statement 3 is not correct.
Article 243ZE of the Indian constitution states that there shall be constituted in every Metropolitan area
a Metropolitan Planning Committee to prepare a draft development plan for the Metropolitan area as a
whole.

ADDITIONAL INFORMATION:
METROPOLITAN PLANNING COMMITTEE
About Every metropolitan area shall have a metropolitan planning committee to prepare a draft
development plan. The state legislature may make provisions with respect to the
following:
➢ The composition of such committees;
➢ The manner of election of members of such committees;

50
➢ The representation in such committees of the Central government, state government
and other organisations;
➢ The functions of such committees in relation to planning and coordination for the
metropolitan area;
➢ The manner of election of chairpersons of such committees.
Article 243ZE of the Indian constitution lies down those two-thirds of the members of a
metropolitan planning committee should be elected by the elected members of the
municipalities and chairpersons of the panchayats in the metropolitan area from
amongst themselves. The representation of these members in the committee should be
in proportion to the ratio between the population of the municipalities and the
panchayats in that metropolitan area.

48. Consider the following statements with reference to the Republic Day of India:
1. The Republic Day is celebrated to commemorate the date of the adoption of the Constitution of
India.
2. The Red Fort was chosen as the site for the Republic Day celebrations due to its profound
historical significance.
3. The President of India hoists the National flag, which is positioned at the top of a pole.
How many of the above statements are correct?
(a) Only one
(b) Only two
(c) All three
(d) None
EXPLANATION:
After gaining independence from British rule, the new charter was drafted by using the drafting
committee helmed by means of Dr BR Ambedkar. The Indian Constitution went into effect on January
26 1950, which affirmed India's life as an unbiased republic. Therefore, Republic Day is celebrated to
commemorate the 'date of commencement' of the Constitution of India on January 26, 1950. January
26 was selected because the date is on this day in 1930 when the Indian National Congress proclaimed
Purna Swaraj, the assertion of India's independence from colonial rule.
Republic Day is a source of colossal satisfaction for Indians globally, symbolizing the cease of the war
for independence and the establishment of a self-governing country. It serves as a tribute to the
democratic ideas laid out in our Constitution, guiding the nation. Additionally, Republic Day showcases
India's cultural richness and unity in diversity.
So, Statement 1 is not correct.
The Kartavya Path, formerly known as the Rajpath (not the Red Fort), was chosen as the site for the
Republic Day celebrations due to its profound historical significance. Republic Day celebrations span
across the country, with the grand parade in the capital being the focal point. The armed forces presence,
folk dances, and patriotic songs contribute to the occasion's beauty and spirit.
So, Statement 2 is not correct.

51
On Republic Day every year, the President of India, who is the head of state, 'unfurls' the tricolor, while
on Independence Day, the prime minister, who heads the Union government, 'hoists' the national flag.
On August 15 (Independence Day), the flag, positioned at the bottom of the pole, is raised ('hoisted') by
the prime minister from the bottom to the top. Whereas, on January 26(Republic Day), the flag is folded
or rolled up and attached to the top of a pole. It is then unveiled ('unfurled') by the President, who does
so without pulling it up. So, Statement 3 is not correct.

ADDITIONAL INFORMATION:
THE REPUBLIC DAY
About ➢ The roots of Republic Day trace back to India's struggle for independence from
British rule.
➢ Following freedom on August 15, 1947, the nation operated under the Government
of India Act, 1935, lacking a permanent constitution.
➢ Led by Dr. B.R. Ambedkar, a dedicated drafting committee worked tirelessly. On
January 26, 1950, the Constitution of India was officially adopted, signifying a
transformative moment as India became a sovereign, socialist, secular, and
democratic republic.
➢ The themes for Republic Day 2024, "Viksit Bharat" (Developed India) and "Bharat -
Loktantra ki Matruka" (India - Mother of Democracy), encapsulate the aspirations
and democratic ethos of the nation.
➢ The chief guest for the Republic Day celebrations this year is French President
Emmanuel Macron.
➢ The President of India distributes Padma Awards to the deserving civilians of the
country, and brave soldiers are awarded Paramvir Chakra, Ashok Chakra and Vir
Chakra.

49. Consider the following statements:


1. The directives to the Union and States are provided only in Part IV of the Indian Constitution.
2. Provisions of Part IV are not enforceable by any courts in India.
3. Providing instruction in the mother tongue at the primary stage of education to children belonging
to linguistic minority groups is also a directive enshrined in the Indian Constitution.
How many of the statements given above are correct?
(a) Only one
(b) Only two
(c) All three
(d) None

52
EXPLANATION:
The Directive Principles of State Policy are enumerated in Part IV of the Constitution from Articles 36 to
51. The directives to the Union and States are not enshrined only in Part IV of the Constitution. Apart
from the directive principles in part IV ,there are some of the directives in other parts of the
constitution.They are:
➢ Claims of SCs and STs to services in Article 335 in Part XVI
➢ Instruction in mother tongue in Article 350-A in Part XVII
➢ Development of th Hindi Language in Article 350-A in Part XVII
So, Statement 1 is not correct.
The Directive Principles are non-justiciable in nature; that is, they are not legally enforceable by the
courts for their violation. Therefore, the government (Central, State and Local) cannot be compelled to
implement them. Nevertheless, the Constitution (Article 37) itself says that these principles are
fundamental in the governance of the country, and it shall be the duty of the State to apply these
principles in making laws. Thus, the Provisions of Part IV are not enforceable by any courts in India. So,
Statement 2 is correct.
Apart from the Directives included in Part IV, there are some other Directives contained in other parts
of the Constitution. It includes instruction in the mother tongue. It shall be the endeavor of every state
and every local authority within the state to provide adequate facilities for instruction in the
mothertongue at the primary stage of education to children belonging to linguistic minority groups
(Article 350-A in Part XVII). So, Statement 3 is correct.
ADDITIONAL INFORMATION:
DIRECTIVE PRINCIPLES OF STATE POLICY
About ➢ The framers of the Constitution borrowed the idea of Directive Principles of State
Policy from the Irish Constitution of 1937, which had copied it from the Spanish
Constitution.
➢ Dr. B.R. Ambedkar described these principles as ‘novel features’ of the Indian
Constitution.
➢ The Directive Principles, along with the Fundamental Rights, contain the philosophy
of the Constitution and are the soul of the Constitution.
➢ They aim to establish social and economic democracy in the country and have moral
and political sanctions.
➢ They promote the welfare of the community. Hence, they are societarian and
socialistic.
➢ They require legislation for their implementation. They are not automatically
enforced.
➢ The courts cannot declare a law violative of any of the Directive Principles as
unconstitutional and invalid. However, they can uphold the validity of a law on the
ground that it was enacted to give effect to a directive.

50. With reference to the election of Members to the Council of States, consider the following statements:
1. Only elected Members of the State Legislative Assemblies can vote in a Rajya Sabha election.
2. When vacancies arise due to resignation, death or disqualification of members, new Members
serve out only the remainder of their predecessor’s term.
3. The ‘none of the above’ option is not available in the election of Members to the Council of States.
4. A Member of the State Legislative Assembly not voting for the party candidate will not attract
disqualification under the anti-defection law.
How many of the statements given above are correct?
53
(a) Only one
(b) Only two
(c) Only three
(d) All four
EXPLANATION:
The Rajya Sabha (Upper House) represents the state of India. It is an indirectly elected body. The
representatives of states in the Rajya Sabha are elected by the elected members of state legislative
assemblies. The election is held in accordance with the system of proportional representation by means
of the single transferable vote. The seats are allotted to the states in the Rajya Sabha on the basis of
population. So, Statement 1 is correct.
Rajya Sabha is a permanent House and cannot be dissolved. To ensure continuity, one-third of its
members retire after every second year, under Article 83(1) of the Constitution, and “biennial elections”
are held to fill these vacancies.
Vacancies arising due to resignation, death or disqualification are filled up through by-polls, and those
elected serve out the remainder of their predecessors’ term. So, Statement 2 is correct.
The Election Commission of India granted the Rajya Sabha members the option to press the None Of
The Above (NOTA) button in the Upper House polls through two circulars in 2014 and 2015. However,
in 2018, the Supreme Court of India struck down the provision, holding that the NOTA option is only for
general elections held on the basis of universal adult suffrage and cannot be applied to indirect elections
based on proportional representation.
Thus, the ‘none of the above’ option is not available in the election of Members to the Council of States
(Rajya Sabha). So, Statement 3 is correct.
The Supreme Court ruled that a Member of the State Legislative Assembly (MLA) not voting for the party
candidate will not attract disqualification under the anti-defection law. As voters, MLAs retain their
freedom to vote for a candidate of their choice. However, the Court observed that since the party would
know who voted against its candidate, it is free to take disciplinary action against the legislator
concerned. So, Statement 4 is correct.

ADDITIONAL INFORMATION:
RAJYA SABHA
About ➢ The Fourth Schedule to the Constitution provides for the allocation of Rajya Sabha
seats to the states and Union Territories on the basis of the population of each state.
➢ The maximum strength of the Rajya Sabha is fixed at 250, out of which 238 are to
be the representatives of the states and union territories (elected indirectly), and the
president nominates 12.
➢ At present, the Rajya Sabha has 245 members. Of these, 229 members represent
the states, 4 members represent the union territories, and the president nominates
12 members.
➢ The president nominates 12 members to the Rajya Sabha from people who have
special knowledge or practical experience in art, literature, science and social
service.
➢ The rationale behind this principle of nomination is to provide eminent persons a
place in the Rajya Sabha without going through the process of election.
Open ballot in ➢ The Rajya Sabha polls have a system of open ballot, but it is a limited form of
Rajya Sabha openness.
➢ As a measure to check rampant cross-voting, which was taken to mean that the vote
had been purchased by corrupt means, the system of each party MLA showing his

54
or her marked ballots to the party’s authorized agent before they are put into the
ballot box has been introduced.
➢ Showing a marked ballot to anyone other than one’s own party’s authorized agent
will render the vote invalid.
➢ Not showing the ballot to the authorized agent will also mean that the vote cannot
be counted.
➢ And independent candidates are barred from showing their ballots to anyone.

51. In which of the following rights, the state can make reasonable restrictions on the grounds of public
order?
1. Right to assemble peaceably
2. Right to move freely throughout the territory of India
3. Right to practice any profession
4. Right to practice and propagate religion
Select the correct answer using the code given below:
(a) 1 and 4 only
(b) 2 and 3 only
(c) 2, 3 and 4 only
(d) 1, 2, 3 and 4
EXPLANATION:
“Reasonable restrictions” are outlined in Article 19 (2) of the Indian Constitution. The State can impose
‘reasonable’ restrictions on the enjoyment of the six rights mentioned in Article 19 only on the grounds
mentioned in this article and not on any other grounds. These six rights are protected against only state
action and not private individuals. Moreover, these rights are available only to the citizens but not to
foreigners or legal persons like companies or corporations etc.,
Article 19(1) (b) speaks about the Right to assemble peaceably and without arms. The ground of
restrictions under this article is the sovereignty and integrity of India (added by the 16th Amendment Act,
1963) and public order. So, Statement 1 is correct.
Article 19(1) (d) speaks about the Right to move freely throughout the territory of India. The ground of
restriction is placed under Article 19(5). They are,
➢ In the interest of the general public
➢ Protection of the interests of any scheduled tribe.
Therefore, the Right to move freely throughout the territory of India is not restricted under the grounds
of the public order. So, Statement 2 is not correct.
Article 19(1) (g) speaks about the Right to practice any profession or to carry on any occupation, trade
or business. The ground of restriction is placed under Article 19(6). They are,
In the interests of the general public
Requirement of necessary professional or technical qualifications
Any trade, business industry or service carried on by the state to the exclusion of citizens (added by the
1st Amendment Act, 1951).
Therefore, the Right to practice any profession or to carry on any occupation, trade or business is not
restricted under the grounds of the public order. So, Statement 3 is not correct.
Article 25 of the Indian Constitution says that all persons are equally entitled to freedom of conscience
and the right to freely profess, practice and propagate religion. The implications of these are Freedom of
conscience, Right to profess, Right to practice, and Right to propagate. These rights are available to all
55
persons (citizens as well as non-citizens). However, these rights are subject to public order, morality,
health and other provisions relating to fundamental rights. Further state is permitted to,
➢ Regulate or restrict any economic, financial, political or other secular activity associated with
religious practice and
➢ Provide for social welfare and reform or throw open Hindu religious institutions of a public character
to all classes and sections of Hindus.
Therefore, the Right to practice and propagate religion is restricted under the grounds of the public order.
So, Statement 4 is correct.

52. Which one of the following suggested the establishment of a permanent commission exclusively for
Nomadic, Semi-Nomadic and De-Notified Tribes while proposing their non-inclusion under the
SC/ST/OBC category?
(a) First Administrative Reforms Commission (1966)
(b) Ranganath Misra Commission (2004)
(c) Rohini Commission (2017)
(d) Idate Commission (2018)
EXPLANATION:
The first Administrative Reforms Commission (ARC) as the Commission of Inquiry was set up by the
Ministry of Home Affairs, Government of India, on January 5, 1966. It was initially chaired by Morarji R.
Desai and later on by Hanumanthaiya, who became its Chairman when Morarji R. Desai became the
Deputy PM of India.
The Commission was mandated to consider the need to ensure the highest standards of efficiency and
integrity in public services. It also aimed to make public administration a fit instrument for carrying out
the social and economic policies of the Government and achieving social and economic goals of
development as one which is responsive to the people. So, Option (a) is not correct.
The Government of India constituted the National Commission for Religious and Linguistic Minorities on
October 29, 2004, to look into various issues related to the linguistic and religious minorities in the
country. Former CJI Ranganath Misra chaired it. Thus, it is not related to Nomadic, Semi-Nomadic and
De-Notified Tribes. So, Option (b) is not correct.
The Rohini Commission was instituted on October 2, 2017, led by Justice G Rohini, the former Chief
Justice of the Delhi High Court. This four-member commission was formed to ensure a more equitable
distribution of reservation benefits among the Other Backward Classes (OBCs) in India. The commission
was set up by the President of India, mandated under Article 340 of the Indian Constitution, which
reads, "The President may, by order, appoint a Commission consisting of such persons as he thinks fit
to investigate the conditions of socially and educationally backward classes within the territory of India
and the difficulties under which they labour and to make recommendations as to the steps that should
be taken by the Union or any State to remove such difficulties and to improve their condition." So,
Option (c) is not correct.
Recently, in January 2024, The National Human Rights Commission (NHRC) stressed the need to
implement the Idate Commission report that has recommended setting up a permanent commission for
Nomadic, Semi Nomadic, and De-Notified Tribes (NTs, SNTs, and DNTs) in India.
The report of the Bhiku Ramji Idate Commission made the recommendation to set up a permanent
commission for the communities on the lines of similar commissions for Scheduled Castes, Scheduled
Tribes, and Other Backward Classes. In its report submitted to the social justice ministry in January
2018, the Idate Commission said such a permanent commission should have a prominent community
leader as its chairperson and a senior Union government bureaucrat, an anthropologist, and a sociologist
as members. So, Option (d) is correct.

56
53. Consider the following statements:
1. The interim budget was mentioned in the Article 112 of the Indian Constitution.
2. The annual budget covers all aspects of government finances, while the interim budget focuses
primarily on maintaining essential spending on ongoing schemes and critical public services.
3. Unlike the Union budget, the interim budget is not debatable in the Lok Sabha before passage.
How many statements given above are correct?
(a) Only one
(b) Only two
(c) All three
(d) None
EXPLANATION:
An interim budget called a short-term financial plan is shown before the general elections that shall be
conducted in a new government. It could be viewed as a temporary solution to mention the expenses of
the government till the new government takes office and makes a new budget. Due to the possibility of
an executive change after the polls, an incumbent administration could not show the complete budget
in an election year. Therefore, an interim budget is required.
As there is no constitutional provision for an interim budget, the Centre might opt to utilize the votes on
account provision to recommend the Lower House to approve the funds needed for the transition period
till the following administration submits a comprehensive budget. Thus, the interim budget was not
mentioned in the Indian Constitution. So, Statement 1 is not correct.
An annual budget is a detailed, comprehensive plan, while an interim budget is a temporary stopgap
measure.
An annual budget covers all aspects of government finances, including revenue generation, expenditure
allocations, and policy announcements. In contrast, the interim budget focuses primarily on maintaining
essential spending on ongoing schemes and critical public services until the new government takes
charge. So, Statement 2 is correct.
Both the Union budget and the interim budget undergo parliamentary scrutiny, but the intensity differs.
The annual budget faces rigorous debate and analysis, while the interim budget receives less scrutiny
due to its limited scope and temporary nature.
Article 112 of the Indian Constitution, a statement of the estimated receipts and expenditure of the
Government of India for a specific financial year referred to as ‘annual financial statement’ is laid before
both Houses of Parliament.
Similarly, the interim budget too will be presented to both Houses on February 1 by the Finance Minister,
put to vote and then sent for Presidential approval. It will list the current status of the Indian economy,
its projections and the government’s projected expenditure.
Thus, both the Union and interim budgets are debatable in the Lok Sabha before passage. So,
Statement 3 is not correct.

ADDITIONAL INFORMATION:
ANNUAL FINANCIAL STATEMENT
About ➢ The Constitution refers to the budget as the ‘annual financial statement.’ In other
words, the term ‘budget’ has nowhere been used in the Constitution and has been
dealt with in Article 112 of the Constitution.

57
➢ The budget is a statement of the estimated receipts and expenditures of the
Government of India in a financial year, which begins on April 1 and ends on March
31 of the following year.
➢ In addition to the estimates of receipts and expenditures, the budget contains
certain other elements. Overall, the budget contains the following:
• Estimates of revenue and capital receipts.
• Ways and means to raise the revenue.
• Estimates of expenditure.
• Details of the actual receipts and expenditures of the closing financial year
and the reasons for any deficit or surplus in that year.
• The economic and financial policy of the coming year, that is, taxation
proposals, prospects of revenue, spending program and introduction of new
schemes/projects.
➢ Till 2017, the Government of India had two budgets, namely, the Railway Budget
and the General Budget. While the former consisted of the estimates of receipts
and expenditures of only the Ministry of Railways, the latter consisted of the
estimates of receipts and expenditures of all the ministries of the Government of
India (except the railways).

54. Consider the following election officers:


1. Electoral Registration Officer
2. Presiding officer
3. Returning officer
4. Election Expenditure Observers
5. District Election Officer
The Election Commission of India appoints or designates officers of the government for how many of
the posts given above?
(a) Only two
(b) Only three
(c) Only four
(d) All five
EXPLANATION:
Under Article 324 of the Constitution of India, the Election Commission of India is vested with the power
of superintendence, direction and control of conducting the elections to the Lok Sabha and State
Legislative Assemblies.
The Electoral Registration Officer is responsible for the preparation of electoral rolls for a Parliamentary
/ assembly constituency. The Election Commission of India, in consultation with the State / UT
government, appoints an officer of the government or the local authorities as the Electoral Registration
Officer. In addition, the Election Commission of India also appoints one or more Assistant Electoral
Registration Officers to assist the Electoral Registration Officer in the performance of his functions in
the matter of preparation / revision of electoral rolls. So, Statement 1 is correct.
The Presiding Officer with the assistance of polling officers conducts the poll at a polling station. The
District Election Officer appoints the Presiding Officers and the Polling Officers. In the case of Union
Territories, such appointments are made by the Returning Officers. Thus, Presiding officers are not
appointed by the Election Commission of India. So, Statement 2 is not correct.
The Returning Officer of a Parliamentary or assembly constituency is responsible for the conduct of
elections in the Parliamentary or assembly constituency concerned. The Election Commission of India

58
nominates or designates an officer of the Government or a local authority as the Returning Officer for
each of the assembly and parliamentary constituencies in consultation with the State Government /
Union Territory Administration. In addition, the Election Commission of India also appoints one or more
Assistant Returning Officers for each of the assembly and Parliamentary constituencies to assist the
Returning Officer in the performance of his functions in connection with the conduct of elections. So,
Statement 3 is correct.
The Election Commission of India (ECI) nominates officers of Government as Observers for Parliamentary
and Assembly Constituencies. Expenditure Observers from Central Government Services are appointed
by the ECI to keep a close watch on election expenditure of the candidates and to ensure that no
inducement is offered to the electors during the entire election process for exercise of their franchise. So,
Statement 4 is correct.
The Election Commission of India nominates or designates an officer of the state Government as the
District Election Officer in consultation with the state government. Subject to the superintendence,
direction and control of the Chief Electoral Officer, the District Election Officer supervises the election
work of a district. So, Statement 5 is correct.

55. Consider the following statements with respect to the First-past-post system:
1. The constitution has adopted territorial representation for the elections of Parliament.
2. In this system, all the sections of people get representation in the legislature.
Which of the statements given above is/are correct?
(a) 1 only
(b) 2 only
(c) Both 1 and 2
(d) Neither 1 nor 2
EXPLANATION:
The Constitution has adopted the system of proportional representation in the case of Rajya Sabha, it
has not preferred the same system in the case of Lok Sabha. Instead, it has adopted the system of
territorial representation (First-past-the-post system) for the election of members to the Lok Sabha.
Thus, the constitution has not adopted territorial representation for the elections of both house of the
parliament but only for Lok Sabha elections. So, Statement 1 is not correct.
Under territorial representation, every member of the legislature represents a geographical area known
as a constituency. From each constituency, only one representative is elected. Hence, such a
constituency is known as a single-member constituency. In this system, a candidate who secures a
majority of votes is declared elected. This simple majority system of representation does not represent
the whole electorate. In other words, it does not secure due representation to minorities (small groups).
The system of proportional representation aims at removing the defects of territorial representation.
Under this system, all sections of the people get representation in proportion to their number. Even the
smallest section of the population gets its due share of representation in the legislature.
Thus, all the sections of people get representation in the legislature under the system of proportional
representation (not under the First-past-post system). So, Statement 2 is not correct.

ADDITIONAL INFORMATION:
SYSTEM OF PROPORTIONAL REPRESENTATION
About ➢ There are two kinds of proportional representation, namely, single transferable vote
system and list system.

59
➢ In India, the first kind is adopted for the election of members to the Rajya Sabha and
state legislative council and for electing the President and the Vice-President.
➢ Though some members of the Constituent Assembly had advocated the system of
proportional representation for the election of members to the Lok Sabha, the
Constitution has not adopted the system due to two reasons.
• Difficult for the voters to understand the system (which is complicated) due to low
literacy scale in the country.
• Unsuitability to the parliamentary government due to the tendency of the system to
multiply political parties leading to instability in government.
➢ Additionally, the system of proportional representation has the following demerits:
• It is highly expensive.
• It does not give any scope for organising by-elections.
• It eliminates intimate contacts between voters and representatives.
• It promotes minority thinking and group interests.
• It increases the significance of party system and decreases that of voter.

56. Consider the following pairs :


S.No. GI tagged product State it belongs to
1. Red ant chutney - Jharkhand
2. Tangail Saree - Karnataka
3. Lanjia Saura Paintings - Odisha
4. Ramban Anardana - Himachal Pradesh
How many of the pairs given above are correct?
(a) Only one
(b) Only two
(c) Only three
(d) All four
EXPLANATION:
Kai Chutney, also known as Red Ant Chutney from Odisha (not Jharhand), recently received the
Geographical Indication (GI) tag for its unique taste and texture. Kai chutney is a classic coarse chutney
made with a melange of spices, herbs, and Weaver Red Ants.
According to health experts, this chutney is packed with nutrients like protein, calcium, zinc, Vitamin
B-12, iron, magnesium, potassium etc. So, Pair (1) is not correct.

The three varieties of handloom sarees, namely, Tangail, Korial and Garad from West Bengal, have
received the GI tag. The Tangail sarees are woven in the Nadia and Purba Bardhhaman districts. The

60
hugely popular Tangail cotton sarees have a finer count and are decorated with extra warp designs using
colored yarn. Therefore, the tangail sarees that recently received the GI tag are from the state of West
Bengal (not Karnataka). So, Pair (2) is not correct.

Lanjia Saura's painting, which received the GI tag, is from the state of Odisha. These paintings are a
style of mural art. The art form belongs to the 'Lanjia Saura' or 'Lanjia Savara/Sabara' tribe, one of the
PVTGs of Odisha's Rayagada district. The paintings were originally in the form of exterior murals in tribal
homes. The pattern was white paintings over a crimson-maroon background. Those paintings, also called
ekons or the idital, have a significant spiritual importance for the tribe.
So, Pair (3) is correct.

Ramban Anardana, which received the GI tag, is from the state of Jammu and Kashmir (not Himachal
Pradesh). Anardana is the wild sour pomegranate produced in the Ramban district of Jammu and
Kashmir. The fruit was harvested in September /October, and its seeds were sun-dried to make
Anardana. It was used in the preparation of. The rind of the wild pomegranate is also sold in markets,
and it is used in making medicines. So, Pair (4) is not correct.

61
ADDITIONAL INFORMATION:
GEOGRAPHICAL INDICATION TAG
Recently in Recently, Jaggery, painting, and shawl among 7 Odisha products to get GI tag.
News
About The GI tag was given to Dungaria Kondh embroidered shawl, Dhenkanal Magji (food),
Nayagarh Kanteimundi Brinjal and Koraput Kalajeera Rice. All these products are from
Odisha. With this, Odisha has received 25 GI tags to date.
➢ Dungaria Kondh embroidered shawl from Odisha received the Geographical
Indication (GI) tag. The women of Dungaria Kondh, particularly the vulnerable tribal
group (PVTG) living in the Niyamgiri Hills of Rayagada and Kalahandi districts, craft
embroidery shawls, which are gifted to guests as a mark of respect and affection.

➢ Kalajeera Rice is grown by tribal farmers in Koraput district. This particular variety
is in demand as locals believe it has medicinal properties such as improving
memory, controlling diabetes and increasing hemoglobin levels.
➢ Kanteimundi Brinjals, which has seeds and prickly thorns, is said to have originated
in the Badabanapur and Ratnapur areas of Nayagarh district and has a unique
taste.

62
➢ Odisha 'Khajuri Guda' or jaggery, which originated in the Gajapati district, is a
natural sweetener obtained from date palm trees.
➢ Magji, a sweet item from the Dhenkanal district, is made of buffalo milk cheese.

57. Which of the following Schedules of the Indian Constitution supports the concept of decentralized
administration?
1. Sixth Schedule
2. Eleventh Schedule
3. Twelfth Schedule
Select the correct answer using the code given below:
(a) 1 and 2 only
(b) 2 and 3 only
(c) 1 and 3 only
(d) 1, 2 and 3
EXPLANATION:
The Sixth Schedule under Article 244 provides for the formation of autonomous administrative divisions
— Autonomous District Councils (ADCs) — that have some legislative, judicial, and administrative
autonomy within a state. The Sixth Schedule applies to the Northeastern states of Assam, Meghalaya,
Mizoram (three Councils each), and Tripura (one Council). Thus, the Sixth Schedule of the Constitution
supports the concept of decentralized administration. So, Statement 1 is correct.
According to Article 243G of the Constitution, State Legislatures may by law endow Panchayats with
such powers and authority as may be necessary to enable them to function as institutions of self-
government. In addition, such law may contain provisions for devolution of powers and responsibilities
upon Panchayats at appropriate level subject to such conditions as may be specified therein. The
Eleventh Schedule of the Constitution gives a list of 29 subjects in respect of which functions, finances
and functionaries may be devolved by State to Panchayati Raj Institutions. The authority relating to the
transfer of powers and funds to Panchayati Raj Institutions vests with the State Governments. Thus, the
Eleventh Schedule of the Constitution supports the concept of decentralized administration. So,
Statement 2 is correct.

63
The twelfth Schedule specifies the powers, authority and responsibilities of Municipalities. It has 18
matters. This schedule was added by the 74th Amendment Act of 1992. Thus, the Twelfth Schedule of
the Constitution supports the concept of decentralized administration. So, Statement 3 is correct.

ADDITIONAL INFORMATION:
SCHEDULES OF THE INDIAN CONSTITUTION
First Schedule Names of the States, Union Territories and their territorial jurisdiction.
Second Provisions relating to the emoluments, allowances, privileges and so on of the
Schedule President, Governor, Speakers and Deputy Speakers of the Parliament and State
Legislature, Judges of the Supreme Court and High Court, The Comptroller and
Auditor-General of India
Third Forms of Oaths or Affirmations for the Union and State ministers, the candidates for
Schedule election to the Parliament and State Legislature, the members of the Parliament and
State Legislature, the judges of the Supreme Court and the High Court, the
Comptroller and Auditor-General of India.
Fourth Allocation of seats in the Rajya Sabha to the states and the union territories.
Schedule
Seventh Division of powers between the Union and the States in terms of List I (Union List),
Schedule List II (State List) and List III (Concurrent List).
Eighth Languages recognized by the Constitution. Originally, it had 14 languages, but
Schedule presently, there are 22 languages.
Ninth ➢ Acts and Regulations (originally 13 but presently 282) of the state legislatures
Schedule dealing with land reforms and abolition of the zamindari system and the
Parliament dealing with other matters.
➢ This schedule was added by the 1st Amendment (1951) to protect the laws included
in it from judicial scrutiny on the grounds of violation of fundamental rights.
➢ However, in 2007, the Supreme Court ruled that the laws included in this schedule
after April 24, 1973, are now open to judicial review.
Tenth ➢ Provisions relating to disqualification of the members of Parliament and State
Schedule Legislatures on the ground of defection.
➢ This schedule was added by the 52nd Amendment Act of 1985, also known as Anti-
defection Law.

58. Consider the following statements with respect to the Chargesheet:


1. Chargesheets are filed by law enforcement agencies after a person is convicted by a competent
court.
2. Chargesheets are defined under the Criminal Code of Procedure.
3. Chargesheets can be made publicly available as it is a public document under the Indian Evidence
Act.
How many of the statements given above are correct?
(a) Only one
(b) Only two
(c) All three
(d) None
EXPLANATION:

64
A chargesheet is filed by the police or law-enforcement/ investigative agency only after they have gathered
sufficient evidence against the accused in respect of the offenses mentioned in the FIR; otherwise, a
‘cancellation report’ or ‘untraced report’ can be filed when due to lack of evidence. After preparing the
chargesheet, the officer-in-charge of the police station forwards it to a Magistrate, who is empowered to
take notice of the offenses mentioned in it. The term ‘chargesheet’ has been expressly defined under
Section 173 of the CrPC (Criminal Code of Procedure).
Thus, the Chargesheet is the final report filed toward the end of an investigation. They are not filed after
a competent court convicts a person.
So, Statement 1 is not correct, and Statement 2 is correct.
According to the Supreme Court of India, a chargesheet cannot be made publicly available as it’s not a
‘public document’ under Sections 74 and 76 of the Evidence Act.
Documents mentioned in Section 74 of the Evidence Act can only be said to be public documents,
certified copies of which are to be given by the concerned public authority having the custody of such a
public document. Copy of chargesheets along with necessary public documents cannot be said to be
‘public documents’ under Section 74 of the Evidence Act.” So, Statement 3 is not correct.

ADDITIONAL INFORMATION:
CHARGESHEET
Recently in Recently, The Supreme Court held that chargesheets are not ‘public documents’ and
News enabling their free public access violates the provisions of the Criminal Code of
Procedure as it compromises the rights of the accused, victim, and the investigation
agencies.
About ➢ A chargesheet must be filed against the accused within a prescribed period of 60-
90 days. Otherwise, the arrest is illegal, and the accused is entitled to bail.
➢ The charge sheet should contain details of names, the nature of the information,
and offenses. Whether the accused is under arrest, in custody, or has been released,
whether any action was taken against him are all important questions that the
chargesheet answers.
Chargesheet ➢ The term ‘chargesheet’ has been expressly defined under Section 173 of the CrPC,
vs FIR but ‘First Information Report’ or FIR has not been defined in either the Indian Penal
Code (IPC) or the CrPC. Instead, it finds a place under the police regulations/ rules
under Section 154 of CrPC, which deals with ‘Information in Cognizable Cases’.
➢ While the chargesheet is the final report filed towards the end of an investigation,
an FIR is filed at the ‘first’ instance’ that the police is informed of a cognizable offense
or offence for which one can be arrested without a warrant; such as rape, murder,
kidnapping.
➢ Further, an FIR does not decide a person’s guilt but a chargesheet is complete with
evidence and is often used during the trial to prove the offenses the accused is
charged with.
➢ After filing an FIR, the investigation takes place. Only if the police have sufficient
evidence can the case be forwarded to the Magistrate, otherwise, the accused is
released from custody under Section 169 of the CrPC.
59. Consider the following :
1. Central Consumer Protection Authority
2. Central Consumer Protection Council
3. National Consumer Disputes Redressal Commission
How many of the following are established through the Consumer Protection Act of 2019?
65
(a) Only one
(b) Only two
(c) All three
(d) None
EXPLANATION:
The Consumer Protection Act, 2019, has come into force from 20th July, 2020. As provided in section
10 of the Act, the Central Consumer Protection Authority (CCPA) has been established.
The objective of the Central Consumer Protection Authority (CCPA) is to promote, protect and enforce
the rights of consumers as a class. It will be empowered to conduct investigations into violations of
consumer rights and institute complaints or prosecution, order recalls of unsafe goods and services,
order discontinuation of unfair trade practices and misleading advertisements, and impose penalties on
manufacturers, endorsers and publishers of misleading advertisements. So, Statement 1 is correct.
The Central Consumer Protection Council is established through the Consumer Protection Act of 2019.
The objective of the Council shall be to render advice on the promotion and protection of the consumer's
rights under this Act. So, Statement 2 is correct.
The National Consumer Disputes Redressal Commission (NCDRC) is established through the Consumer
Protection Act of 2019. It is a quasi-judicial commission to provide inexpensive, speedy and summary
redressal of consumer disputes at the national level. Its head office is in New Delhi. Any person aggrieved
by an order of NCDRC may prefer an Appeal against such order to the Supreme Court of India within 30
days. So, Statement 3 is correct.
ADDITIONAL INFORMATION:
CONSUMER PROTECTION ACT OF 2019
About ➢ To strengthen the provisions for consumer protection, especially in the new era of
globalization, online platforms, e-commerce markets etc., the Consumer Protection
Act of 2019 was enacted to replace the Consumer Protection Act of 1986. Its inter-
alia provides for improved protection for consumers involved in online
transactions.
➢ The Consumer Protection Act 2019 has widened the scope of the definition of
"consumer" to include persons who buy or avail of goods or services online or
through electronic means, which was not present in the Consumer Protection Act
1986.
➢ The Act has also included definition of advertisement as any audio or visual
publicity, representation, endorsement or pronouncement made by means of,
inter-alia, electronic media, internet or website.
➢ The act explicitly includes e-commerce transactions within its scope, defining e-
commerce as the buying or selling of goods or services including digital products
over digital or electronic network.
Guarenteed Consumers are guaranteed the following six consumer rights under the under the
Consumer Consumer Protection Act of 2019. It includes,
Rights ➢ Right to Safety
➢ Right to be Informed
➢ Right to Choose
➢ Right to be heard
➢ Right to seek Redressal
➢ Right to Consumer Awareness

66
60. Which of the following has the authority to grant the Union Public Service Commission additional
powers with respect to the services of any local authority ?
(a) President
(b) Governor
(c) Parliament
(d) Union government
EXPLANATION:
Article 321 of the Indian Constitution provides the power to extend the functions of the Public Service
Commission. As per this Article, An Act made by Parliament or, as the case may be, the Legislature of a
State may provide for the exercise of additional functions by the Union Public Service Commission or the
State Public Service Commission with respect the services of the Union or the State and also as respects
the services of any local authority or other body corporate constituted by law or of any public institution.
Thus, the Parliament has the authority to grant the Union Public Service Commission additional powers
with respect to the services of any local authority. So, Option (c) is correct.
ADDITIONAL INFORMATION:
UNION PUBLIC SERVICE COMMISSION
About ➢ The Union Public Service Commission (UPSC) is the central recruiting agency in
India. It is an independent constitutional body in the sense that it has been directly
created by the Constitution
➢ The UPSC consists of a chairman and other members appointed by the president of
India.
➢ The Constitution, without specifying the strength of the Commission has left the
matter to the discretion of the president, who determines its composition.
➢ Usually, the Commission consists of nine to eleven members including the
chairman.
➢ Further, no qualifications are prescribed for the Commission’s membership except
that one-half of the members of the Commission should be such persons who have
held office for at least ten years either under the Government of India or under the
government of a state.
➢ The Constitution also authorises the president to determine the conditions of
service of the chairman and other members of the Commission
➢ The chairman and members of the Commission hold office for a term of six years or
until they attain the age of 65 years, whichever is earlier. However, they can
relinquish their offices at any time by addressing their resignation to the president.
➢ They can also be removed before the expiry of their term by the president in the
manner as provided in the Constitution.

61. The term "REJUPAVE," recently seen in the news, refers to:
(a) Newly discovered fig wasp species
(b) India's indigenous road construction technology
(c) India's unique and inexpensive space platform
(d) None of the above
EXPLANATION:
The indigenous technology, 'Rejupave,' a bio-oil-based product developed by India's oldest and premier
road research organization, CSIR-Central Road Research Institute (CSIR-CRRI), is beneficial in
constructing high-altitude bituminous roads at low and subzero temperature conditions.

67
This technology brings down the production and rolling temperature of bituminous mixes by 30 degrees
Celsius to 400 degrees Celsius, with negligible heat loss in the bituminous mix during transit.
The Border Roads Organisation (BRO) has successfully utilized an indigenous road construction
technology, "Rejupave," to build high-altitude bituminous road sections at the Sela tunnel and LGG-
Damteng-Yangste (LDY) road near the India-China border in Arunachal Pradesh at an altitude of 14,000
feet and 18,000 feet. The road built using the 'Rejupave' asphalt modifier in cold climatic regions will
have improved long-term durability and better resistance to thermal cracking under low-temperature
conditions. So, Option (b) is correct.
ADDITIONAL INFORMATION:
FIG WASP AND SPACE PLATFORM
New Fig Wasp ➢ Evidence of a new fig wasp (insect) has been found in the natural vegetation of East
Siang district of Arunachal Pradesh and named it Sycoscapter benjaminae.
➢ The new wasp species is named after its weeping fig host, benjamina, found in the
Pasighat area of Arunachal Pradesh.

India's unique, ➢ POEM-3, the PSLV Orbital Experimental Module-3, India's unique, inexpensive
inexpensive space platform using the spent PS4 stage of the PSLV-C58 vehicle that launched
space XPoSat on January 1, 2024, has successfully achieved all its objectives.
platform ➢ POEM-3 (the PSLV Orbital Experimental Module-3) uses the spent PS4 stage of the
PSLV-C58 rocket that launched XPoSat
➢ It is a three-axis-attitude controlled platform with power generation and
telecommand & telemetry capabilities for supporting Payloads.
➢ It is predicted that POEM-3 will continue orbiting for approximately 73 more days
before re-entering the Earth's atmosphere. With the likely re-entry of POEM-3 in
the next 75 days, the PSLV-C58 XPoSat mission will be leaving zero debris in space.
62. With reference to the National Commission for Women, consider the following statements:
1. The Chairperson of the Commission should be a retired Chief Justice of India or a judge of the
Supreme Court.
2. The Commission should have at least one member each belonging to the Scheduled Castes,
Scheduled Tribes and Other Backward Classes.
3. The Central Government can remove the Chairperson of the Commission if he/she is absent for
three consecutive meetings.
How many of the statements given above are correct?
(a) Only one
(b) Only two
(c) All three
(d) None
EXPLANATION:
The National Commission for Women (NCW) was constituted in 1992 to protect, promote and safeguard
the interests and rights of women.

68
The Commission is a multi-member body consisting of a chairperson, five members and a member-
secretary. The chairperson should be someone who is committed to the cause of women. However, there
is no specification for the appointment of a retired Chief Justice of India or a judge of the Supreme Court
as the chairperson of NCW. So, Statement 1 is not correct.
The five members to be appointed as members of NCW should be from amongst persons of ability,
integrity and standing who have had experience in law or legislation, trade unionism, management of an
industry or an organization committed to increasing the employment potential of women, women's
voluntary organizations (including women activists), administration, economic development, health,
education or social welfare. At least one member each should belong to the Scheduled Castes and
Scheduled Tribes, respectively.
However, there is no special mention of the appointment of an OBC person in the Commission as a
member of NCW. So, Statement 2 is not correct.
The Central Government can remove the chairperson or a member from office (before the expiry of
his/her term) under the following circumstances:
➢ If the individual becomes 'an undischarged insolvent
➢ If the individual gets convicted and sentenced to imprisonment for an offense which (in the opinion
of the Central Government) involves moral turpitude
➢ If the individual is declared of unsound mind by a competent court
➢ If the individual refuses to act or becomes incapable of acting
➢ If the individual absents himself/herself from three consecutive meetings of the Commission. So,
Statement 3 is correct.
➢ If the individual has abused one's official position, which (in the opinion of the Central Government)
renders one's continuance in the office detrimental to the public interest.

ADDITIONAL INFORMATION:
NATIONAL COMMISSION FOR WOMEN
About ➢ The National Commission for Women is an autonomous statutory (and not a
constitutional) body. It was established under the legislation enacted by the
Parliament, namely the National Commission for Women Act, 1990.
➢ The Ministry of Women and Child Development of the Government of India is the
nodal ministry for the Commission.
➢ The Commission has a wide mandate covering almost all aspects of women's
development and empowerment.
➢ The chairperson, members and member secretary of the National Commission for
Women are all nominated by the Central Government (Ministry of Women and
Child Development).
➢ Their salaries, allowances and other service conditions are also prescribed by the
Central Government.
➢ The chairperson and members hold office for three years. However, they can
relinquish their office at any time by addressing their resignation to the Central
Government.
➢ The Commission processes the complaints received verbally or in writing. It also
takes into account Suo mota notice of cases related to women.

63. What is the current position in India with regard to the recusal of Judges from hearing a case?
(a) Grounds for the recusal of Judges are well-defined in the Constitution of India.
(b) There is a well-defined law providing the grounds for the recusal of Judges.

69
(c) There are statutory rules governing the process of the recusal of Judges.
(d) There are no codified rules governing the process of recusal of Judges
EXPLANATION:
Judicial recusal means when a judge withdraws from a case due to a conflict of interest or a reasonable
apprehension of bias.
The Supreme Court has, over time, outlined various factors to be taken into consideration for deciding
the impartiality of a judge. In Ranjit Thakur v. Union of India (1987), the SC held that to determine if a
judge should recuse, what is relevant is the reasonableness of the apprehension of bias in the mind of
the concerned party. “The proper approach for the Judge is not to look at his own mind and ask himself,
however honestly, “Am I biased?” but to look at the mind of the party before him,” the Court ruled. But
Grounds for the recusal of Judges are not defined in the Constitution of India. So, Option (a) is not
correct.
There is no specific legislation in India to direct a judge’s recusal. Even though there are no specific laws
regarding the same, there has been customary practice. This is based on a probable existing bias, where
judges are expected to recuse. So, Option (b) is not correct.
There are no statutory rules governing the process, it is often left to the judges themselves to record
reasons for recusals. Some judges specify oral reasons in open court; others issue a written order
recording the reasons while in some cases the reasons are speculative. So, Option (c) is not correct.
If a judge recuses himself, the case is listed before the Chief Justice for allotment to an alternate Bench.
India has no codified rules governing recusals, although several Supreme Court judgments have dealt
with the issue. So, Option (d) is correct.

ADDITIONAL INFORMATION:
ATTORNEY GENERAL OF INDIA
About ➢ Judicial recusal is when a judge withdraws from a case due to a conflict of interest
or a reasonable apprehension of bias.
➢ This is done to ensure that the case is decided fairly and that the outcome is not
influenced by the personal biases of the judge.
➢ The practice of judicial recusal stems from the cardinal principle of due process of
law, which requires that all parties to a case be treated fairly and impartially.
➢ If a judge recuses themselves, the case is assigned to a different judge. Judicial
recusal is an important part of ensuring that the justice system is fair and
impartial.
Types ➢ Automatic recusal:
This occurs when a judge is required to recuse themselves by law. Example - A
judge must recuse themselves if they have a financial interest in the outcome of
the case.
➢ Discretionary recusal:
This occurs when a judge has the discretion to recuse themselves. Example - A
judge may recuse themselves if they have a close personal relationship with one of
the parties.

64. Consider the following statements:


Statement-I:
The incorporation of fundamental duties in the Constitution has not changed the status of the
fundamental rights guaranteed to the people of India.

70
Statement-II:
The rights and the duties of the citizens are correlative and inseparable.
Which of the following is correct in respect of the above statements?
(a) Both Statement-I and Statement-II are correct and Statement-II is the correct Explanation for
Statement-I.
(b) Both Statement-I and Statement-II are correct and Statement-II is not the correct
Explanation for Statement-I.
(c) Statement-I is correct but Statement-II is incorrect.
(d) Statement-I is incorrect but Statement-II is correct.
EXPLANATION:
The fundamental duties were incorporated in Part IV-A of the Constitution by the Constitution (42nd
Amendment) Act, 1976. Article 51(A) describes 11 fundamental duties in which
10 came with the 42nd Amendment; the 11th was added by the 86th Amendment in 2002.
As citizens, we must abide by the Constitution, defend our country, promote harmony among all citizens,
and protect the environment. However, it must be noted that our Constitution does not make the
enjoyment of rights dependent or conditional upon the fulfillment of duties. In this sense, the inclusion
of fundamental duties has not changed the status of our fundamental rights guaranteed to the people of
India. So, Statement 1 is correct.
The fundamental rights and duties of the citizens are correlative and inseparable, yet the original
constitution contained only the fundamental rights and not the fundamental duties. However, they
incorporated the duties of the State in the Constitution in the form of Directive Principles of State Polity.
Later, in 1976, the fundamental duties of citizens were added to the Constitution.
So, Statement 2 is correct.
In the Indian context, the incorporation of fundamental duties in the Constitution has not changed the
status of the fundamental rights guaranteed to the people of India. That is, fundamental duties are not
justiciable, while fundamental rights are justiciable.
Both Statement I and Statement II are correct, and Statement II is not the correct explanation
for Statement I.

ADDITIONAL INFORMATION:
FUNDAMENTAL DUTIES
About ➢ The Constitution of the erstwhile USSR inspires the Fundamental Duties in the
Indian Constitution.
➢ Notably, none of the Constitutions of major democratic countries like the USA,
Canada, France, Germany, Australia and so on specifically contain a list of duties
of citizens.
➢ Japanese Constitution is, perhaps, the only democratic Constitution in the world
that contains a list of duties of citizens.
➢ The socialist countries, on the contrary, gave equal importance to the fundamental
rights and duties of their citizens.
➢ In 1976, the Indian government set up the Sardar Swaran Singh Committee to
make recommendations about fundamental duties, the need and necessity of which
was felt during the operation of the internal emergency (1975–1977).
➢ The committee recommended the inclusion of a separate chapter on fundamental
duties in the Constitution.
71
➢ It stressed that the citizens should become conscious that in addition to the
enjoyment of rights, they also have certain duties to perform as well.
➢ The Central Government accepted these recommendations and enacted the 42nd
Constitutional Amendment Act in 1976.
➢ This amendment added a new part, namely, Part IVA to the Constitution. This new
part consists of only one Article, that is, Article 51A, which, for the first time,
specified a code of ten fundamental duties of the citizens.
➢ Though the Swaran Singh Committee suggested the incorporation of eight
Fundamental Duties in the Constitution, the 42nd Constitutional Amendment Act
(1976) included ten Fundamental Duties.
Recommenda The central government did not accept certain recommendations of the Committee and,
tions hence, not incorporated in the Constitution. These include:
rejected by ➢ The Parliament may provide for the imposition of such penalty or punishment as
the may be considered appropriate for any non-compliance with or refusal to observe
Government any of the duties.
➢ No law imposing such penalty or punishment shall be called into question in any
court on the ground of infringement of any of Fundamental Rights or on the ground
of repugnancy to any other provision of the Constitution.
➢ Duty to pay taxes should also be a Fundamental duty of the citizens.

65. Which of the following Indian cities are nominated for Wetland City Accreditation under the Ramsar
Convention?
(a) Indore, Bhopal and Udaipur
(b) Kollam, Bhubaneswar and Bhopal
(c) Kanpur, Kollam and Indore
(d) Vadodara, Udaipur and Bhubaneswar
EXPLANATION:
Ministry of Environment, Forest, and Climate Change has submitted three nominations from India for
Wetland City Accreditation (WCA) of Indore (Madhya Pradesh), Bhopal (Madhya Pradesh) & Udaipur
(Rajasthan) under the Ramsar Convention on Wetlands. These are the first three Indian cities for which
nominations have been submitted for WCA based on the proposals received from respective State
Wetlands Authorities in collaboration with the Municipal Corporations.
Sirpur Wetland (Ramsar site in Indore), Yashwant Sagar (Ramsar site closer to Indore), Bhoj Wetland
(Ramsar Site in Bhopal), and several Wetlands (Lakes) in and around Udaipur are lifelines to these cities.
So, Option (a) is correct.

ADDITIONAL INFORMATION:
WETLAND CITY ACCREDITATION (WCA)
Recently in India has submitted nominations of three cities - Indore, Bhopal and Udaipur - for
News Wetland City Accreditation (WCA) under the Ramsar Convention.
About ➢ Recognizing the importance of wetlands in urban and peri-urban environments and
taking appropriate measures to conserve and protect these wetlands, the Ramsar
Convention during COP12 held in the year 2015 approved a voluntary Wetland City
Accreditation system under Resolution XII.10, which recognizes cities that have
taken exceptional steps to safeguard their urban wetlands.

72
➢ The Wetland City Accreditation scheme aims to further promote the conservation
and wise use of urban and peri-urban wetlands, as well as sustainable socio-
economic benefits for local populations.
➢ To be formally accredited, a candidate for the Wetland City Accreditation should
satisfy the standards used to implement each of the six international criteria
mentioned in Operational Guidance for WCA of the Ramsar Convention on
Wetlands.
➢ This voluntary scheme provides an opportunity for cities that value their natural or
human-made wetlands to gain international recognition and positive branding
opportunities for their efforts in demonstrating strong positive relationships with
wetlands.
➢ The on-going AmritDharohar initiative of the MoEF&CC announced as part of the
2023-24 budget also aims to achieve similar goals by promoting unique
conservation values of Ramsar Sites.

66. With reference to the Constitution of India, consider the following statements:
1. The power to review its own judgments is the exclusive power of the Supreme Court.
2. There is no specific time limit within which a review petition must be filed after the pronouncement
of the judgment.
3. The review petition should be heard by the same bench of judges that delivered the original
judgment.
Which of the above statements is/are correct?
(a) 2 only
(b) 2 and 3 only
(c) 1, 2 and 3
(d) 3 only
EXPLANATION:
The Supreme Court has the power to review its judgment or order. Thus, it is not bound by its previous
decision and can depart from it in the interest of justice or community welfare. In brief, the Supreme
Court is a self-correcting agency. However, the power to review its judgments is not the exclusive power
of the Supreme Court. The high court also has the power of judicial review.
Judicial review is the power of a high court to examine the constitutionality of legislative enactments
and executive orders of both the Central and state governments. On examination, if they are found to be
violative of the Constitution (ultra-vires), they can be declared illegal, unconstitutional and invalid (null
and void) by the high court. So, Statement 1 is not correct.
As per the Rules of the Supreme Court, a review petition must be filed within 30 days from the date of
the impugned judgment or order. While a judgment is the final decision in a case, an order is an interim
ruling that is subject to its final verdict. However, in certain circumstances, the court can condone the
delay in filing a review petition if the petitioner can establish compelling reasons to justify it. So,
Statement 2 is not correct.
As far as practicable, the review petition must be presented before the same bench that delivered the
original decision. In case a judge has retired or is unavailable, a replacement is made by the Chief Justice
in the exercise of his administrative powers, keeping in mind the seniority of the judges in the top court.
So, Statement 3 is correct.

ADDITIONAL INFORMATION:
CURATIVE PETITION

73
About ➢ Since the Supreme Court is the ‘court of last resort,’ its judgments cannot result in
a miscarriage of justice.
➢ Accordingly, in Roopa Hurra v. Ashok Hurra (2002), the court itself evolved the
concept of a curative petition, which can be heard after a review is dismissed to
prevent abuse of its process.
➢ Like a review petition, a curative petition is also entertained on very narrow grounds
and is generally not allowed an open court hearing.
➢ Such petitions are heard by the five senior-most judges of the Supreme Court only
on the grounds of violation of principles of natural justice, question of bias against
the presiding judge, and abuse of the court process.
➢ This jurisdiction is exercised sparingly only in the rarest of circumstances.
➢ Some notable interventions in this regard include - National Commission for Women
v. Bhaskar Lal Sharma (2013), Navneet Kaur v. NCT Delhi (2014), Yakub Menon v.
State of Maharashtra (2015), and Union Carbide v. Union of India (2023).

67. Which one of the following best describes the concept of 'Judicial Dynamism' prevailing in India?
(a) The dedicated role played by the judiciary in molding the laws to meet the needs of
contemporary society
(b) The power of the judiciary to examine and interpret laws in the light of constitutional parameters.
(c) The self-discipline exercised by the judiciary to limit their interference with the functions of the
legislature or the executive.
(d) None of the above
EXPLANATION:
Judicial activism is also known as “judicial dynamism”. It is the antithesis of “judicial restraint”, which
means the self-control exercised by the judiciary. “Judicial activism is a way of exercising judicial power
that motivates judges to depart from normally practised strict adherence to judicial precedent in favour
of progressive and new social policies. It is commonly marked by decision calling for social engineering,
and occasionally these decisions represent intrusion in the legislative and executive matters. Thus,
Judicial Dynamism is the dedicated role played by the judiciary in moulding the laws to meet the needs
of contemporary society. So, Option (a) is correct.
Judicial review is indeed also a part of the basic features of the Constitution. Entrustment to the judiciary
of the power to test the validity of law is an established constitutional principle which co-exists with the
separation of powers. So, Option (b) is not correct.
Judicial restraint is a theory that encourages judges to limit the exercise of their own power. Judicial
overreach refers to the act of the judiciary interfering with the proper functioning of the legislative and
executive organs of the government. So, Option (c) is not correct.

ADDITIONAL INFORMATION:
JUDICIAL REVIEW
About The concepts of judicial review and judicial activism are closely related to each other. But,
there is a difference between them. The following points bring out this difference:
➢ Since about the mid-20th century, a version of judicial review has acquired the nick-
name of judicial activism, especially in the USA. In India, the participants in the debate
mix up judicial activism with judicial review. The former is that form of latter in which
judges participate in law-making policies, i.e., not only they uphold or invalidate laws
in terms of constitutional provisions, but also exercise their policy preferences in doing
so.
74
➢ The concept of judicial activism is inherent in judicial review, which empowers the
court to uphold the constitution and declare the laws and action inconsistent with the
constitution as void. Judicial activism is necessary for ensuring proper discharge of
duties by other organs.
➢ The term “judicial activism” came into currency sometime in the twentieth century to
describe the act of judicial legislation i.e., judges making positive law. However, there
is no standard definition of the term “judicial activism”. As a whole it can be said that
judicial activism stresses the importance of judicial review and a powerful judiciary in
the protection and promotion of certain core rights.
➢ The expanded concept of locus standi in connection with PIL, by judicial interpretation
from time-to-time, has expanded the jurisdictional limits of the courts exercising
judicial review. This expanded role has been given the title of “judicial activism” by
those who are critical of this expanded role of the judiciary.
➢ Judicial activism, as regards constitutional cases, falls under the rubric of what is
commonly called judicial review, and at the broadest level, it is any occasion where a
court intervenes and strikes down a piece of duly enacted legislation.

68. Consider the following situations:


1. A Ministry resigns after its defeat in the State Assembly, and no other party is willing or able to
form a Ministry commanding a majority in the State Assembly.
2. When a State Government fails to carry out the directives of the Union Government under Articles
256 and 257 of the Constitution.
3. When Ministers in a State Government face massive allegations of corruption.
The president’s rule in a state is/are not imposed under which of the situation mentioned above?
(a) 3 only
(b) 1 and 2 only
(c) 1 and 3 only
(d) 2 and 3 only

EXPLANATION:
Article 355 imposes a duty on the Centre to ensure that the government of every state is carried on in
accordance with the provisions of the Constitution. It is this duty in the performance of which the Centre
takes over the government of a state under Article 356 in case of failure of constitutional machinery in
the state. This is popularly known as ‘President’s Rule’. It is also known as ‘State Emergency’ or
‘Constitutional Emergency.’
Based on the report of the Sarkaria Commission on Centre-state Relations (1988), the Supreme Court
in the Bommai case (1994) enlisted the situations where the exercise of power under Article 356 could
be proper or improper.
The imposition of the President’s Rule in a state would be improper under the following situations:
➢ Where a ministry resigns or is dismissed on losing majority support in the assembly, and the governor
recommends imposition of President’s Rule without probing the possibility of forming an alternative
ministry.
➢ Where the governor makes his assessment of the support of a ministry in the assembly and
recommends imposition of President’s Rule without allowing the ministry to prove its majority on the
floor of the Assembly.

75
➢ Where the ruling party, enjoying majority support in the assembly, suffered a massive defeat in the
general elections to the Lok Sabha, such as in 1977 and 1980.
➢ Internal disturbances do not amount to internal subversion or physical breakdown.
➢ Maladministration in the state, allegations of corruption against the ministry or stringent financial
difficulties of the state. So, Statement 3 is correct.
➢ Where the state government is not given prior warning to rectify itself except in case of extreme
urgency leading to disastrous consequences.
➢ Where the power is used to sort out intraparty problems of the ruling party or for a purpose
extraneous or irrelevant to the one for which the Constitution has conferred it.
Imposition of President’s Rule in a state would be proper in the following situations:
➢ Where after general elections to the assembly, no party secures a majority, that is, ‘Hung Assembly.’
➢ Where the party having a majority in the assembly declines to form a ministry, and the governor
cannot find a coalition ministry commanding a majority in the assembly.
➢ Where a ministry resigns after its defeat in the assembly, and no other party is willing or able to form
a ministry commanding a majority in the assembly. So, Statement 1 is not correct.
➢ Where the state government disregards the constitutional direction of the Central government.
Therefore, when a State Government fails to carry out the directives of the Union Government under
Articles 256 and 257 of the Constitution. Articles 256 and 257 mention a wide range of subjects on
which the Union Government may give executive directions to State Governments. So, Statement 2
is not correct.
➢ Internal subversion is where, for example, a government is deliberately acting against the
Constitution and the law or is fomenting a violent revolt.
➢ Physical breakdown where the government wilfully refuses to discharge its constitutional obligations,
endangering the security of the state.

ADDITIONAL INFORMATION:
PRESIDENT’S RULE
About The President’s Rule can be proclaimed under Article 356 on two grounds–one
mentioned in Article 356 itself and another in Article 365:
➢ Article 356 empowers the President to proclaim if he is satisfied that a situation
has arisen in which the government of a state cannot be carried on in accordance
with the provisions of the Constitution. Notably, the president can act either on a
report of the governor of the state or otherwise too (ie, even without the governor’s
report).
➢ Article 365 says that whenever a state fails to comply with or to give effect to any
direction from the Centre, it will be lawful for the president to hold that a situation
has arisen in which the government of the state cannot be carried on in accordance
with the provisions of the Constitution.
Consequences The President acquires the following extraordinary powers when the President’s Rule
is imposed in a state:
➢ He can take up the functions of the state government and powers vested in the
governor or any other executive authority in the state.
➢ He can declare that the powers of the state legislature are to be exercised by the
Parliament.
➢ He can take all other necessary steps, including the suspension of the
constitutional provisions relating to anybody or authority in the state.

76
➢ Therefore, when the President’s Rule is imposed in a state, the President dismisses
the state council of ministers headed by the chief minister. The state governor, on
behalf of the President, carries on the state administration with the help of the
chief secretary of the state or the advisors appointed by the President.
➢ When the state legislature is thus suspended or dissolved:
➢ The Parliament can delegate the power to make laws for the state to the President
or any other authority specified by him in this regard
➢ The Parliament or, in case of delegation, the President or any other specified
authority can make laws conferring powers and imposing duties on the Centre or
its officers and authorities.
➢ The President can authorize, when the Lok Sabha is not in session, expenditure
from the state consolidated fund pending its sanction by the Parliament.
➢ The President can promulgate when the Parliament is not in session, ordinances
for the governance of the state.
➢ A law made by the Parliament or president or any other specified authority
continues to be operative even after the President’s Rule. This means that the
period for which such a law remains in force is not coterminous with the duration
of the proclamation. But it can be repealed, altered or re-enacted by the state
legislature.

69. Consider the following statements:


1. The Archaeological Survey of India (ASI) is responsible for maintaining ancient monuments in the
country, which includes religious sites.
2. Prayers are restricted at the ancient religious sites where there has been no continuity of worship
since they became 'protected sites.'
Which of the above statements is/are not correct?
(a) 1 only
(b) 2 only
(c) Both 1 and 2
(d) Neither 1 nor 2
EXPLANATION:
The Archaeological Survey of India (ASI), an attached office under the Department of Culture, Ministry
of Tourism and Culture, is the nodal agency of the Government of India for archaeological research and
protection of the cultural heritage of the nation.
The maintenance of ancient monuments, archaeological sites and remains of national importance is the
prime concern of the Archaeological Survey of India (ASI). Besides, it regulates all archaeological activities
in the country as per the provisions of the Ancient Monuments and Archaeological Sites and Remains
Act 1958 (as amended in 2010). It also regulates Antiquities and Art Treasure Act, 1972.
There are, at present, more than 3696 ancient monuments and archaeological sites and remains of
national importance. They include temples, mosques, tombs, churches, cemeteries, forts, palaces, step-
wells, rock-cut caves, and secular architecture, as well as ancient mounds and sites which represent the
remains of ancient habitation. Hence, the Archaeological Survey of India (ASI) is responsible for
maintaining ancient monuments, including religious sites in the country.
So, Statement 1 is correct.
Recently, A parliamentary panel has recommended the government to explore the possibility of
permitting puja and worship at monuments protected by the Archaeological Survey of India (ASI) that
have religious significance. According to Archaeological Survey of India, prayers are allowed at its

77
protected sites only if they were "functioning places of worship" at the time it took charge of them. "No
religious rituals can be conducted at non-living monuments where there has been no continuity of
worship when it became an ASI-protected site."
Therefore, Prayers are restricted at the ancient religious sites where there has been no continuity of
worship since they became 'protected sites.' So, Statement 2 is correct.

ADDITIONAL INFORMATION:
ARCHAEOLOGICAL SURVEY OF INDIA (ASI)
About ➢ The Archaeological Survey of India (ASI) is the premier organization for
archaeological research, scientific analysis, excavation of archaeological sites,
conservation and preservation of protected monuments and areas of national
importance, maintenance of site museums and overall regulation of legislations
related to antiquities and art treasures.
➢ Archaeological and historical pursuits in India started with the efforts of Sir William
Jones, who put together a group of antiquarians to form the Asiatic Society on 15th
January 1784 in Calcutta.
➢ For the maintenance of ancient monuments and archaeological sites and remains
of national importance, the entire country is divided into 37 Circles.
➢ The Circles look after the research on these monuments and conservation activities.
At the same time, the Science Branch, with its headquarters in New Delhi, carries
out chemical preservation, and the Horticulture Branch, under circle offices, is
entrusted with the laying out of gardens and environmental development.

70. With reference to Tribunals, consider the following statements:


1. The Administrative Tribunals Act of 1985 empowers the State Government to establish State
Administrative Tribunals.
2. A State Government may create a hierarchy of tribunals for the adjudication of disputes relating
to Rent and tenancy rights.
3. A tribunal established by the Central Government on matters other than public service matters
is not subject to the writ jurisdiction of the High Court.
How many of the statements given above are correct?
(a) Only one
(b) Only two
(c) All three
(d) None
EXPLANATION:
The Administrative Tribunals Act of 1985 empowers the Central government (not state government) to
establish the State Administrative Tribunals (SATs) on specific request of the concerned state
governments. So far (2019), the SATs have been set up in the nine states of Andhra Pradesh, Himachal
Pradesh, Odisha, Karnataka, Madhya Pradesh, Maharashtra, Tamil Nadu, West Bengal and Kerala.
However, the Madhya Pradesh, Tamil Nadu and Himachal Pradesh Tribunals have since been abolished.
So, Statement 1 is not correct.
Under Article 323 B, the Parliament and the state legislatures are authorised to provide for the
establishment of tribunals for the adjudication of disputes relating to the following matters:
➢ Taxation
➢ Foreign exchange, import and export
➢ Industrial and labour
78
➢ Land reforms
➢ Ceiling on urban property
➢ Elections to Parliament and state legislatures
➢ Food stuffs
➢ Rent and tenancy rights
Under Article323 A, only one tribunal for the Centre and one for each state or two or more states may
be established. There is no question of the hierarchy of tribunals, whereas under Article 323 B a
hierarchy of tribunals may be created. Thus, a State Government may create a hierarchy of tribunals
for the adjudication of disputes relating to Rent and tenancy rights. So, Statement 2 is correct.
In Chandra Kumar case4 (1997), the Supreme Court declared those provisions of these two articles
(Article 323 A and Article 323 B) which excluded the jurisdiction of the high courts and the Supreme
Court as unconstitutional. Hence, the judicial remedies are now available against the orders of these
tribunals. Thus, a tribunal established by the Central Government on matters other than public service
matters is subject to the writ jurisdiction of the High Court. So, Statement 3 is not correct.

ADDITIONAL INFORMATION:
CENTRAL ADMINISTRATIVE TRIBUNAL (CAT)
About ➢ The Central Administrative Tribunal (CAT) exercises original jurisdiction in
relation to recruitment and all service matters of public servants covered by it.
➢ Its jurisdiction extends to the all-India services, the Central civil services, civil
posts under the Centre and civilian employees of defence services.
➢ However, the members of the defence forces, officers and servants of the
Supreme Court and the secretarial staff of the Parliament are not covered by it.
➢ The CAT is a multi-member body consisting of a chairman and members.
Originally, the CAT consisted of a Chairman, ViceChairman and members. Later,
in 2006, the provision for the ViceChairman was removed by the Administrative
Tribunals (Amendment) Act, 2006. Hence, there are now no Vice-Chairman in
the CAT. At present (2019), the sanctioned strength of the Chairman is one and
sanctioned strength of the Members is 65.
➢ They are drawn from both judicial and administrative streams and are appointed
by the president.
➢ They hold office for a term of five years or until they attain the age of 65 years in
case of chairman and 62 years in case of members, whichever is earlier.
➢ The appointment of Members in CAT is made on the basis of recommendations
of a high powered selection committee chaired by a sitting Judge of Supreme
Court who is nominated by the Chief Justice of India.
➢ The CAT is not bound by the procedure laid down in the Civil Procedure Code of
1908. It is guided by the principles of natural justice.

71. Which of the following statements is not correct with reference to the National Commission for
Scheduled Tribes?
(a) The Commission submits an an annual report to the President of India when it thinks necessary.
(b) The Union and every State Government shall consult the Commission on all major policy matters
affecting Scheduled Tribes.
(c) The members of the Commission are eligible for reappointment only once.
(d) The Commission is required to discharge similar functions with regard to the Anglo-Indian
Community as it does with respect to the Scheduled Tribes

79
EXPLANATION:
According to Article 338 A(3)(d) of the Constitution, the National Commission for Scheduled Tribes
submit its report to the President annually and at such other times as the Commission may deem fit,
upon/ working of Safeguards, Measures required for effective implementation of Programmers/ Schemes
relating to Welfare and Socio-economic development of STs. So, Option (a) is correct.
In accordance with clause 9 of Article 338A of the Constitution, the Union and every State Government
shall consult the National Commission for Scheduled Tribes on all major policy matters affecting
Scheduled Tribes. So, Option (b) is correct.
The Chairperson, the Vice-Chairperson and other Member shall hold office for a term of three years from
the date on which he/she assumes such office. The Chairperson, the Vice-Chairperson and other
Members shall not be eligible for appointment for more than two terms. Thus, the members of the
Commission are eligible for reappointment only once. So, Option (c) is correct.
The National Commission for Scheduled Castes is an Indian constitutional body established with a view
to provide safeguards against the exploitation of Scheduled Castes and Anglo Indian communities to
promote and protect their social, educational, economic and cultural interests, special provisions were
made in the Constitution. Article 338 of the Indian constitution deals with National Commission for
Scheduled Castes. The National Commission for Scheduled Castes (Not the National Commission for
Scheduled Tribes) is also required to discharge similar functions with regard to the Anglo-Indian
Community as it does with respect to the SCs. So, Option (d) is not correct.

ADDITIONAL INFORMATION:
THE NATIONAL COMMISSION FOR SCHEDULED TRIBES
About The National Commission for Scheduled Tribes (NCST) was established by amending
Article 338 and inserting a new Article 338A in the Constitution through the
Constitution (89th Amendment) Act, 2003. By this amendment, the erstwhile National
Commission for Scheduled Castes and Scheduled Tribes was replaced by two separate
Commissions namely- (i) the National Commission for Scheduled Castes (NCSC), and
(ii) the National Commission for Scheduled Tribes (NCST).
Powers of the For Investigation and Inquiry, the Commission is vested with powers of a civil court
Commission having authority to:
➢ Summon and enforce attendance of any person and examine on oath;
➢ Discovery & production of any documents;
➢ Receive evidence on affidavits;
➢ Requisition any public record or copy thereof from any court or office;
➢ Issue Commissions for examination of witnesses and documents;
➢ Any matter which President, by rule, may determine.

72. Which among the following constituted the GST Appellate Tribunal as per the Central Goods and
Service Act of 2017?
(a) Supreme Court of India
(b) Parliament
(c) Union government
(d) GST Council
EXPLANATION:
The Union government, chaired by the Prime Minister, has approved the creation of the National Bench
of the Goods and Services Tax Appellate Tribunal (GSTAT). The National Bench of the Appellate Tribunal

80
shall be situated in New Delhi. GSTAT shall be presided over by its President and shall consist of one
Technical Member (Centre) and one Technical Member (State).
Goods and Services Tax Appellate Tribunal is the forum of the second appeal in GST laws and the first
common forum of dispute resolution between the Centre and States. Being a common forum, the GST
Appellate Tribunal will ensure that there is uniformity in the redressal of disputes arising under GST
and, therefore, in the implementation of GST across the country. So, Option (c) is correct.
ADDITIONAL INFORMATION:
GST Council
About ➢ The 101st Amendment Act of 2016 paved the way for the introduction of a new
tax regime (i.e., goods and services tax - GST) in the country.
➢ The smooth and efficient administration of this tax requires cooperation and
coordination between the center and the states.
➢ In order to facilitate this consultation process, the amendment provided for the
establishment of a Goods and Services Tax Council or the GST Council. The
Secretariat of the Council is located in New Delhi.
Composition of ➢ The GST Council is a joint forum of the center and the states and consists of the
the Council following members:
• The Union Finance Minister, as the Chairperson
• The Union Minister of State in-charge of Revenue or Finance
• The Minister in charge of Finance or Taxation or any other Minister
nominated by each state government.
➢ The members of the Council from the states have to choose one amongst
themselves to be the Vice-Chairperson of the Council. They can also decide his
term.
➢ The Union Cabinet also decided to include the Chairperson of the Central Board
of Excise and Customs (CBEC) as a permanent invitee (non-voting) to all
proceedings of the Council.

73. With reference to the 'Justice clock,' sometimes seen in the news, consider the following statements:
1. It aims to raise awareness among the public about the fair election process in the country.
2. In addition to being installed in high courts, it is also deployed at every polling station where
elections are held.
Which of the above statements are correct?
(a) 1 only
(b) 2 only
(c) Both 1 and 2
(d) Neither 1 nor 2
EXPLANATION:
The Justice Clock is an electronic signage system which provides information about disposal by courts,
various schemes and services offered in the court complexes and giving status of various fields to the
public and other information through which the citizens can benefit.
The purpose of Justice Clock is to bring awareness to the public about the justice sector, advertise the
various schemes of the department, give the status of various fields to the public and make effective use
of the database created through the National Judicial Data Grid (NJDG) and to make the information
available to public, LED Display Message Sign Board System have been installed.
The schemes of the Department of Justice and how citizens can benefit from the projects such as
eCourts, Access to Justice and Legal Aid to Poor are displayed on the Justice Clock. Information
81
regarding top district courts, which disposed highest percentage of cases for 2 years, 2-5 years and above
10 years old court cases and other information through which the citizens can benefit, such as legal aid
programs and access to justice schemes, is displayed on the Justice Clock.
A total of 39 Justice Clocks are functional in 25 High Courts. An amount of Rs. 13 lakhs has been
sanctioned by the e-committee for each Justice Clock. Thus, Justice clocks have been installed in High
courts only and are not to be deployed at every polling station where elections are held.
Therefore, Justice clock is not related to the election matters rather is displays the details about court
cases and various schemes. So, Statements 1 and 2 are not correct.

74. Consider the following statements with respect to the Central Information Commission (CIC):
1. It shall address the public authority to reject an information request to secure compliance with
the provisions of the Right to Information Act.
2. It has the power of a Civil Court while issuing summons for examining the witness or a document.
3. It shall seek an annual report from a public authority.
How many of the statements given above are correct?
(a) Only one
(b) Only two
(c) All three
(d) None
EXPLANATION:
Under the provision of Section 12 of the Right to Information Act (RTI Act) 2005, the Central Government
shall, by notification in the Official Gazette, constitute a body to be known as the Central Information
Commission.
The Central Information Commission has the power to secure compliance with its decisions from the
public authority. This includes:
➢ Providing access to information in a particular form;
➢ Rejecting an information request to secure compliance with the provisions of the Right to Information
Act. So, Statement 1 is correct.
➢ Enhancing training provision for officials on the right to information;
➢ Seeking an annual report from the public authority on compliance with this Act. So, Statement 3
is correct.
While inquiring, the Central Information Commission has the powers of a civil court in respect of the
following matters:
➢ Summoning and enforcing attendance of persons and compelling them to give oral or written evidence
on oath and to produce documents or things;
➢ Requiring the discovery and inspection of documents;
➢ Receiving evidence on affidavit;

82
➢ Requisitioning any public record from any court or office;
➢ Issuing summons for examination of witnesses or documents; So, Statement 2 is correct.
➢ Any other matter which may be prescribed.

ADDITIONAL INFORMATION:
CENTRAL INFORMATION COMMISSION
About ➢ The Central Information Commission is not a constitutional body and falls under
the Ministry of Personnel.
➢ It is a high-powered independent body that, among other things, looks into the
complaints made to it and decides the appeals.
➢ It entertains complaints and appeals pertaining to offices, financial institutions,
public sector undertakings, etc., under the Central Government and the Union
Territories.
Composition of ➢ The Commission consists of a Chief Information Commissioner and not more
the Central than ten Information Commissioners.
Information ➢ The President appoints them on the recommendation of a committee consisting
Commission of the Prime Minister as Chairperson, the Leader of the Opposition in the Lok
Sabha and a Union Cabinet Minister nominated by the Prime Minister.
➢ The Chief Information Commissioner and an Information Commissioner shall
hold office for such term as prescribed by the Central Government or until they
attain the age of 65 years, whichever is earlier. They are not eligible for
reappointment.

75. Consider the following statements:


1. The Adjournment terminates only the session of the house, not the sitting.
2. The Prorogation terminates only sitting in the house, not a session.
3. The Adjournment sine die terminates both the sitting and session of the house.
How many statements given above are correct?
(a) Only one
(b) Only two
(c) All three
(d) None
EXPLANATION:
The Adjournment terminates only a sitting and not a session of the House. It is done by the presiding
officer of the House. It does not affect the bills or any other business pending before the House, and the
same can be resumed when the House meets again. So, Statement 1 is not correct.
Prorogation not only terminates a sitting but also a session of the House. The president of India does it.
It also does not affect the bills or any other business pending before the House. However, all pending
notices (other than those for introducing bills) lapse on prorogation, and fresh notices have to be given
for the next session. In Britain, prorogation brings to an end all bills or any other business pending
before the House. So, Statement 2 is not correct.
Adjournment sine die means terminating a sitting of Parliament for an indefinite period (session is not
terminated). In other words, when the House is adjourned without naming a day for reassembly, it is
called adjournment sine die. The power of adjournment sine die lies with the presiding officer of the
House. So, Statement 3 is not correct.

83
ADDITIONAL INFORMATION:
SUMMONING OF THE PARLIAMENT
Summoning ➢ The president, from time to time, summons each House of Parliament to meet.
➢ But, the maximum gap between two sessions of Parliament cannot be more than
six months. In other words, the Parliament should meet at least twice a year.
➢ There are usually three sessions in a year, viz,
• The Budget Session (February to May);
• The Monsoon Session (July to September); and
• The Winter Session (November to December).
➢ A ‘session’ of Parliament is the period spanning between the first sitting of a House
and its prorogation (or dissolution in the case of the Lok Sabha).
➢ During a session, the House meets everyday to transact business. The period
spanning between the prorogation of a House and its reassembly in a new session
is called ‘recess’.

76. Which of the following best describes the concept of ‘Secularism’ being followed in India?
(a) The state can give grants to religious communities when required but doesnot have the power to
restrict them.
(b) The state has the power to both restrict and give grants to religious communities as
required.
(c) The state can restrict the religious communities when required but has no power to give grants
to them.
(d) The state cannot give grants to religious communities and has no power to restrict them either.
EXPLANATION:
Secularism means the separation of religion from political, economic, social, and cultural aspects of life.
Thus, religion is only a purely personal matter. It provides full freedom to all religions and tolerance of
all religions.
The Indian Constitution mandates that the Indian State be secular. According to the Constitution, only
a secular State can realize its objectives to ensure the following:
➢ One religious community does not dominate another;
➢ Some members do not dominate other members of the same religious community;
➢ The State does not enforce any particular religion nor take away the religious freedom of individuals.
The Indian Constitution embodies the positive concept of secularism, i.e., giving equal respect to all
religions or protecting all religions equally. The state had to interfere in the affairs of religion. Such
intervention was not always negative. The state could also help religious communities by giving grants
to educational institutions run by them. Thus, the state may restrict religious communities depending
on which mode of action promotes values such as freedom and equality. In India, separation between
religion and state did not mean their mutual exclusion.
Thus, the Indian Constitution grants all religious minorities the right to establish and maintain their
own educational institutions, which may receive assistance from the state.
So, Option (b) is correct.

77. With reference to the 'Right to Clean and Healthy Environment,' often seen in the news, consider the
following statements:
1. It is recognized in the 1948 Universal Declaration of Human Rights as part of the right to an
adequate standard of living.
2. In India, it is a fundamental right guaranteed in Article 21 of the Constitution.
84
Which of the above statements are not correct?
(a) 1 only
(b) 2 only
(c) Both 1 and 2
(d) Neither 1 nor 2
EXPLANATION:
The Universal Declaration of Human Rights was adopted in 1948 by the United Nations General
Assembly. Under Article 25 of this UDHR Everyone has the right to a standard of living adequate for the
health and well-being of himself and of his family, including food, clothing, housing and medical care
and necessary social services, and the right to security in the event of unemployment, sickness,
disability, widowhood, old age or other lack of livelihood in circumstances beyond his control.
In 2022, the UN General Assembly adopted a historic resolution, declaring access to a clean, healthy
and sustainable environment, a universal human right. The decision will help States accelerate the
implementation of their environmental and human rights obligations and commitments. Thus, Right to
Clean and Healthy Environment was not recognized in the 1948 Universal Declaration of Human Rights
as part of the right to an adequate standard of living. It was recognized recently. So, Statement 1 is not
correct.
The Right to a healthy environment is the core of the Right to life. It is in many ways connected to the
Right to clean drinking water and the right to health. It also prevents humans from damaging flora and
fauna. This right has been recognized and advocated by the United Nations and was accepted as a global
human right at Stockholm Conference also known as Magna Carta of human environment. It provides a
much-required basis for environmental protection to environmental activists. Also, this conference linked
environmental protection with sustainable development. The Right to a clean environment has been
adopted by the Indian Constitution under Article 21 as the fundamental right. So, Statement 2 is
correct.

78. Consider the following statements:


1. In India, the Right to Information (RTI) Act overrides the Official Secrets Act (OSA) which exists
since the colonial regime.
2. The request for information filed under the RTI Act cannot be denied on the basis that it has been
marked 'secret' under the OSA.
Which of the statements given above is/are correct?
(a) 1 only
(b) 2 only
(c) Both 1 and 2
(d) Neither 1 nor 2
EXPLANATION:
The Official Secrets Act, 1923 (OSA) commenced during colonial rule to ensure and protect the
confidentiality and secrecy related to the government's work, especially matters related to the safety and
security of a nation. On the contrary, the Right to Information Act (RTI) came into existence to know
about the administration of the government.
There is a huge contrast between these two Acts as the Official Secrets Act 1923 was brought in India
for the culture of secrecy and specific denial of any claim against the workings of the government, and
the RTI was established in India to bring transparency and accountability in the workings of the
authorities and Government of India.
RTI is a fundamental right guaranteed under Article 19(1) of the Constitution of India. Thus, the citizens
of the country have the right to have information regarding the operations of the government. The OSA

85
generally promotes secrecy and confidentiality in government operations, which is against the provisions
of the RTI. Sometimes, records that are generally kept aside for the purpose of secrecy are crucial in
bringing them to the public notice. In every instance, the OSA hampers the progressive democracy in
society. Further, it is also clear that provisions of the RTI override the OSA.So, Statement 1 is correct.
In order to ensure that other laws and constraints could not be used to deny information to the real
rulers of democracy-the citizens, legislature has enacted a Non-obstante clause in Section 22 of the RTI
Act, which provides, "the provisions of this Act shall have effect notwithstanding anything inconsistent
in addition to that contained in the Official Secrets Act, 1923, and any other law for the time being in
force or any instrument having effect by virtue of any law other than this Act".
It clearly means that the RTI Act will prevail over all laws and rules, including the Official Secrets Act,
as far as providing information under RTI are concerned. It does not mean that the Official Secrets Act
or other acts are repealed. When a request for information is filed under the RTI Act, it can be only denied
if the provisions of the RTI Act provide for an exemption. So, Statement 2 is correct.

79. Which of the following provisions with respect to the Speaker of Lok Sabha is not correct?
(a) A resolution to remove the Speaker is passed only in the House of the People.
(b) If the House of the People is dissolved due to the passing of a no-confidence motion, the
speaker should also vacate her office.
(c) Parliamentary privileges are extended to the Speaker of Lok Sabha but not to the Chairman of
Rajya Sabha.
(d) If the Speaker is absent for the joint sitting of the Parliament, such a person who is determined
by rules of procedure presides over the proceedings.
EXPLANATION:
The Presiding Officer of Lok Sabha is known as the Speaker. He is elected by the members of Lok Sabha
from amongst themselves.
Usually, the Speaker remains in office during the life of the Lok Sabha. However, he has to vacate his
office earlier in any of the following three cases:
➢ If he ceases to be a member of the Lok Sabha,
➢ If he resigns by writing to the Deputy Speaker;
➢ If he is removed by a resolution passed by a majority of all, then members of the Lok Sabha. Such a
resolution can be moved only after giving 14 days’ advance notice.
Thus, A resolution to remove the Speaker of Lok Sabha is passed only in the House of the People. So,
Option (a) is correct.
It should be noted here that whenever the Lok Sabha is dissolved due to the passing of a no-confidence
motion, the Speaker does not vacate his office and continues till the newly-elected Lok Sabha meets. So,
Option (b) is not correct.
Parliamentary privileges are special rights, immunities and exemptions enjoyed by the two Houses of
Parliament, their committees and their members. The Constitution has also extended parliamentary
privileges to those persons who are entitled to speak and take part in the proceedings of a House of
Parliament or any of its committees. These include the attorney general of India and Union ministers.
Thus, parliamentary privileges are extended to the Speaker of Lok Sabha.
The Chairman of Rajya Sabha is not a member of either House of Parliament, and the parliamentary
privileges are not extended to him. So, Option (c) is correct.
Joint sitting is an extraordinary machinery provided by the Constitution to resolve a deadlock between
the two Houses over the passage of a bill.
The Speaker of Lok Sabha presides over a joint sitting of the two Houses and the Deputy Speaker in his
absence. If the Deputy Speaker is also absent from a joint sitting, the Deputy Chairman of the Rajya Sabha
86
presides. If he is also absent, such other person as may be determined by the members present at the
joint sitting presides over the meeting. The Chairman of Rajya Sabha does not preside over a joint sitting
as he is not a member of either House of Parliament. The joint sitting is governed by the Rules of Procedure
of Lok Sabha and not of Rajya Sabha. So, Option (d) is correct.

ADDITIONAL INFORMATION:
SPEAKER OF LOK SABHA
About ➢ The Speaker is the head of the Lok Sabha and its representative.
➢ He is the guardian of the powers and privileges of the members, the House as a whole
and its committees.
➢ He is the principal spokesman of the House, and his decision in all Parliamentary
matters is final.
Powers and The powers and duties of the Speaker of Lok Sabha includes,
Duties of the ➢ He maintains order and decorum in the House for conducting its business and
Speaker regulating its proceedings. This is his primary responsibility and he has final power
in this regard.
➢ He is the final interpreter of the provisions of the Constitution of India, the Rules of
Procedure and Conduct of Business of Lok Sabha, and the parliamentary precedents,
within the House.
➢ He adjourns the House or suspends the meeting in absence of a quorum. The quorum
to constitute a meeting of the House is one-tenth of the total strength of the House.
➢ He does not vote in the first instance. But he can exercise a casting vote in the case
of a tie. In other words, only when the House is divided equally on any question, the
Speaker is entitled to vote. Such vote is called casting vote, and its purpose is to
resolve a deadlock.
➢ He can allow a ‘secret’ sitting of the House at the request of the Leader of the House.
When the House sits in secret, no stranger can be present in the chamber, lobby or
galleries except with the permission of the Speaker. He decides whether a bill is a
money bill or not, and his decision on this question is final.
➢ He decides the questions of disqualification of a member of the Lok Sabha arising on
the ground of defection under the provisions of the Tenth Schedule. In 1992, the
Supreme Court ruled that the decision of the Speaker in this regard is subject to
judicial review.

80. Consider the following provisions with respect to the 73rd Constitutional Amendment Act:
1. A member of a Panchayat not chosen by direct election shall have the right to vote in the meetings
of the Panchayats.
2. A State Legislature shall provide representation to the members of the Lok Sabha in the District
Panchayat if they are registered as electors within the respective Panchayat area.
Which of the statements given above are correct?
(a) 1 only
(b) 2 only
(c) Both 1 and 2
(d) Neither 1 nor 2

EXPLANATION:
According to Article 243(C) (3) of the Indian Constitution, The Legislature of a State may, by law, provide
for the representation,

87
➢ Of the Chairpersons of the Panchayats at the village level, in the Panchayats at the intermediate level
or, in the case of a State not having Panchayats at the intermediate level, in the Panchayats at the
district level.
➢ Of the Chairpersons of the Panchayats at the intermediate level, in the Panchayats at the district
level.
➢ Of the members of the House of the People and the members of the Legislative Assembly of the State
representing constituencies which comprise wholly or partly a Panchayat area at a level other than
the village level, in such Panchayat. So, Statement 2 is correct.
➢ Of the members of the Council of States and the members of the Legislative Council of the State,
where they are registered as electors within
• A Panchayat area at the intermediate level, in Panchayat at the intermediate level;
• A Panchayat area at the district level, in Panchayat at the district level.
➢ The Chairperson of a Panchayat and other members of a Panchayat whether or not chosen by direct
election from territorial constituencies in the Panchayat area shall have the right to vote in the
meetings of the Panchayats. So, Statement 1 is correct.
➢ The Chairperson of
• a panchayat at the village level shall be elected in such manner as the Legislature of a State may,
by law, provide; and
• A Panchayat at the intermediate level or district level shall be elected by, and from amongst, the
elected members thereof.

ADDITIONAL INFORMATION:
73rd CONSTITUTIONAL AMENDMENT ACT (1992)
Compulsory ➢ Organisation of Gram Sabha in a village or group of villages.
Provisions ➢ Establishment of panchayats at the village, intermediate and district levels.
➢ Direct elections to all seats in panchayats at the village, intermediate and district
levels.
➢ Indirect elections to the post of chairperson of panchayats at the intermediate
and district levels.
➢ Voting rights of the chairperson and other members of a panchayat elected
directly or indirectly.
➢ 21 years to be the minimum age for contesting elections to panchayats.
➢ Reservation of seats (both members and chairpersons) for SCs and STs in
panchayats at all three levels.
➢ Reservation of one-third of seats (both members and chairpersons) for women in
panchayats at all three levels.
➢ Fixing tenure of five years for panchayats at all levels and holding fresh elections
within six months in the event of supersession of any panchayat.
➢ Establishment of a State Election Commission for conducting elections to the
panchayats.
➢ Constitution of a State Finance Commission after every five years to review the
financial position of the panchayats.

81. With reference to the National Informatics Centre, consider the following statements:
1. It is an attached office of the Union Ministry of Science and Technology.
2. It aims to provide technology-driven solutions to both central and state governments.

88
Which of the above statements are not correct?
(a) 1 only
(b) 2 only
(c) Both 1 and 2
(d) Neither 1 nor 2
EXPLANATION:
National Informatics Centre is providing network backbone and e-Governance support to Central
Government, State Governments, UT Administrations, Districts and other Government bodies. It offers
a wide range of ICT services including Nationwide Communication Network for decentralised planning,
improvement in Government servics and wider transparency of national and local Governments.
NIC assists in implementing Information Technology Projects, in close collaboration with Central and
State Governments, in the areas of,
➢ Centrally sponsored schemes and Central sector schemes,
➢ State sector and State sponsored projects, and
➢ District Administration sponsored projects.
NIC endeavours to ensure that the latest technology in all areas of IT is available to its users. The National
Informatics Centre (NIC) is an attached office under the Ministry of Electronics and Information
Technology (MeitY) (not under the Ministry of Science and Technology). It is the technology partner of
the Government of India. So, Statement 1 is not correct.
New Delhi based National Informatics Centre (NIC) was established in 1976 with an objective to provide
technology-driven solutions to Central and State Governments in various aspects of development. NIC
has been instrumental in adopting and providing Information and Communication Technology (ICT) and
governance support to the Central Government. So, Statement 2 is correct.

ADDITIONAL INFORMATION:
NATIONAL INFORMATICS CENTRE (NIC)
About ➢ Its state-of-the-art IT infrastructure includes Multi-Gigabit PAN India Network
NICNET, National Knowledge Network, National Data Centres, National Cloud,
Video Conferencing, Email and Messaging Services, Command and Control Centre,
Multi-layered GIS based Platform, Domain Registration and Webcast.
➢ NIC has also developed several digital platforms for the socio-economic development
of the country with the ‘One-Nation One-Platform’ initiative to empower citizens
digitally.
➢ With an objective of focused study of new technology, exploring and experimenting
with their use in governance, NIC has set up a Centre of Excellence (CoE) in Data
Analytics, Artificial Intelligence, Blockchain and Application Security.
Mandate ➢ Technology partner of the Government
➢ Design and Develop IT Systems for the Government
➢ Provide ICT Infrastructure to the Government
➢ Explore & Advise on the use of Emerging Technologies

82. With reference to the PESA Act of 1996, consider the following statements:
1. The act is to cover the sixth scheduled tribal areas which were not covered in Part IX of the Indian
Constitution.
2. The prior recommendation of the gram sabha was mandatory for granting the exploitation of
minor minerals in the scheduled areas.

89
Which of the statements given above is/are correct?
(a) 1 only
(b) 2 only
(c) Both 1 and 2
(d) Neither 1 nor 2
EXPLANATION:
The provisions of Part IX of the constitution relating to the Panchayats are not applicable to the Fifth
Schedule areas. However, the Parliament may extend these provisions to such areas, subject to such
exceptions and modifications as it may specify. Under this provision, the Parliament enacted the
“Provisions of the Panchayats (Extension to the Scheduled Areas) Act,” 1996, popularly known as the
PESA Act or the Extension Act.
At present, ten states have Fifth Schedule Areas. These are Andhra Pradesh, Telangana, Chhatisgarh,
Gujarat, Himachal Pradesh, Jharkhand, Madhya Pradesh, Maharashtra, Odisha and Rajasthan. All the
ten states have enacted requisite compliance legislation by amending the respective Panchayati Raj Acts.
Thus, the PESA Act covers the fifth schedule areas (Not the Sixth Schedule areas), which were not
covered in Part IX of the Indian Constitution. So, Statement 1 is not correct.
According to the PESA Act, the prior recommendation of the Gram Sabha or the Panchayats at the
appropriate level shall be mandatory for the grant of concession for the exploitation of minor minerals
by auction in the Fifth Schedule areas. So, Statement 2 is correct.

ADDITIONAL INFORMATION:
PANCHAYATS (EXTENSION TO THE SCHEDULED AREAS) ACT 1996
Objectives The objectives of the Panchayats Extension To The Scheduled Areas (PESA) Act
are as follows:
➢ To extend the provisions of Part IX of the Constitution relating to the panchayats
to the scheduled areas with certain modifications.
➢ To provide self-rule for the bulk of the tribal population.
➢ To have village governance with participatory democracy and to make the gram
Sabha a nucleus of all activities.
➢ To evolve a suitable administrative framework consistent with traditional
practices.
➢ To safeguard and preserve the traditions and customs of tribal communities.
➢ To empower panchayats at the appropriate levels with specific powers conducive
to tribal requirements
➢ To prevent panchayats at the higher level from assuming the powers and
authority of panchayats at the lower level of the gram Sabha.
Important The features (or the provisions) of the PESA Act are as follows:
Features ➢ State legislation on the Panchayats in the Scheduled Areas shall be in
accordance with the customary law, social and religious practices and
traditional management practices of community resources.
➢ Every Gram Sabha shall be competent to safeguard and preserve the traditions
and customs of the people, their cultural identity, community resources and the
customary mode of dispute resolution.
➢ Every Gram Sabha shall (i) approve of the plans, programs and projects for social
and economic development before the Panchayat takes them up for
implementation at the village level and (ii) be responsible for the identification of
beneficiaries under poverty alleviation and other programs.
90
➢ Planning and management of minor water bodies in the Scheduled Areas shall
be entrusted to Panchayats at the appropriate level.

83. Consider the following statements with respect to the Electoral bonds:
1. It is a money instrument that can be bought by companies and individuals at any State Bank of
India branch.
2. There is no limit on the number of electoral bonds that a person or company can purchase.
3. The bonds go for sale in 10-day windows at the beginning of every quarter for four specified
months, besides an additional 30-day period during Lok Sabha election years.
How many of the statements given above are correct?
(a) Only one
(b) Only two
(c) All three
(d) None
EXPLANATION:
Electoral bonds are money instruments like promissory notes that can be bought by companies and
individuals in India from the State Bank of India (SBI) and donated to a political party, which can then
encash these bonds. The electoral bond does not bear the name of the donor and is, in effect, anonymous.
So, Statement 1 is correct.
Under the scheme, bonds are available for purchase at any SBI branch in multiples of ₹1,000, ₹10,000,
₹1 lakh, ₹10 lakh and ₹1 crore and can be bought through a KYC-compliant account. There is no limit
on the number of electoral bonds that a person or company can purchase. So, Statement 2 is correct.
The electoral bonds go for sale in 10-day windows at the beginning of every quarter, i.e., in January,
April, July, and October, besides an additional 30-day period specified by the Central Government during
Lok Sabha election years. So, Statement 3 is correct.
ADDITIONAL INFORMATION:
ELECTORAL BONDS
Recently in The government kicks off the New Year with fresh electoral bond sales on January 2.
News
About ➢ In 2018, the central government notified the Electoral Bond Scheme. This scheme
was announced in the 2017 budget.
➢ Electoral Bond is touted as an alternative to cash donations made to the political
parties.
➢ It was introduced to “cleanse the system of political funding in the country” by
eradicating the “menace of unaccounted money coming into the country’s
economy through political funding.”
➢ The government contended that it would make political donations transparent
while also protecting the identity of the donor.
➢ Every party registered under section 29A of the Representation of the Peoples Act,
1951 and having secured at least one percent of the votes polled in the most
recent Lok Sabha or State election has been allotted a verified account by the
Election Commission of India. The donor can donate the bond to a party of their
choice, which can cash it within 15 days, only through the allotted account.
➢ Electoral bonds are not the only way in which parties now receive donations. They
can also receive cash donations of less than ₹2,000 from anonymous sources
through cheque or by digital mode, in addition to electoral bonds.

91
84. Consider the following statements with respect to the Indian constitution :
1. Article 25 of the Indian constitution guarantees the freedom of conscience and the right to freely
profess, practice, and propagate religion.
2. Article 27 allows the state to provide financial aid to religious institutions for educational
purposes.
3. Article 26 grants every religious denomination the right to manage its own affairs in matters of
religion.
How many of the statements given above are correct?
(a) Only one
(b) Only two
(c) All three
(d) None
EXPLANATION:
Article 25 states that all persons are equally entitled to freedom of conscience and the right to freely
profess, practice and propagate religion. The implications of these are:
➢ Freedom of conscience: Inner freedom of an individual to mold his relation with God or Creatures in
whatever way he desires.
➢ Right to profess: Declaration of one’s religious beliefs and faith openly and freely.
➢ Right to practice: Performance of religious worship, rituals, ceremonies and exhibition of beliefs and
ideas.
➢ Right to propagate: Transmission and dissemination of one’s religious beliefs to others or exposition
of the tenets of one’s religion. But, it does not include a right to convert another person to one’s
religion. Forcible conversions impinge on the ‘freedom of conscience’ guaranteed to all persons alike.
From the above, it is clear that Article 25 covers not only religious beliefs (doctrines) but also religious
practices (rituals). Moreover, these rights are available to all persons–citizens as well as non-citizens. So,
Statement 1 is correct.
According to Article 27, No person shall be compelled to pay any taxes, the proceeds of which are
specifically appropriated in payment of expenses for the promotion or maintenance of any particular
religion or religious denomination.
It ensures that individuals are not forced to contribute through taxes towards the promotion or
maintenance of any specific religion or religious denomination. It upholds the principle of religious
neutrality and prevents the use of public funds for the advancement of a particular religious belief or
institution. Therefore, Article 27 does not allow the state to provide financial aid to religious institutions
for educational purposes. So, Statement 2 is not correct.
According to Article 26, every religious denomination or any of its sections shall have the following rights:
➢ Right to establish and maintain institutions for religious and charitable purposes.
➢ Right to manage its own affairs in matters of religion.
➢ Right to own and acquire movable and immovable property.
➢ Right to administer such property in accordance with law.
Article 25 guarantees the rights of individuals, while Article 26 guarantees the rights of religious
denominations or their sections. In other words, Article 26 protects collective freedom of religion. Like
the rights under Article 25, the rights under Article 26 are also subject to public order, morality and
health but not subject to other provisions relating to the Fundamental Rights. So, Statement 3 is
correct.

ADDITIONAL INFORMATION:
CULTURAL RIGHTS
92
Article 29 ➢ Article 29 provides that any section of the citizens residing in any part of India
having a distinct language, script or culture of its own shall have the right to
conserve the same.
➢ Further, no citizen shall be denied admission into any educational institution
maintained by the State or receiving aid out of State funds on grounds only of
religion, race, caste, or language.
➢ The first provision protects the right of a group while the second provision
guarantees the right of a citizen as an individual irrespective of the community to
which he belongs.
➢ Article 29 grants protection to both religious minorities as well as linguistic
minorities.
➢ However, the Supreme Court held that the scope of this article is not necessarily
restricted to minorities only, as it is commonly assumed to be. This is because of
the use of the words ‘section of citizens’ in the Article that includes minorities as
well as the majority.
➢ The Supreme Court also held that the right to conserve the language includes the
right to agitate for the protection of the language. Hence, the political speeches or
promises made for the conservation of the language of a section of the citizens do
not amount to corrupt practice under the Representation of the People Act, 1951.

85. With reference to the Central Empowered Committee (CEC), consider the following statements:
1. CEC is a permanent, statutory body set up by an act of Parliament.
2. CEC is constituted to oversee the cases related to environmental and forest issues.
3. The members of the committee are nominated by the Union Ministry of Environment, Forest and
Climate Change.
4. CEC submits its reports and recommendations to the National Green Tribunal.
Which of the above statements are correct?
(a) 1 and 4 only
(b) 2 only
(c) 2 and 3 only
(d) 2, 3 and 4 only
EXPLANATION:
The Central Empowered Committee (CEC) has been formed by an Order passed by the Hon’ble Supreme
Court of India in Writ Petition titled T.N. Godavarman Thirumalpad Vs Union of India and Others. The
Ministry of Environment, Forest and Climate Change had issued a notification under Section 3(3) of the
Environment (Protection) Act, 1986, constituting the CEC as a permanent body. Central Empowered
Committee (CEC) is the permanent committee for consultation on issues linked to environment and
forest cases, replacing the two-decade-old Central Empowered Committee.
The notification was issued after the apex court found that the old panel had members who were over
75 years of age and living abroad. Therefore, it is not the statutory body established under the act of the
Parliament. So, Statement 1 is not correct, and Statement 2 is correct.
Under the revamped structure, the Central Empowered Committee will comprise a chairperson, a
member secretary and three expert members; all are nominated and appointed by the Union Government
(MoEFCC). The Chairperson, with a minimum of 25 years of experience in environmental, forestry, or
wildlife fields or substantial administrative expertise in government, will serve a maximum term of three
years.

93
The member secretary must hold a rank not lower than deputy inspector general or director in the
government and possess at least 12 years of experience in environmental, forestry, or wildlife matters.
The three expert members, one each from the environment, forest, and wildlife sectors, should have a
minimum of 20 years of expertise. So, Statement 3 is correct.
The Central Empowered Committee (CEC) brought into place a much-needed Authority that could
critically examine decisions taken by the Ministry of Environment, Forest and Climate Change (MoEFCC)
and its various agencies and submit its independent report directly to the Supreme Court (not to the
National Green Tribunal) without worrying about the fallout from the MoEFCC.
So, Statement 4 is not correct.

86. which of the following statements is not correct with respect to the Parliamentary form of Government
in India?
(a) A nominated President is the executive head of the Union Government
(b) The President acts on the advice of his ministers, and whether he so acts is not questionable in
the courts
(c) Other than impeachment, there is no other mode to remove the President of India
(d) The Constitution of India contains no provision authorizing the President to act at his discretion
on any matter
EXPLANATION:
In India's Parliamentary system, the President is the nominal executive (de jure executive or titular
executive), while the Prime Minister is the real executive (de facto executive). Thus, the President is head
of the State, while the Prime Minister is head of the government.
Article 74 provides for a council of ministers headed by the Prime Minister to aid and advise the President
in the exercise of his functions. The advice so tendered is binding on the President. The President is
elected by members of an electoral college consisting of elected members of both Houses of Parliament
and Legislative Assemblies of the states in accordance with the system of proportional representation by
means of a single transferable vote.
Thus, in the Indian parliamentary system, the President is the executive head of the Union Government,
and he is elected. So, Option (a) is not correct.
The Indian presidency differs from most presidencies across the world. The president does not exercise
executive powers - he is the head of the state and is required by the Constitution to act on the advice of
ministers. The President shall not be answerable to any court for the exercise and performance of the
powers and duties of his office or any act done or purporting to be done by him in the exercise and
performance of those powers and duties.
Thus, the President acts on the advice of his ministers, and whether he so acts is not questionable in
the courts. So, Option (b) is correct.
The President holds office for a term of five years from the date on which he enters his office. However,
he can resign from his office at any time by addressing the resignation letter to the vice president.
Further, he can also be removed from office before the completion of his term by the process of
impeachment. Other than impeachment, there is no other mode to remove the President of India. So,
Option (c) is correct.
The President has no constitutional discretion, but he has some situational discretion. In other words,
the President can act at his discretion (that is, without the advice of the ministers) under the following
situations:
➢ Appointment of Prime Minister when no party has a clear majority in the Lok Sabha or when the
Prime Minister in office dies suddenly, and there is no obvious successor.
➢ Dismissal of the Council of Ministers when it cannot prove the confidence of the Lok Sabha.
94
➢ Dissolution of the Lok Sabha if the council of ministers has lost its majority.
So, Option (d) is correct.

ADDITIONAL INFORMATION:
PARLIAMENTARY FORM OF GOVERNMENT
About ➢ The Constitution of India has opted for the British Parliamentary System of
Government rather than the American Presidential System of Government.
➢ The parliamentary system is based on the principle of cooperation and
coordination between the legislative and executive organs. In contrast, the
presidential system is based on the doctrine of separation of powers between the
two organs.
➢ The parliamentary system is also known as the 'Westminster' 10 Model of
Government, responsible Government and Cabinet Government. The
Constitution establishes the parliamentary system not only at the Centre but
also in the states.
➢ The features of parliamentary government in India are:
• Presence of nominal and real executives;
• Majority party rule,
• Collective responsibility of the executive to the legislature,
• Membership of the ministers in the legislature,
• Leadership of the Prime Minister or the Chief Minister,
• Dissolution of the lower House (Lok Sabha or Assembly).
➢ Even though the Indian parliamentary system is largely based on the British
pattern, there are some fundamental differences between the two.
➢ For example, the Indian Parliament is not a sovereign body like the British
Parliament.
➢ Further, the Indian State has an elected head (republic), while the British State
has a hereditary head (monarchy).
➢ In a parliamentary system, whether in India or Britain, the role of the Prime
Minister has become so significant and crucial that political scientists like to call
it a 'Prime Ministerial Government.'

87. Consider the following statements:


1. A person who is not a citizen of India and holds any office of profit under the government of India
shall not accept any title from any foreign State.
2. A person holding office of profit under the local authority, with the consent of the President, shall
accept any emoluments from a foreign State.
Which of the statements given above are correct?
(a) 1 Only
(b) 2 Only
(c) Both 1 and 2
(d) Neither 1 nor 2
EXPLANATION:
Article 18 of the Indian Constitution abolishes titles and makes four provisions in that regard :
➢ No title, not being a military or academic distinction, shall be conferred by the State.
➢ No citizen of India shall accept any title from any foreign State.

95
➢ No person who is not a citizen of India shall, while he holds any office of profit or trust under the
State, accept without the consent of the President any title from any foreign State. So, Statement 1
is not correct.
➢ A person holding any office of profit or trust under the State shall, with the consent of the President,
accept any present, emolument, or office of any kind from or under any foreign State. So, Statement
2 is correct.
It is clear that the hereditary titles of nobility like Maharaja, Raj Bahadur, and Rai Bahadur, which
colonial States confers, are banned by Article 18 as these are against the principle of equal status of all.
ADDITIONAL INFORMATION:
CASE REGARDING THE ARTICLE 18 OF THE INDIAN CONSTITUTION
About ➢ In the Balaji Raghavan Case (1995), the Supreme Court upheld the constitutional
validity of the National Awards (Bharat Ratna, Padma Vibhusan, Padma Bhushan
and Padma Shri).
➢ It ruled that these awards do not amount to ‘titles’ within the meaning of Article 18
as the theory of equality does not mandate that merit should not be recognized.
➢ However, it also ruled that they should not be used as suffixes or prefixes to the
names of awardees. Otherwise, they should forfeit the awards.

88. A member holding office as Speaker or Deputy Speaker of a State Legislative Assembly shall be
removed by a resolution passed by which of the following majority?
(a) A majority of all the then members of the Assembly.
(b) A majority of not less than two-thirds of the total membership of the Assembly.
(c) A majority of not less than two-thirds of the members of that House present and voting.
(d) A majority of the total membership of that Assembly and by a majority of not less than two-thirds
of the members of the assembly present and voting.
EXPLANATION:
The Legislative assembly elects the Speaker and Deputy Speakers from amongst its members. Usually,
both the Speaker and deputy speaker remain in office during the life of the assembly. Article 179 of the
Indian Constitution deals with the vacation and resignation of and removal from the offices of Speaker
and Deputy Speaker. The Speaker and Deputy Speakers vacate their office earlier in any of the following
three cases:
➢ If they cease to be a member of the assembly
➢ If they resign by writing to the deputy speaker
➢ Suppose they are removed by a resolution passed by a majority of all the then members of the
assembly (i.e., an effective majority). Such a resolution can be moved only after giving 14 days
advance notice.
Thus, a member holding office as Speaker or Deputy Speaker of a State Assembly shall be removed by a
resolution passed by a majority of all the then present members of the Assembly. So, Option (a) is
correct.
ADDITIONAL INFORMATION:
PRESIDING OFFICERS OF STATE LEGISLATURE
About Each House of the state legislature has its presiding officer. There is a Speaker and
a Deputy Speaker for the legislative assembly and a Chairman and a Deputy
Chairman for the legislative council. A panel of chairman for the assembly and a
panel of vice-chairmen for the council is also appointed.
Speaker The Speaker has the following powers and duties:

96
➢ He maintains order and decorum in the assembly for conducting its business
and regulating its proceedings. This is his primary responsibility, and he has
final power in this regard.
➢ He is the final interpreter of the provisions of
• The Constitution of India,
• The rules of procedure and conduct of the business of the assembly,
• The legislative precedents within the assembly.
➢ He adjourns the assembly or suspends the meeting in the absence of a quorum.
➢ He does not vote in the first instance. But he can exercise a casting vote in the
case of a tie.
➢ He can allow a ‘secret’ sitting of the House at the request of the leader of the
House.
➢ He decides whether a bill is a Money Bill or not, and his decision on this question
is final.
➢ He decides the questions of disqualification of a member of the assembly arising
on the ground of defection under the provisions of the Tenth Schedule.
➢ He appoints the chairman of all the committees of the assembly and supervises
their functioning. He is the chairman of the Business Advisory Committee, the
Rules Committee and the General Purpose Committee.
Deputy Speaker ➢ Like the Speaker, the Deputy Speaker is also elected by the assembly itself from
amongst its members. He is elected after the election of the Speaker has taken
place.
➢ The Deputy Speaker performs the duties of the Speaker’s office when it is vacant.
➢ He also acts as the Speaker when the latter is absent from the sitting of the
assembly. In both cases, he has all the powers of the Speaker.

89. Consider the following statements with respect to the Leprosy:


1. Leprosy is a chronic infectious disease caused by a type of bacteria.
2. Leprosy is a curable disease through multi-drug therapy consisting of dapsone, rifampicin, and
clofazimine.
3. India has achieved the elimination of leprosy as a public health problem as per WHO criteria in
2005.
Which of the statements given above are correct?
(a) 1 and 2 only
(b) 2 and 3 only
(c) 1 and 3 only
(d) 1, 2 and 3
EXPLANATION:
Leprosy is an age-old disease and is described in the literature of ancient civilizations. Leprosy is a
chronic infectious disease caused by a type of bacteria, Mycobacterium leprae.
The disease predominantly affects the skin and peripheral nerves, mucosa of the upper respiratory tract,
and the eyes. The disease may cause progressive and permanent disabilities if left untreated. The
bacteria are transmitted via droplets from the nose and mouth during close and frequent contact with
untreated cases. So, Statement 1 is correct.
Leprosy is a curable disease. The currently recommended treatment regimen consists of three drugs:
dapsone, rifampicin and clofazimine. The combination is referred to as multi-drug therapy (MDT). MDT

97
kills the pathogen and cures the patient. Early diagnosis and prompt treatment can help to prevent
disabilities. So, Statement 2 is correct.
India has achieved the elimination of leprosy as a public health problem as per WHO criteria of less than
1 case per 10,000 population at the National level in 2005. However, there are few districts within States
where leprosy is still endemic. So, Statement 3 is correct.
ADDITIONAL INFORMATION:
NATIONAL LEPROSY ERADICATION PROGRAMME (NLEP)
About ➢ The Government is implementing the National Leprosy Eradication Programme
(NLEP) with the goal of making India leprosy-free.
➢ NLEP is a Centrally Sponsored Scheme under the umbrella of the National
Health Mission (NHM).
➢ This program is implemented in all the States and Union Territories.
➢ The major focus of the Programme is to detect cases of leprosy at an early stage
and to provide complete treatment, free of cost, in order to prevent the
occurrence of disability.
Initiatives Major initiatives under NLEP are as follows:
➢ Leprosy Case Detections Campaign (LCDC) in high endemic districts.
➢ Focused Leprosy Campaign (FLC) in low endemic districts for case detection.
➢ Special plans for Hard-to-Reach areas for early case detection and treatment.
➢ ASHA Based Surveillance for Leprosy Suspects (ABSULS).
➢ Sparsh Leprosy Awareness Campaign on 30th January every year.
➢ Active Case Detection and Surveillance both in rural and urban areas.
➢ Convergence of leprosy screening under Rashtriya Bal Swasthya Karyakram
(RBSK) for screening of children (0-18 years) and under Ayushman Bharat for
screening of people above 30 years of age.
➢ Monitoring of the program by the Joint Monitoring Investigation and Advisory
Group (JMIAG).
➢ Certificates, awards and performance-linked financial incentives are given to the
districts for achieving leprosy elimination.
➢ Contact tracing is done, and Post Exposure Prophylaxis (PEP) with a Single dose
of Rifampicin (SDR) is given to the eligible contacts of the index case in order to
interrupt the chain of transmission.

90. With reference to the Public and Private member bills, consider the following statement:
1. Any member of the parliament can introduce public bills in the House.
2. Rejection of private member bills can lead to a loss of parliamentary confidence in the house.
3. The responsibility for drafting both bills lies with the concerned department in consultation with
the law department.
How many statements given above are correct?
(a) Only one
(b) Only two
(c) All three
(d) None
EXPLANATION:
Bills introduced in the Parliament are of two kinds: public bills (government bills )and and private bills
(private members’ bills ).

98
A public bill is introduced in the Parliament by a minister (not by any member of the parliament). It
reflects the policies of the government (ruling party).
In contrast, private bills are introduced by any member of Parliament other than a minister. It reflects
the stand of the opposition party on public matters. So, Statement 1 is not correct.
A public bill has a greater chance of being approved by the Parliament. Rejection of public bills by the
House amounts to the expression of want of parliamentary confidence in the government and may lead
to its resignation.
However, the Rejection of private member bills has no implication on the parliamentary confidence in
the government or its resignation. So, Statement 2 is not correct.
A public bill is drafted by the concerned department in consultation with the law department. Whereas,
drafting of the Private bill is the responsibility of the member concerned. Thus, the responsibility for
drafting both bills is different. So, Statement 3 is not correct.

ADDITIONAL INFORMATION:
BILLS
About ➢ A bill is a proposal for legislation, and it becomes an act or law when duly enacted.
Every bill has to pass through the same stages in each House. The legislative
procedure is identical in both the Houses of Parliament.
➢ The bills introduced in the Parliament can also be classified into four categories apart
from public bills and private member bills:
• Ordinary bills which are concerned with any matter other than financial subjects.
• Money bills, which are concerned with financial matters like taxation, public
expenditure, etc.
• Financial bills, which are also concerned with financial matters (but are different
from money bills).
• Constitution amendment bills, which are concerned with the amendment of the
provisions of the Constitution.

91. Who/Which of the following is the final deciding authority when a member of the House of the People
is disqualified under the Representation of the People Act of 1951?
(a) The Speaker
(b) The President of India
(c) The Election Commission of India
(d) The Supreme Court of India
EXPLANATION:
The Parliament has laid down the following additional disqualifications in the Representation of People
Act (1951):
➢ He must not have been found guilty of certain election offenses or corrupt practices in the elections.
➢ He must not have been convicted for any offense resulting in imprisonment for two or more years.
But, the detention of a person under a preventive detention law is not a disqualification.
➢ He must not have failed to lodge an account of his election expenses within the time.
➢ He must not have any interest in government contracts, works or services.
➢ He must not be a director or managing agent nor hold an office of profit in a corporation in which
the government has at least 25 percent share.
➢ He must not have been dismissed from government service for corruption or disloyalty to the State.

99
➢ He must not have been convicted for promoting enmity between different groups or for the offense of
bribery.
➢ He must not have been punished for preaching and practicing social crimes such as untouchability,
dowry and sati.
As per Article 103 of the Constitution of India, if any question arises as to whether a member of either
House of Parliament has become subject to any of the disqualifications mentioned above, the question
shall be referred to the decision of the President, and his decision shall be final. Before giving any decision
on any such question, the President shall obtain the opinion of the Election Commission and shall act
according to such opinion. So, Option (b) is correct.

ADDITIONAL INFORMATION:
DISQUALIFICATION ON THE GROUND OF DEFECTION
About ➢ The Constitution also lays down that a person shall be disqualified from being a
member of Parliament if he is so disqualified on the ground of defection under
the provisions of the Tenth Schedule. A member incurs disqualification under
the defection law:
• If he voluntarily gives up the membership of the political party on whose
ticket he is elected to the House.
• If he votes or abstains from voting in the House contrary to any direction
given by his political party.
• If any independently elected member joins any political party.
• If any nominated member joins any political party after the expiry of six
months.
➢ The question of disqualification under the Tenth Schedule is decided by the
Chairman in the case of Rajya Sabha and the Speaker in the case of Lok Sabha
(and not by the president of India).
➢ In 1992, the Supreme Court ruled that the decision of the Chairman/ Speaker
in this regard is subject to judicial review.

92. Consider the following statements:


1. The Supreme Court of India had constituted the Cauvery Water Disputes Tribunal.
2. The Central Government established the Cauvery Water Management Authority (CWMA) by
introducing the Cauvery Water Management Scheme 2018.
3. The decisions of the CWMA shall be final and binding on all the party States.
How many of the statements given above are correct?
(a) Only one
(b) Only two
(c) All three
(d) None
EXPLANATION:
The Government of India (not the Supreme Court of India) constituted the Cauvery Water Disputes
Tribunal (CWDT) on June 2, 1990, to adjudicate the water dispute between the states of Tamil Nadu,
Karnataka, Kerala and Puducherry with respect to the inter-state Cauvery water and river basin. So,
Statement 1 is not correct.
In the exercise of the powers conferred by section 6A of the Inter-State River Water Disputes Act, 1956,
the Central Government notified the Cauvery Water Management Scheme on June 1, 2018, inter-alia,

100
constituting the ‘Cauvery Water Management Authority’ and the ‘Cauvery Water Regulation Committee’
to give effect to the decision of the Cauvery Water Disputes Tribunal.
The decisions of the Cauvery Water Management Authority shall be final and binding on all the party
States. So, Statements 2 and 3 are correct.

ADDITIONAL INFORMATION:
INTER-STATE WATER DISPUTES
About ➢ Article 262 of the Constitution provides for the adjudication of interstate water
disputes. It makes two provisions:
• Parliament may by law provide for the adjudication of any dispute or
complaint with respect to the use, distribution and control of waters of any
inter-state river and river valley.
• Parliament may also provide that neither the Supreme Court nor any other
court is to exercise jurisdiction in respect of any such dispute or complaint.
➢ Under this provision, the Parliament has enacted two laws, namely,
• The River Boards Act (1956)
• The Inter-State Water Disputes Act (1956)
The River ➢ The River Boards Act of 1956 provides for the establishment of river boards for
Boards Act the regulation and development of inter-state river and river valleys.
(1956) ➢ A river board is established by the Central government on the request of the
state governments concerned to advise them.
Inter-State ➢ The Inter-State Water Disputes Act of 1956 empowers the Central government
Water Disputes to set up an ad hoc tribunal for the adjudication of a dispute between two or
Act (1956) more states in relation to the waters of an inter-state river or river valley.
➢ The decision of the tribunal would be final and binding on the parties to the
dispute.
➢ Neither the Supreme Court nor any other court is to have jurisdiction in respect
of any water dispute which may be referred to such a tribunal under this Act.

93. The “Paris Principles” recently seen in the news refers to:
(a) A guideline that sets out the minimum standards required for national human rights
institutions to operate efficiently and be considered credible.
(b) Promote cooperation among nations by exchanging information on the impact of human activities
on the ozone layer.
(c) A Way to find a sustainable debt-relief solutions for countries that are unable to repay their
bilateral loans.
(d) Principle of international environmental law establishing that all states are responsible for
addressing global environmental destruction yet not equally responsible.
EXPLANATION:
The United Nations Paris Principles, adopted in 1993 by the U.N. General Assembly ("Principles Relating
to the Status of National Human Rights Institutions") set out the minimum standards required by
national human rights institutions to be considered credible and to operate effectively.
➢ Paris Principles are a crucial step in the development of standards for national human rights
institutions across the world.

101
➢ The six principles require a country's human rights agency to be independent of the government in
its structure, composition, decision-making and method of operation.

So, Option (a) is correct.


The 1985 Vienna Convention encouraged intergovernmental cooperation on research, systematic
observation of the ozone layer, monitoring of CFC production and the exchange of information. The
Convention aimed to promote cooperation among nations by exchanging information on the effects of
human activities on the ozone layer. So, Option (b) is not correct.
The Paris Club is a group of mostly Western creditor countries that grew from a 1956 meeting in which
Argentina agreed to meet its public creditors in Paris. Their objective is to find sustainable debt-relief
solutions for countries that are unable to repay their bilateral loans. So, Option (c) is not correct.
Common but differentiated responsibilities (CBDR), the principle of international environmental
law establishing that all states are responsible for addressing global environmental destruction yet not
equally responsible. The principle balances, on the one hand, the need for all states to take responsibility
for global environmental problems and, on the other hand, the need to recognize the wide differences in
levels of economic development between states. So, Option (d) is not correct.

ADDITIONAL INFORMATION:
PARIS PRINCIPLES
About ➢ It is officially known as 'Principles Relating to the Status of National Human
Rights Institutions.'
➢ The Paris Principles set out the minimum standards that NHRIs must meet in
order to be considered credible and to operate effectively.
➢ The key pillars of the Paris Principles are pluralism, independence and
effectiveness.
➢ Paris Principles was adopted in Paris in October 1991, and adopted by the
General Assembly of the United Nations in 1993.

102
94. With reference to the Official Language Committee, consider the following statements:
1. The Committee shall consist of twenty members from the House of the People and ten members
from the Council of States.
2. The Speaker and the Chairman shall nominate the members of the committee from the House of
the People and Council of States, respectively.
3. The duty of the Committee is to examine the recommendations of the Official Language
Commission and to report to the President of India.
How many statements given above are correct?
(a) Only one
(b) Only two
(c) All three
(d) None
EXPLANATION:
Article 344 deals with the Commission and Committee of Parliament on official language. It says that
the President shall, at the expiration of five years from the commencement of this Constitution and
thereafter at the expiration of ten years from such commencement, by order constitute a Commission
which shall consist of a Chairman and such other members representing the different languages
specified in the Eighth Schedule.
It shall be the duty of the Commission to make recommendations to the President as to,
➢ The progressive use of the Hindi language for the official purposes of the Union.
➢ Restrictions on the use of the English language for all or any of the official purposes of the Union.
➢ The language to be used for all or any of the purposes mentioned in article 348.
➢ The form of numerals to be used for any one or more specified purposes of the Union.
➢ Any other matter referred to the Commission by the President as regards the official language of the
Union and the language for communication between the Union and a State or between one State and
another and their use. So, Statement 1 is correct.
Under the Article 344(4), it provides that the Committee shall consist of 30 members of whom 20 shall
be from the Lok Sabha and 10 from the Rajya Sabha to be elected respectively by the members of the
Lok Sabha and the members of Rajya Sabha in accordance with the system of proportional
representation by means of a single transferable vote. Hence, The Speaker and the Chairman does not
nominate the members of the committee. So,Statement 2 is not correct.
According to the Article 344 (5) of the Indian Constitution, The Committee has to review the progress
made in the use of Hindi for the official purposes of the Union and submit a report to the President,
making recommendations thereon. The President shall cause the report to be laid before each House of
Parliament and sent to all the State Governments. So, Statement 3 is correct.

ADDITIONAL INFORMATION:
OFFICIAL LANGUAGES ACT AND ARTICLE 343
About Official Official Language Act of 1963 is an act to provide for the languages which may be
Language Act of used for the official purpose of the Union, for the transaction of business in
1963 Parliament, for Central and State Acts and for certain purposes in High Courts.
Section 3 of the act states Notwithstanding the expiration of the period of fifteen
years from the commencement of the Constitution, the English language may
continue to be used, in addition to Hindi,
➢ For all the official purposes of the Union for which it was being used immediately
before that day and
➢ For the transaction of business in Parliament.
103
After the expiration of ten years from the date on which section 3 comes into force,
there shall be constituted a Committee on Official Language on a resolution to that
effect be moved in either House of Parliament with the previous sanction of the
President and passed by both Houses.
Article 343 ➢ The official language of the Union shall be Hindi in Devnagari script. The form of
numerals to be used for the official purposes of the Union shall be the
international form of Indian numerals.
➢ Notwithstanding anything in clause (1), for fifteen years from the commencement
of this Constitution, the English language shall continue to be used for all the
official purposes of the Union for which it was being used immediately before
such commencement:
Provided that the President may, during the said period, by order, authorize the
use of the Hindi language in addition to the English language and of the
Devanagari form of numerals in addition to the international form of Indian
numerals for any of the official purposes of the Union.
➢ Notwithstanding anything in this article, Parliament may by law provide for the
use, after the said period of fifteen years, of-
• the English language, or
• The Devanagari form of numerals, for such purposes as may be specified in
the law.

95. With reference to the Comptroller and Auditor General (CAG) of India, consider the following
statements:
1. CAG shall not be eligible for further office either under the Government of India or under the
Government of any State after he ceases to hold office.
2. CAG acts as a guide, friend and philosopher of the Estimates Committee of the Parliament.
3. CAG audits and submits the account of union and states to the President, who shall place before
the both the houses of parliament.
How many of the statements given above are not correct?
(a) Only one
(b) Only two
(c) All three
(d) None
EXPLANATION:
The Indian Constitution (Article 148) provides for an independent office of the Comptroller and Auditor
General of India (CAG).
➢ He is the head of the Indian Audit and Accounts Department.
➢ He is the guardian of the public purse and controls the entire financial system of the country at
both the levels–the Centre and the state.
➢ He shall not be eligible for further office either under the Government of India or under the
Government of any State after he has ceased to hold office. So, Statement 1 is correct.
Public Accounts Committee will examine the appropriation accounts and the finance accounts of the
Union government and any other accounts laid before the Lok Sabha. The appropriation accounts
compare the actual expenditure with the expenditure sanctioned by the Parliament through the
Appropriation Act, while the finance accounts shows the annual receipts and disbursements of the Union
Government. For this purpose the Public Accounts Committee is assisted by the CAG. In fact, the CAG
acts as a guide, friend and philosopher of the committee. So, Statement 2 is not correct.

104
CAG submits his audit reports relating to the accounts of the Centre to President, who shall, in turn,
place them before both the Houses of Parliament (Article 151).
CAG submits his audit reports relating to the accounts of a state to Governor, who shall, in turn, place
them before the State Legislature (Article 151). So, Statement 3 is not correct.

ADDITIONAL INFORMATION:
COMPTROLLER AND AUDITOR GENERAL OF INDIA (CAG)
Duties and ➢ He audits the accounts related to all expenditures from the Consolidated Fund of
Functions of India, the consolidated fund of each state, and the consolidated fund of each union
the CAG territory having a Legislative Assembly.
➢ He audits all expenditure from the Contingency Fund of India and the Public
Account of India as well as the contingency fund of each state and the public
account of each state.
➢ He audits all trading, manufacturing, profit and loss accounts, balance sheets and
other subsidiary accounts kept by any department of the Central Government and
state governments.
➢ He audits the receipts and expenditures of the Centre and each state to satisfy
himself that the rules and procedures on that behalf are designed to secure an
effective check on the assessment, collection, and proper allocation of revenue.
➢ He audits the receipts and expenditure of the following: (a) All bodies and
authorities substantially financed from the Central or state revenues; (b)
Government companies; and (c) Other corporations and bodies, when so required
by related laws.
➢ He audits all transactions of the Central and state governments related to debt,
sinking funds, deposits, advances, suspense accounts and remittance business.
He also audits receipts, stock accounts and others, with approval of the President,
or when required by the President.
➢ He audits the accounts of any other authority when requested by the President or
Governor. For example, the audit of local bodies.
➢ He advises the President with regard to prescription of the form in which the
accounts of the Centre and the states shall be kept (Article 150).
➢ He ascertains and certifies the net proceeds of any tax or duty (Article 279). His
certificate is final. The ‘net proceeds’ means the proceeds of a tax or a duty minus
the cost of collection.
➢ He acts as a guide, friend and philosopher of the Public Accounts Committee of the
Parliament.
➢ He compiles and maintains the accounts of state governments. In 1976, he was
relieved of his responsibilities with regard to the compilation and maintenance of
accounts of the Central Government due to the separation of accounts from audit,
that is, departmentalisation of accounts.

96. With reference to the 'State' formation bill under Article 3 of the Constitution of India, consider the
following statements:
1. The bill can be introduced in either House of the Parliament with the recommendation of the
President.
2. The President will get the opinion of the respective state legislatures in this regard.
3. There is no fixed time frame within which the state legislature has to express its views on the bill.

105
Which of the above statements is/are not correct?
(a) 1 and 3 only
(b) 2 only
(c) 1, 2 and 3
(d) 3 only
EXPLANATION:
➢ Article 3 of the Indian Constitution authorizes the Parliament to:
• form a new state by separation of territory from any state or by uniting two or more states or parts
of states or by uniting any territory to a part of any state;
• increase the area of any state;
• diminish the area of any state;
• alter the boundaries of any state and
• alter the name of any state.
➢ Further, the power of Parliament to form new states includes the power to form a new state or union
territory by uniting a part of any state or union territory to any other state or union territory.
➢ However, Article 3 lays down two conditions in this regard: one, a bill contemplating the above
changes can be introduced in Parliament only with the prior recommendation of the President.
Therefore, the bill can be introduced in either House of the Parliament only with the recommendation
of the President. So, Statement 1 is correct.
➢ And two, before recommending the bill, the President has to refer the same to the state legislature
concerned for expressing its views within a specified period as stipulated by the President. Hence,
The President will get the opinion of the respective state legislatures within the specified time. So,
Statement 2 is correct and Statement 3 is not correct.
➢ Further, it is not necessary to make a fresh reference to the state legislature every time an
amendment to the bill is moved and accepted in Parliament. In case of a union territory, no reference
need be made to the concerned legislature to ascertain its views and the Parliament can itself take
any action as it deems fit.
➢ Originally, Article 3, did not prescribe a time limit for expression of views by the States on the States
reorganization laws. It was feared that the States could forestall the passage of the State
Reorganisation Act by not expressing their views for any length of time. By the The Constitution (Fifth
Amendment) Act, 1955, the Article now provides a time limit within which the State has to express
their views. If they do not express their views within the specified time the Bill may be passed by
Parliament.

ADDITIONAL INFORMATION:
ARTICLE 3
About ➢ The President (or Parliament) is not bound by the views of the state legislature and
may either accept or reject them, even if the views are received in time.
➢ Further, it is not necessary to make a fresh reference to the state legislature every
time an amendment to the bill is moved and accepted in Parliament.
➢ In case of a union territory, no reference need be made to the concerned legislature
to ascertain its views and the Parliament can itself take any action as it deems fit.
➢ The Constitution authorizes the Parliament to form new states or alter the areas,
boundaries or names of the existing states without their consent.
➢ In other words, the Parliament can redraw the political map of India according to
its will. Hence, the territorial integrity or continued existence of any state is not
guaranteed by the Constitution.

106
➢ Therefore, India is rightly described as 'an indestructible union of destructible
states.' The Union Government can destroy the states, whereas the state
governments cannot destroy the Union.
➢ In the USA, on the other hand, the territorial integrity or continued existence of a
state is guaranteed by the Constitution. The American Federal Government cannot
form new states or alter the borders of existing states without the consent of the
states concerned. That is why the USA is described as 'an indestructible union of
indestructible states.

97. With respect to the Anusandhan National Research Foundation (NRF) Act, 2023, consider the
following statements:
1. The Act provides for establishing the NRF and dissolves the Science and Engineering Research
Board (SERB) set up under the SERB Act of 2008.
2. The Governing Board of the NRF is headed by the Union Minister for Science and Technology.
3. In addition to the grants and loans from the Central Government, the NRF can also receive any
sum of money from philanthropic organizations, foundations, or international bodies for research
and development.
Which of the statements given above is/are correct ?
(a) 1 and 2 only
(b) 1 and 3 only
(c) 2 and 3 only
(d) 1, 2 and 3
EXPLANATION:
The Anusandhan National Research Foundation Act 2023 repeals the Science and Engineering Research
Board Act of 2008 and dissolves the Science and Engineering Research Board (SERB) set up under it. The
Act, in turn provides for establishing the Anusandhan National Research Foundation (NRF). The National
Research Foundation (NRF) stands as a cornerstone of research excellence, spearheading groundbreaking
initiatives that contribute to the advancement of knowledge and technology. It was established to support
and fund innovative projects across various disciplines. So, Statement 1 is correct.
National Research Foundation (NRF) will have a Governing Board headed by the Prime Minister of India
(Not the Union Minister of Science and Technology). The Board will provide strategic direction to the
Foundation and monitor the implementation. Other members of the Board are:
➢ The Union Ministers of Science and Technology, Education as Vice Presidents.
➢ The Principal Scientific Advisor as Member Secretary.
➢ Secretaries to the Departments of Science and Technology, Biotechnology, and Scientific and
Industrial Research. So, Statement 2 is not correct.
The National Research Foundation will be financed through grants and loans from the central government,
donations to the fund, income from investments of the amounts received by the Foundation, and all
amounts with the Fund for Science and Engineering Research set up under the 2008 Act.
Thus, in addition to the grants and loans from the Central Government, the NRF can also receive
contributions from philanthropic organizations, foundations, or international bodies for research and
development. So, Statement 3 is correct.

ADDITIONAL INFORMATION:
NATIONAL RESEARCH FOUNDATION (NRF)
About ➢ NRF will be the apex body in the country to provide strategic direction for
research, innovation, and entrepreneurship in the fields of:
• Natural sciences, including mathematics
• Engineering and technology
107
• Environmental and earth sciences
• Health and agriculture
• Scientific and technological interfaces of humanities and social sciences.
➢ The Principal Scientific Advisor will be the chairperson of the Executive Council.
Other members of the Council include:
• Secretaries to various departments of the central government, including Science
and Technology, Higher Education, Health Research, Agricultural Research, and
Defence Research.
• The Chief Executive Officer of the Foundation.
• The President of the Foundation may nominate or appoint to the Council up to
two secretaries of departments not covered under the Bill and up to three
experts.
Key Functions ➢ Preparing short-term, medium-term, and long-term roadmaps and formulating
programs for research and development (R&D).
➢ Facilitating and financing the growth of R&D and related infrastructure in
universities, colleges, and research institutions.
➢ Providing grants for research proposals
➢ Supporting translation of research into capital-intensive technology.
➢ Encouraging international collaboration
➢ Encouraging investments in the Foundation by private and public sector entities.
➢ Undertaking annual surveys of scientific research, outcomes, and spending.

98. The safeguards provided in Article 22 of the Indian Constitution to persons arrested or detained under
a law other than a preventive detention law applicable in case of :
(a) Civil arrest only
(b) Civil arrest or legal custody in jail
(c) Arrest for committing an act of a criminal nature
(d) Arrest on failure to pay income tax
EXPLANATION:
Article 22 grants protection to persons who are arrested or detained. Detention is of two types, namely,
punitive and preventive.
Article 22 has two parts: the first part deals with the cases of ordinary law and the second part deals
with the cases of preventive detention law. The first part of Article 22 confers the following rights on a
person who is arrested or detained under ordinary law:
➢ Right to be informed of the grounds of arrest.
➢ Right to consult and be defended by a legal practitioner.
➢ Right to be produced before a magistrate within 24 hours, including the journey time.
➢ Right to be released after 24 hours unless the magistrate authorizes further detention.
These safeguards are not available to an enemy alien or a person arrested or detained under a preventive
detention law. The Supreme Court also ruled that the arrest and detention in the first part of Article 22
do not cover arrest under the orders of a court, civil arrest, arrest on failure to pay the income tax, and
deportation of an alien.
They apply only to an act of a criminal or quasi-criminal nature or some activity prejudicial to the public
interest.
So, Option (c) is correct.

108
ADDITIONAL INFORMATION:
PUNITIVE AND PREVENTIVE DETENTION
About ➢ Detention is of two types, namely, punitive and preventive.
➢ Punitive detention is to punish a person for an offence committed by him after trial
and conviction in a court.
➢ Preventive detention, on the other hand, means the detention of a person without
trial and conviction by a court. Its purpose is not to punish a person for a past offense
but to prevent him from committing an offense in the near future. Thus, preventive
detention is only a precautionary measure based on suspicion.
The second The second part of Article 22 grants protection to persons who are arrested or detained
part of under a preventive detention law. This protection is available to both citizens as well as
Article 22 aliens and includes the following:
➢ The detention of a person cannot exceed three months unless the advisory board
reports sufficient cause for extended detention. The board is to consist of judges of
a high court.
➢ The grounds of detention should be communicated to the Detenu. However, the facts
considered to be against the public interest need not be disclosed.
➢ The detenu should be afforded an opportunity to make representation against the
detention order.
99. Consider the following statements with respect to the Drugs and Clinical Trial (Amendment) Rules,
2023:
1. It authorizes researchers to use non-animal and human-relevant methods to test the safety and
efficacy of new drugs.
2. The technology can be used for clinical trials such as organoids, Organ-on-a-chip and 3D
Bioprinter.
Which of the statements given above are correct?
(a) 1 only
(b) 2 only
(c) Both 1 and 2
(d) Neither 1 nor 2
EXPLANATION:
An amendment to the New Drugs and Clinical Trial Rules (2023), recently passed by the Government of
India, aims to replace the use of animals in research, especially in drug testing. The amendment
authorizes researchers to instead use non-animal and human-relevant methods to test the safety and
efficacy of new drugs. So, Statement 1 is correct.
In the last few decades, several technologies have been used for clinical trials, such as organoids, Organ-
on-a-chip, and 3D Bioprinter have been developed.
➢ Millimeter-sized three-dimensional cellular structures that mimic specific organs of the body are
called “organoids” or “mini-organs.”
➢ Another popular technology is the “organ-on-a-chip”: they are AA-battery-sized chips lined with
human cells connected to microchannels to mimic blood flow inside the body. These systems capture
several aspects of human physiology, including tissue-tissue interactions and physical and chemical
signals inside the body.
➢ Several innovations in the last decade now allow a 3D bioprinter to ‘print’ biological tissues using
human cells and fluids as ‘bio-ink.’ Such technologies, researchers say, are bringing us closer to
recreating a human tissue or organ system in the laboratory.
So, Statement 2 is correct.

109
ADDITIONAL INFORMATION:
DRUGS AND CLINICAL TRIAL (AMENDMENT) RULES, 2023
Recently in India takes the first step to remove animals from the drug-testing process. A new
News Rule authorizes researchers to use non-animal methods to test new drugs.
Status of ➢ In 2021, the European Union passed a resolution on an action plan to facilitate
Regulations the transition towards technologies that don’t use animals in research,
worldwide regulatory testing, and education.
➢ The U.S. passed the Food and Drug Administration (FDA) Modernization Act 2.0
in December 2022, allowing researchers to use new technologies to test the
safety and efficacy of new drugs.
➢ South Korea introduced a Bill called ‘Vitalization of Development,
Dissemination, and Use of Alternatives to Animal Testing Methods’ in December
2022.
➢ In June 2023, Canada amended its Environmental Protection Act to replace,
reduce or refine the use of vertebrate animals in toxicity testing.
➢ In March 2023, the Indian government embraced these systems in the drug-
development pipeline by amending the New Drugs and Clinical Trials Rules
2019.
• It did so after inviting comments from the people and in consultation with
the Drug Technical Advisory Board, the statutory body that advises
Central and State governments on drug-related technical matters.

100. With reference to the PM-DevINE scheme, consider the following statements:
1. It is a centrally sponsored scheme which launched in the union budget of 2022-23.
2. The main objective is to fund the infrastructure and social development projects in the
Northeastern region.
3. The scheme was implemented by the Ministry of Finance.
How many of the statements given above are correct?
(a) Only one
(b) Only two
(c) All three
(d) None
EXPLANATION:
Prime Minister’s Development Initiative for North Eastern Region (PM-DevINE) was announced as a new
Central Sector scheme (not a centrally sponsored scheme), with 100% Central funding in the Union
Budget 2022-23 with an initial list of seven projects and an initial allocation of Rs. 1500 crore. The PM-
DevINE scheme will be implemented by the Ministry of Development of the North Eastern Region (not by
the Ministry of Finance). So, Statements 1 and 3 are not correct.

110
The objectives of the PM-DevINE scheme are to:
➢ Fund infrastructure convergently, in the spirit of PM GatiShakti;
➢ Support social development projects based on the felt needs of the Northeastern Region (NER);
➢ Enable livelihood activities for youth and women;
➢ Fill the development gaps in various sectors.
So, Statement 2 is correct.

ADDITIONAL INFORMATION:
PRIME MINISTER’S DEVELOPMENT INITIATIVE FOR NORTH EASTERN REGION
Recently in Cabinet approves new Scheme “Prime Minister’s Development Initiative for North
News East Region (PM-DevINE) for the remaining four years of the 15th Finance
Commission from 2022-23 to 2025-26
About ➢ The PM-DevINE scheme has an outlay of Rs.6,600 crore for the 4 years from
2022-23 to 2025-26.
➢ It will lead to the creation of infrastructure, support industries, social
development projects and livelihood activities for youth and women, thus leading
to employment generation.
➢ NITI Aayog and the Ministry of Development of the North Eastern Region
(MDoNER), in collaboration with UNDP, have prepared the Northeastern Region
District SDG Index (Baseline Report 2021-22), which inter-alia points to the SDG
gaps at the district level in the Region.
➢ This Report (and its updates) would increasingly be used for project
identification and formulation under PM-DevINE.
➢ The State Governments will constitute a State Level Empowered Committee
(SLEC) chaired by the Chief Secretary and comprising Planning, Finance, and
other concerned Secretaries of the respective State Government, as deemed
necessary.

111

You might also like